Anda di halaman 1dari 177

Behavioral Science

How to Use the Workbook with the Videos

Video Part Questions Behavioral Science Epi/Biostats part 1 112 Behavioral Science Epi/Biostats part 2 1330 Behavioral Science Epi/Biostats part 3 3143 Behavioral Science Ethics 4473 Behavioral Science Development 7480 Behavioral Science Physiology 8192

file:///D|/USMLE%20Vids/Workbooks%202012/Chapter%201%20-%20Behavioral%20Science%202012.txt[8/26/2013 2:13:05 AM]

Questions

EPIDEMIOLOGY/BIOSTATISTICS 1. A case control study is described as _______________ (experimental/observational) and _______________ (prospective/retrospective). (p. 52) 2. A cohort study is described as _______________ (experimental/observational) and _______________ (prospective/retrospective). (p. 52) 3. True or False: In a cohort study, subjects are chosen on the basis of the presence or absence of risk factors. (p. 52) _______________________________________________________________ 4. True or False: A cohort study involves following subjects over a period of time to study the development of disease. (p. 52) _____________________________________________________ 5. True or False: A cross-sectional research study can show the correlation of a risk with a disease. (p. 52) _________________________________________________________________________ 6. Describe the two general types of blinded studies. (p. 52) _________________________________ ______________________________________________________________________________ 7. How does a low prevalence of disease affect the positive predictive value of a test? (p. 53) ____ ______________________________________________________________________________ 8. In HIV testing, ELISA screening is _______________ (sensitive/specific) and has a high false-_______________ (negative/positive) rate, with a _______________ (high/low) threshold. (p. 53) 9. In HIV testing, the Western blot test is _______________ (sensitive/specific) and has a high false_______________ (negative/positive) rate, with a _______________ (high/low) threshold. (p. 53) 10. How does a low prevalence of disease affect the negative predictive value of a test? (p. 53) ___ ______________________________________________________________________________ 11. If a diagnostic test has 100% sensitivity, what should the value of the false-negative rate equal? (p.53) __________________________________________________________________________ 12. If a diagnostic test has 100% specificity, what should the value of the false-positive rate equal? (p.53) __________________________________________________________________________ 13. What measure of disease frequency is calculated by dividing the total number of cases in the population at a given time by the total population at a given time? (p. 54) ____________________ 14. What measure of disease frequency is calculated by dividing the number of new cases in the population during a period by the total population at risk during that time? (p. 54) ______________ 15. In chronic disease states such as diabetes, is the prevalence of disease greater than, less than, or equal to the incidence? (p. 54) ______________________________________________________ 16. In acute disease states such as a common cold, is the prevalence of disease greater than, less than, or equal to the incidence? (p. 54) _______________________________________________ 17. The statement "Patients with COPD had higher odds of a history of smoking than those without COPD" pertains to _______________ (odds ratio / relative risk). (p. 54) 18. The statement "Smokers had a higher risk of developing COPD than did nonsmokers" pertains to _______________ (odds ratio / relative risk). (p. 54) 19. True or False: When calculating the incidence of a disease, the total population at risk during a certain time should include people who have the disease. (p. 54) ___________________________ 20. True or False: The odds ratio for a disease based on exposure to a risk factor approximates the relative risk if the prevalence of the disease is low. (p. 54) ________________________________ 21. What epidemiologic measurement is equal to the difference in disease incidence between an exposed group and an unexposed group? (p. 54) _______________________________________ 22. _______________ (Precision/accuracy) refers to the degree to which test measurements represent the true answer (validity), while _______________ (precision/accuracy) refers to the consistency and reproducibility of a test
file:///D|/USMLE%20Vids/Workbooks%202012/Chapter%201%20-%20Behavioral%20Science%202012.txt[8/26/2013 2:13:05 AM]

or the absence of random variation in a test. (p. 55) 23. _______________ (Random/systematic) error reduces precision in a test, while _______________ (random/systematic) error reduces accuracy in a test. (p. 55) 24. Name four ways to reduce bias in epidemiologic studies. (p. 56) ___________________________ ______________________________________________________________________________ 25. What type of bias occurs when there are two closely associated factors and therefore the effect of one factor could change or confuse the effect of the other? (p. 56) __________________________ 26. What is the name of the phenomenon whereby a researcher's belief in the efficacy of a treatment changes the outcome of that treatment? (p. 56) ________________________________________ 27. In a data set that has a distribution with a negative skew, what is the relationship between the mean, the median, and the mode? (p. 57) _____________________________________________ 28. In a data set that has a distribution with a positive skew, what is the relationship between the mean, the median, and the mode? (p. 57) __________________________________________________ 29. Which characteristic is least affected by outliers: mean, median, or mode? (p. 57) ______________ 30. What is the term for the hypothesis that there is no association between the variables being studied? (p. 57) _________________________________________________________________ 31. In statistical calculations, the p value is equal to the probability of making what type of error? (p. 57) ______________________________________________________________________________ 32. In a statistical analysis, if p = 0.03, what is the probability that the null hypothesis is correct? (p. 57) ______________________________________________________________________________ 33. In statistical analyses, if = 0.2, what is the probability that the null hypothesis has been falsely accepted? (p. 57) ________________________________________________________________ 34. The power of a statistical test depends on which three factors? (p. 58) _______________________ ______________________________________________________________________________ 35. What two major factors may limit the reliability of a meta-analysis? (p. 58) ____________________ 36. What study parameter is calculated when the probability of making a type II error is subtracted from 1? (p. 58) ______________________________________________________________________ 37. In a data set that has a normal (Gaussian) distribution, what percentage of the data falls within two standard deviations of the mean? What percentage falls within 3 SDs of the mean? (p. 58) ______________________________________________________________________________ 38. If the 95% confidence interval for a mean difference between two variables includes 0, the null hypothesis _______________ (is/is not) rejected. (p. 58) 39. What type of statistical test is used to check for a difference between the means of three or more groups? (p. 58) __________________________________________________________________ 40. What type of statistical test is used to check for a difference between the means of two groups? (p.58) __________________________________________________________________________ 41. What statistical term's absolute value indicates the strength of the correlation between two variables? (p. 58) ________________________________________________________________ 42. Given an example of primary, secondary, and tertiary disease prevention strategies. (p. 58) _____ ______________________________________________________________________________ 43. What two federally funded healthcare programs originated from amendments to the Social Security Act? Who is eligible for these programs? (p. 59) ________________________________________ ______________________________________________________________________________ ETHICS 44. What are the four core ethical principles of medicine? (p. 59) ______________________________ ______________________________________________________________________________ 45. True or False: Patient autonomy may conflict with beneficence. (p. 59) ______________________ 46. True or False: If the benefits of an intervention outweigh the risks, a patient may make an informed decision to proceed, thus overriding the ethical principle of nonmaleficence. (p. 59) ____________ 47. Which right is being exercised when a patient makes an informed decision to proceed with a medical treatment when the benefits of the intervention outweigh its risks? (p. 59) _____________ 48. What must be explained by a doctor to obtain informed consent? (p. 59) ____________________ ______________________________________________________________________________
file:///D|/USMLE%20Vids/Workbooks%202012/Chapter%201%20-%20Behavioral%20Science%202012.txt[8/26/2013 2:13:05 AM]

49. What are the four exceptions to informed consent? (p. 59) ________________________________ ______________________________________________________________________________ 50. What are four conditions in which a minor may be considered emancipated? (p. 59) ___________ ______________________________________________________________________________ 51. What are the five requirements that must be met in order to determine that a patient has full decision-making capacity? (p. 60) ___________________________________________________ ______________________________________________________________________________ 52. True or False: A patient's family can require that a doctor withhold information from the patient. (p.60) _________________________________________________________________________ 53. What term refers to an incapacitated patient's previous oral statements, which are commonly used to guide medical decisions? (p. 60) __________________________________________________ 54. What four factors give greater validity to a patient's oral advance directive? (p. 60) ___________ ______________________________________________________________________________ 55. What is the term for the legal document that describes treatments the patient wishes to receive or not receive if he/she becomes incapacitated and cannot communicate about treatment decisions? (p. 60) _________________________________________________________________________ 56. What legal term refers to a patient's designated surrogate who may make medical decisions in the event that the patient loses decision-making capacity? (p. 60) _____________________________ 57. True or False: When authorizing a power of attorney, the patient may specify decisions that are to be made in certain clinical situations. (p. 60) ___________________________________________ 58. True or False: A patient's agent authorized with power of attorney retains that power unless it is revoked by the patient. (p. 60) ______________________________________________________ 59. Which type of advanced directive provides greater flexibility, a living will or a durable power of attorney? (p. 60) _________________________________________________________________ 60. What standard should be used to determine the amount of medical information to disclose to a patient's family or friends? (p. 60) ___________________________________________________ 61. In the case of serious infectious diseases, a physician may have a duty to break patient confidentiality to warn certain groups of people. Name the two groups. (p. 60) ________________ ______________________________________________________________________________ 62. What legal precedent requires physicians to directly inform and protect a potential victim from harm, even if it involves a breach of confidentiality? (p. 60) ________________________________ 63. A child presents to the emergency department with multiple fractures and bruises of different ages. The patient's mother requests that authorities not be involved. Must the physician respect her request for confidentiality? Why or why not? (p. 60) _____________________________________ ______________________________________________________________________________ 64. A young woman confides to her physician that she has considered ending her life by ingesting a bottles worth of her prescription pills, and that she does not want anyone else to know of her plan. Must the physician respect her request for nondisclosure? Why or why not? (p. 60) ___________ ______________________________________________________________________________ 65. A patient discloses that he frequently drives after consuming four or five drinks at the bar. In the office, the patient clearly has been drinking alcohol and has arrived on his own. Is the physician obligated to uphold confidentiality? Why or why not? (p. 60) _______________________________ ______________________________________________________________________________ 66. What is an appropriate response to a patient who is upset about how he or she was treated by another doctor? (p. 61) ____________________________________________________________ 67. What is an appropriate response to a child who wishes to know more about his or her illness? (p.61) _________________________________________________________________________ 68. What is an appropriate response to a patient who is noncompliant? (p. 61) __________________ ______________________________________________________________________________ 69. What is an appropriate response to a 17-year-old girl who is pregnant and requests an abortion? (p. 61) _________________________________________________________________________ 70. What is an appropriate response to a terminally ill patient who requests physician assistance with ending his or her life? (p. 61) _______________________________________________________ 71. What is an appropriate response to a patient who states that he or she finds you attractive? (p. 61)
file:///D|/USMLE%20Vids/Workbooks%202012/Chapter%201%20-%20Behavioral%20Science%202012.txt[8/26/2013 2:13:05 AM]

______________________________________________________________________________ 72. What is an appropriate response to a patient who refuses a necessary procedure or desires an unnecessary one? (p. 61) __________________________________________________________ 73. What is an appropriate response to a patient who continues to smoke? (p. 61) ________________ ______________________________________________________________________________ DEVELOPMENT 74. What are the five criteria for calculating an Apgar score? (p. 62) ____________________________ 75. What are five major complications of low birth weight? (p. 62) _____________________________ ______________________________________________________________________________ 76. What is the approximate age of a child who is anxious when separated from his or her mother? (p.62) _________________________________________________________________________ 77. What is the approximate age range of a child who plays alongside, but not with, another child? (p.62) _________________________________________________________________________ 78. What sexual changes normally occur in elderly men? (p. 63) ______________________________ ______________________________________________________________________________ 79. True or False: One may experience hallucinations during normal bereavement. (p.63) __________ 80. Name three types of symptoms that a patient with pathologic grief may experience. (p. 63) _____ ______________________________________________________________________________ PHYSIOLOGY 81. What are four physiologic mechanisms that can be affected by stress? (p.63) ______________________________________________________________________________ 82. Which four classes of drugs or substances can cause sexual dysfunction? (p. 63) ___________ ______________________________________________________________________________ 83. Awake and alert states are characterized by what electroencephalogram waveform? (p. 64) ____ ______________________________________________________________________________ 84. REM sleep is characterized by what electroencephalogram waveform? (p. 64) _______________ ______________________________________________________________________________ 85. Which area of the brain is responsible for rapid eye movements during sleep? (p. 64) ___________ 86. Which drug is used to treat enuresis (bed-wetting)? What is its mechanism of action? (p. 64) ______________________________________________________________________________ 87. Which drug class is used to treat night terrors and sleepwalking? What is its mechanism of action? (p. 64) _________________________________________________________________________ 88. Name four physiologic changes that occur during REM sleep. (p. 64) _______________________ ______________________________________________________________________________ 89. What is the principle neurotransmitter involved in REM sleep? (p. 64) ______________________ ______________________________________________________________________________ 90. What sleep disorder is characterized by cataplexy and hypnagogic hallucinations? (p. 65) ______ ______________________________________________________________________________ 91. What is the most common medical treatment for narcolepsy? (p. 65) _______________________ ______________________________________________________________________________ 92. Circadian rhythms dictate the release of which hormones and neurotransmitters? (p. 65) _______ ______________________________________________________________________________ Answers EPIDEMIOLOGY/BIOSTATISTICS 1. Observational; retrospective. 2. Observational; prospective (more common) or retrospective. 3 True. 4. True for prospective cohort studies. 5. True. (However, it cannot show causality.)
file:///D|/USMLE%20Vids/Workbooks%202012/Chapter%201%20-%20Behavioral%20Science%202012.txt[8/26/2013 2:13:05 AM]

6. In single-blind studies, subjects are not aware of their study-group assignment. In double-blind studies, neither researchers nor subjects are aware of study-group assignment. 7. The positive predictive value of the test is lower for a disease that has a lower prevalence. 8. Sensitive; positive; low. 9. Specific; negative; high. 10. The negative predictive value of the test is higher for a disease that has a lower prevalence. 11. It should equal 0. (All cases of the disease are detected by the test.) 12. It should equal 0. (All patients without the disease are identified correctly.) 13. Point prevalence. 14. Incidence. (Incidence refers to new incidents). 15. Much greater than the annual incidence (because of the long duration of the disease). 16. Approximately equal (for diseases of short duration). 17. Odds ratio. 18. Relative risk. 19. False. (The total population at risk during a certain period should not include people who have the disease because incidence is a measure of new cases of a disease; those who have the disease are not at risk of getting the disease.) 20. True. 21. Attributable risk; it is the percentage of cases of a disease caused by a risk factor. 22. Accuracy; precision. 23. Random; systematic. 24. Use blinded studies (preferably double blind), assess placebo responses, perform a crossover study, and use randomization. 25. Confounding bias, in which the causal relationship is better explained by a variable other that the one being studied. 26. Pygmalion effect. 27. Mean < median < mode. 28. Mean > median > mode. 29. Mode. 30. Null hypothesis. 31. Type I error (a). (This type of error occurs when it is incorrectly concluded that an association is present.) 32. 3%. 33. 20%. (This is generally considered an acceptable level for in a study design). 34. The sample size, the difference in compliance between the treatment groups, and the size of the expected effect. 35. The quality of the individual studies, and bias in the selection of studies for analysis. 36. Power. 37. 95% fall within 2SDs of the mean and 99.7% fall within 3SDs of the mean. 38. If the 95% CI includes 0, the null hypothesis is not rejected as there is no significant difference between the two variables. 39. Analysis of variance (ANOVA). 40. t-test. 41. Correlation coefficient (r). 42. Vaccination is a primary disease prevention strategy, as the vaccinated person is not considered susceptible to the disease. A Pap smear is secondary disease prevention, since it can detect cervical cancer at an early, treatable stage. Chemotherapy is tertiary disease prevention, as it is intended to reduce disability from disease once it has occurred. 43. Medicare and Medicaid. MedicarE is for the Elderly, while MedicaiD is for the Destitute. ETHICS 44. Autonomy, beneficence, nonmaleficence, and justice. 45. True. 46. True. 47. Autonomy. 48. The risks and benefits of the proposed intervention, and the risks and benefits of the alternatives (including doing nothing). 49. If the patient lacks decision-making capacity; if the situation is emergent; if disclosure of information would harm the patient; and if the patient waives the right of informed consent.
file:///D|/USMLE%20Vids/Workbooks%202012/Chapter%201%20-%20Behavioral%20Science%202012.txt[8/26/2013 2:13:05 AM]

50. If the minor is married, self-supporting, has children, or is in the military. 51. The patient must make and communicate a choice; the patient must be informed; the decision must remain stable over time; the decision must be consistent with the patient's values and goals; and the decision cannot be a result of delusions or hallucinations. 52. False. (A patient's family cannot require the physician to withhold information from the patient.) 53. Oral advance directive. 54. The patient was informed, the directive is specific, the patient made a choice, and the decision was repeated over time. 55. Living will (i.e., written advance directive). 56. Durable power of attorney. 57. True. 58. True. 59. A durable power of attorney. 60. The disclosure of information to family or friends should be guided by what the patient would want. 61. Public officials and identifiable people who may be at risk. 62. The Tarasoff Decision. 63. No; a physician may break confidentiality to report the abuse (or suspected abuse) of a child or an elderly person. 64. No; a physician may break confidentiality to report a suicidal or homicidal patient. 65. No; a physician can break confidentiality to report an impaired driver. 66. Suggest that the patient speak directly to that physician about the concerns. If the problem is with a member of the office staff, inform the patient that you will speak to that person. 67. Ask the parents what they have told the child about his or her illness. (The parents decide what information should be relayed to the child about the illness.) 68. Attempt to identify the patients reason for noncompliance and whether they are willing to change their behavior. Do not force the patient to comply or refer the patient to another physician. 69. Many states require parental notification or consent for minors to have an abortion. However, parental consent is not required for emergency situations, the treatment of sexually transmitted diseases, medical care during pregnancy, prescriptions for contraceptives, or the management of drug addiction. 70. In most states, physicians should refuse to be involved in any form of assisted suicide; however, the physician may prescribe medically appropriate analgesics that coincidentally shorten the patient's life. 71. Ask direct closed-ended questions, and use a chaperone if necessary. (Romantic relationships with patients are never appropriate.) 72. Attempt to understand why the patient wants or does not want the procedure; address the underlying concerns, and avoid performing unnecessary procedures. 73. Ask how the patient feels about smoking; offer advice about cessation if the patient seems willing to make an effort to quit. DEVELOPMENT 74. Appearance, Pulse, Grimace, Activity, Respiration 75. Infection, neonatal respiratory distress syndrome, necrotizing enterocolitis, intraventricular hemorrhage, and persistent fetal circulation. 76. 15 months. 77. 24 to 48 months (parallel play). 78. Slower erection/ejaculation and a longer refractory period. 79. False. 80. Depressive symptoms, delusions, and hallucinations. PHYSIOLOGY 81. Water absorption, muscular tonicity, gastrocolic reflex, and mucosal circulation. 82. Antihypertensives, neuroleptics, selective serotonin reuptake inhibitors, ethanol. 83. Beta (highest frequency, lowest amplitude). 84. Beta. 85. The paramedian pontine reticular formation/conjugate gaze center. 86. Imipramine; it shortens the duration of stage 3 sleep. 87. Benzodiazepines; they shorten REM and delta sleep. 88. Increased and variable pulse, rapid eye movement, increased and variable blood pressure, and penile/clitoral
file:///D|/USMLE%20Vids/Workbooks%202012/Chapter%201%20-%20Behavioral%20Science%202012.txt[8/26/2013 2:13:05 AM]

tumescence. 89. Acetylcholine. 90. Narcolepsy. 91. Stimulants (such as amphetamines or modafinil). 92. ACTH, prolactin, melatonin, and nocturnal norepinephrine.

file:///D|/USMLE%20Vids/Workbooks%202012/Chapter%201%20-%20Behavioral%20Science%202012.txt[8/26/2013 2:13:05 AM]

Biochemistry

How to Use the Workbook with the Videos

Video Part Questions Biochemistry Molecular part 1 18 Biochemistry Molecular part 2 916 Biochemistry Molecular part 3 1727 Biochemistry Cellular part 1 2837 Biochemistry Cellular part 2 3849 Biochemistry Laboratory Techniques 5057 Biochemistry Genetics part 1 5862 Biochemistry Genetics part 2 6372 Biochemistry Genetics part 3 7379 Biochemistry Nutrition part 1 80 Biochemistry Nutrition part 2 80 Biochemistry Nutrition part 3 8081 Biochemistry Metabolism part 1 7, 8284 Biochemistry Metabolism part 2 8587

file:///D|/USMLE%20Vids/Workbooks%202012/Chapter%202%20-%20Biochemistry%202012.txt[8/26/2013 2:13:07 AM]

Biochemistry Metabolism part 3 88 Biochemistry Metabolism part 4 89 Biochemistry Metabolism part 5 9093 Biochemistry Metabolism part 6 9498

Questions

MOLECULAR 1. Which histone is not part of the nucleosome core? (p. 68) ________________________________ 2. What is DNA called when it is condensed and transcriptionally inactive? (p. 68) _______________ 3. What is the name for transcriptionally active DNA? (p. 68)_________________________________ 4. What physical property is displayed by nucleic acids that have increasing proportions of guanine- cytosine bonds? (p. 68) ___________________________________________________________ 5. Which enzyme is inhibited by hydroxyurea? (p. 69) ______________________________________ 6. 5-Fluorouracil inhibits _______________, whereas both methotrexate and trimethoprim inhibit _______________. (p. 69) 7. A child has megaloblastic anemia not responsive to vitamin B12 or folate administration. She is small for her age. Serum studies show increased orotic acid and hyperammonemia. What is the most likely diagnosis? (pp. 69, 111) __________________________________________________ 8. A 12-year-old boy with moderate mental retardation visits his physician because of painful swollen joints. During the examination, the boy makes several uncontrolled spastic muscle movements. His medical history includes muscular hypotonia diagnosed when he was 5 months old. When he was 3 years old he was referred to a pediatric dentist for severe repetitive biting of his lip and tongue. What is the most likely diagnosis? (p. 70) _____________________________________________ 9. Silent mutations often result from changes in which position of a codon? (p. 71) _______________ 10. What kind of mutation denotes a DNA change that results in the misreading of all nucleotides downstream from it? (p. 71) ________________________________________________________ 11. What enzyme proofreads DNA synthesis with its exonuclease activity in prokaryotic DNA replication? In which direction can it remove nucleotides? (p. 72) ___________________________ 12. What enzyme degrades the RNA primer and replaces it with DNA during prokaryotic DNA replication? (p. 72) _______________________________________________________________ 13. Which category of drugs inhibits DNA gyrase? (p. 72) ____________________________________ 14. What specific DNA repair mechanism is defective in xeroderma pigmentosum? (p. 73) __________ ______________________________________________________________________________ 15. In single-stranded DNA repair, how are nucleotide-excision and base-excision repair different? (p.73) _________________________________________________________________________ 16. Hereditary nonpolyposis colorectal cancer results from the loss of which DNA repair mechanism? (p.73) _________________________________________________________________________

file:///D|/USMLE%20Vids/Workbooks%202012/Chapter%202%20-%20Biochemistry%202012.txt[8/26/2013 2:13:07 AM]

17. What type of RNA is the longest? The smallest? The most abundant? (p. 74) _________________ ______________________________________________________________________________ 18. What commonly results from a mutation within a promoter? (p. 75) _________________________ 19. What poisonous protein that inhibits RNA polymerase II is found in death cap mushrooms? (p. 75) ______________________________________________________________________________ 20. In eukaryotes, what enzyme makes mRNA? (p. 75) _____________________________________ 21. In eukaryotes, what enzyme makes tRNA? (p. 75) ______________________________________ 22. Patients with which disease make antibodies to spliceosomal small nuclear ribonucleoproteins? (p.76)__________________________________________________________________________ 23. -Thalassemia is due to a mutation causing splicing defects in a process that combines different exons within a single gene. What mechanism allows the same gene to encode for various different proteins? (p. 77) _________________________________________________________________ 24. How does tetracycline interfere with protein translation? (p. 78, 208) _______________________ 25. How do aminoglycosides interfere with protein synthesis? (p. 78, 209) ______________________ 26. Why are antibiotics such as chloramphenicol, clindamycin, and macrolides selective for bacteria and not toxic to human cells? (p. 78) _________________________________________________ 27. Explain the three steps of protein elongation. (p. 78) _____________________________________ ______________________________________________________________________________ ______________________________________________________________________________ CELLULAR 28. Fill in the boxes on the image below, noting the phases of the mitotic cell cycle. (p. 79) 29. Which transition in the cell cycle is prevented by Rb and p53 tumor suppressors? (p. 79) ________ ______________________________________________________________________________ 30. Match the cell type with its description. (p. 79) _____ A. Cells remain in G0 and regenerate from stem cells 1. Labile cells _____ B. Enter G1 from G0 when stimulated 2. Permanent cells _____ C. Never go to G0 and divide rapidly with a short G1 3. Stable (quiescent) cells 31. Name two cells that are rich in rough endoplasmic reticulum. (p. 79) ________________________ 32. Name two cells that are rich in smooth endoplasmic reticulum. (p. 79) _______________________ 33. A child presents with coarse facial features, clouded corneas, restricted joint movement, and high plasma levels of lysosomal enzymes. What is the most likely diagnosis? (p. 80) _______________ 34. What syndrome results from a microtubule polymerization defect that leads to impaired lysosomal emptying and poor phagocytosis? (p. 81) _____________________________________________ 35. Which anticancer drug used to treat breast cancer can stabilize microtubules? (p. 81) __________ 36. Which antifungal agent targets microtubules? (p. 81) ____________________________________ 37. Which antihelminthic drugs target microtubules? (p. 81) __________________________________ 38. A 22-year-old woman presents with a history of recurrent pneumonia. X-ray of the chest shows dextrocardia. What is the most likely diagnosis? (p. 81) __________________________________ 39. Cilia, flagella, mitotic spindles, neurons, and centrioles are composed of which cytoskeletal element? (p. 81) _________________________________________________________________ 40. Vimentin, desmin, cytokeratin, glial fibrillary acid proteins, and neurofilaments are examples of which type of cytoskeletal element? (p. 81) ____________________________________________ 41. What effect does digoxins inhibition of Na+-K+ ATPase have on cardiac contractility? (p. 82) _____ ______________________________________________________________________________ 42. What type of collagen is found in each structure? (p. 82) _____ A. Basement membrane or basal lamina 1. Type I collagen _____ B. Bone, skin, tendon, dentin, fascia, cornea, 2. Type II collagen late wound repair 3. Type III collagen _____ C. Cartilage, vitreous body, nucleus pulposus 4. Type IV collagen _____ D. Skin, blood vessels, uterus, fetal tissue, granulation tissue 43. British sailors in the 17th century were often unable to hydroxylate proline and lysine residues for collagen synthesis. What disease did they have, and why did the treatment work? (p. 83)
file:///D|/USMLE%20Vids/Workbooks%202012/Chapter%202%20-%20Biochemistry%202012.txt[8/26/2013 2:13:07 AM]

______________________________________________________________________________ 44. What disease leads to an inability to form procollagen from pro a chains? (pp. 83) _____________ 45. A patient presents with hyperextensible skin, easy bruising, and hypermobile joints. What is the most likely diagnosis? (p. 83) ___________________________________________________ 46. A baby is born with multiple fractures and hearing loss. What finding would most likely be seen during the ophthalmologic examination? (p. 83) ________________________________________ 47. What is the relationship between Alport's syndrome and Goodpasture's syndrome? (pp. 84, 516) ______________________________________________________________________________ 48. Marfan's syndrome is caused by a defect in what protein? (p. 84) __________________________ 49. Which lung disorder can result from excess elastase activity? (p. 84) ________________________ LABORATORY TECHNIQUES 50. Why cant you use human DNA polymerase for PCR? (p. 84) ______________________________ 51. Which would travel further on an agarose gel, a fragment of 10 kD or of 100 kD, and why? (p. 84) ______________________________________________________________________________ 52. Describe each of the following blot techniques: Southern, Northern and Western. (p. 85) Southern blot: ___________________________________________________________________ ______________________________________________________________________________ Northern blot: __________________________________________________________________ ______________________________________________________________________________ Western blot: ___________________________________________________________________ ______________________________________________________________________________ 53. Which assay can measure the expression level of many genes simultaneously? (p. 85) _________ _______________________________________________________________________________ 54. What information can be obtained from an ELISA using a test antibody coupled to a color-generating enzyme? (p. 85) ________________________________________________________ 55. What is the advantage of FISH over karyotyping? (pp. 85-86) _____________________________ ______________________________________________________________________________ 56. A DNA fragment is added to four different tubes along with DNA polymerase; a radiolabeled primer; and the adenine, thymine, cytosine, and guanine deoxynucleotides. Each tube also contains one of the four bases as dideoxynucleotides. The four tubes are then run on electrophoresis gel and visualized by autoradiography. Which laboratory technique does this describe? (p. 86) _________ ______________________________________________________________________________ 57. What is the most direct lab technique for detecting autosomal trisomies? (p. 86) _____________ ______________________________________________________________________________ GENETICS 58. Explain loss of heterozygosity. (p. 87) ________________________________________________ ______________________________________________________________________________ 59. A genetic disease that shows _______________________________ may have mutations at one of several different loci that produce the same phenotype. (p. 87) 60. In terms of p and q, what is the heterozygote prevalence in a population that is in Hardy-Weinberg equilibrium? (p. 88) ___________________________________________________________ 61. How is Prader-Willi syndrome inherited? What are the symptoms? (p. 88) ____________________ ______________________________________________________________________________ 62. How is Angelmans syndrome inherited? What are the symptoms? (p. 88) ____________________ ______________________________________________________________________________ 63. Which mode of inheritance is represented by each pedigree? (p. 89) _____ A. _____ B. _____ C. _____ D. _____ E. 1. Autosomal dominant 2. Autosomal recessive 3. Mitochondrial inheritance 4. X-linked dominant 5. X-linked recessive
file:///D|/USMLE%20Vids/Workbooks%202012/Chapter%202%20-%20Biochemistry%202012.txt[8/26/2013 2:13:07 AM]

64. What percentage of sons of a carrier mother are expected to inherit an X-linked recessive disease? (p. 89) _________________________________________________________________________ 65. True or False: A mother with an X-linked dominant disease may pass the disease to her sons but not to her daughters. (p. 89) ________________________________________________________ 66. Achondroplasia is the result of a defect in which receptor? (p. 90) __________________________ 67. Adult polycystic kidney disease is inherited in an autosomal- _______________ (dominant/ recessive) pattern, whereas infantile polycystic kidney disease is associated with an autosomal-_______________ (dominant/recessive) pattern. (p. 90) 68. A patient has multiple telangiectasias and skin discolorations, and notes recurrent epistaxis. What vascular pathology would you expect to find? (p. 90) ___________________________________ 69. What is an effective cure for hereditary spherocytosis? (p. 90) ____________________________ 70. A tall man comes to the emergency room with the combination of a dissecting ascending aorta and acute mitral valve prolapsed. What disorder best explains these findings? (p. 90) ______________ 71. A patient has hemangioblastomas of the retina and cerebellum, as well as bilateral renal cell carcinoma. What is the most likely diagnosis? (p. 90) ____________________________________ 72. What renal findings might distinguish a patient with tuberous sclerosis from a patient with adult polycystic kidney disease? (p. 90) ___________________________________________________ 73. Cystic fibrosis results from a defect in which gene? Which chromosome? Which ion channel? (p.91) _________________________________________________________________________ 74. Which two pathogens are the most likely to cause recurrent pulmonary infections in a patient with cystic fibrosis? (p. 91) _____________________________________________________________ 75. A patient with cystic fibrosis has an increased risk a deficiency of which vitamins? (p. 91) ______________________________________________________________________________ 76. A 4-year-old boy needs to use his upper extremities to push against his legs in order to stand up. What maneuver is he using? (p. 91) _________________________________________________ 77. A male patient has a long face, a large jaw, large ears, autism, and macroorchidism. What is the most likely diagnosis? (p. 92) _______________________________________________________ 78. Before his anticipated death, a 42-year-old man had received many years of treatment for depression, severe cognitive decline, and involuntary writhing movements. His father had similar symptoms shortly before his death. What is the cause of this patients most likely disease? (p. 92) ______________________________________________________________________________ 79. A newborn is diagnosed with Down syndrome. She is vomiting bilious material. What is the most likely cause? (p. 93) ______________________________________________________________

NUTRITION 80. Match each set of symptoms/conditions with the vitamin that is deficient. (pp. 95-99) _____ A. Bruising, anemia, swollen gums, and poor wound healing 1. Biotin _____ B. Cheilosis and corneal vascularization 2. Folic acid _____ C. Convulsions, hyperirritability, peripheral neuropathy, 3. Vitamin A and sideroblastic anemia 4. Vitamin B1 _____ D. Dermatitis, enteritis, and alopecia 5. Vitamin B2 _____ E. Dermatitis, enteritis, alopecia, and adrenal insufficiency 6. Vitamin B3 _____ F. Diarrhea, dermatitis, and dementia 7. Vitamin B5 _____ G. Hemolytic anemia, muscle weakness, and neuropathy 8. Vitamin B6 _____ H. Hemorrhagic disease of the newborn 9. Vitamin B12 _____ I. Hypocalcemic tetany 10. Vitamin C _____ J. Macrocytic, megaloblastic anemia with no 11. Vitamin D neurologic problems 12. Vitamin E
file:///D|/USMLE%20Vids/Workbooks%202012/Chapter%202%20-%20Biochemistry%202012.txt[8/26/2013 2:13:07 AM]

_____ K. Macrocytic, megaloblastic anemia, subacute combined 13. Vitamin K degeneration, paresthesia, and optic neuropathy _____ L. Night blindness _____ M. Wernicke-Korsakoff syndrome 81. Which vitamin or mineral is a cofactor for many transcription factors, and has dysgeusia as a symptom of its deficiency? (p. 99) ___________________________________________________

METABOLISM 82. Fill in the rectangles in the image below. Which steps of glycolysis are reversible (add one- or two- sided arrows to squares)? (p. 102) 83. Match each of the following processes with its rate-determining enzyme. (pp. 101-102) _____ A. Cholesterol synthesis 1. Acetyl-CoA carboxylase _____ B. De novo purine synthesis 2. Carbamoyl phosphate synthetase I _____ C. De novo pyrimidine synthesis 3. Carbamoyl phosphate synthetase II _____ D. Fatty acid oxidation 4. Carnitine acyltransferase I _____ E. Fatty acid synthesis 5. Fructose-1,6-bisphosphatase _____ F. Glycogen synthesis 6. G6PD _____ G. Glycolysis 7. Glutamine-PRPP amidotransferase _____ H. Gluconeogenesis 8. Glycogen phosphorylase _____ I. Glycogenolysis 9. Glycogen synthase _____ J. HMP shunt 10. HMG-CoA reductase _____ K. Ketogenesis 11. HMG-CoA synthase _____ L. TCA cycle 12. Isocitrate dehydrogenase _____ M. Urea cycle 13. Phosphofructokinase-1 84. How many ATP molecules can be created by the aerobic and anaerobic metabolism of glucose? (p. 103) ________________________________________________________________________ 85. The a-ketoglutarate dehydrogenase complex and the pyruvate dehydrogenase complex require the same 5 cofactors; what are these cofactors? (pp. 104-105)________________________________ _______________________________________________________________________________ 86. List the four irreversible enzymes in gluconeogenesis. (p. 106) ____________________________ _______________________________________________________________________ 87. What are Heinz bodies? What are bite cells? (pp. 108, 378-379) ___________________________ _______________________________________________________________________________ 88. Fill in the boxes on the image below, noting the substrates of the urea cycle. (p. 110) 89. Fill in the boxes on the image below, noting the enzymes that catalyze each step of catecholamine synthesis. (p. 112)

file:///D|/USMLE%20Vids/Workbooks%202012/Chapter%202%20-%20Biochemistry%202012.txt[8/26/2013 2:13:07 AM]

90. A 16-year-old boy presents for a routine visit. Physical examination shows symptoms consistent with Fabry's diseases. What is the inheritance pattern of this disease? (p. 116) ______________________________________________________________________________ 91. For each lysosomal storage disease in the chart below, name the enzyme and accumulated substrate(s). (p. 116) Disease Deficient Enzyme Accumulated Substance(s) Fabrys disease

Gauchers disease

Hunters syndrome

Hurlers syndrome

Krabbes disease

Metachromatic leukodystrophy

Niemann-Pick disease

Tay-Sachs disease

92. Most mucopolysaccharidoses are inherited in a(n) _______________-_______________ pattern; the exception is _______________. (p. 116) 93. Most sphingolipidoses are inherited in a(n) _______________-_______________ pattern; the exception is _______________. (p. 116) 94. Which cell-type(s) cannot use ketones as an energy source? Why? (p. 119) __________________ _______________________________________________________________________________ 95. Which apolipoprotein binds to the LDL receptor? On which lipoproteins is it found? (p. 120) ______ ______________________________________________________________________________
file:///D|/USMLE%20Vids/Workbooks%202012/Chapter%202%20-%20Biochemistry%202012.txt[8/26/2013 2:13:07 AM]

96. What is the target enzyme of the lovastatin? (p. 120) ____________________________________ 97. Match the lipoprotein with its function. (p. 121) _____ A. Delivers dietary TGs to peripheral tissues 1. Chylomicron _____ B. Delivers hepatic cholesterol to peripheral tissues 2. HDL _____ C. Delivers hepatic TGs to peripheral tissues 3. IDL _____ D. Delivers TGs and cholesterol to liver 4. LDL _____ E. Mediates cholesterol transport from periphery to liver 5. VLDL 98. In which organ(s) is HDL produced? (p. 121) __________________________________________

Answers MOLECULAR 1. H1. 2. Heterochromatin. 3. Euchromatin. (It is less condensed and accessible to transcription factors.) 4. Increasing melting temperatures due to the tighter binding of the strands. 5. Ribonucleotide reductase. 6. Thymidylate synthase; dihydrofolate reductase. 7. Ornithine transcarbamylase deficiency. (Orotic aciduria does not have hyperammonemia.) 8. This child has Lesch-Nyhan syndrome, which is characterized by mental retardation, self-mutilation, aggression, hyperuricemia, gout, and choreoathetosis. It is caused by the absence of HGPRT, which leads to a defective purine salvage pathway. 9. The third position (due to tRNA wobble). 10. Frameshift. 11. DNA polymerase III has 3' to 5' exonuclease activity for proofreading. 12. DNA polymerase I. 13. Fluoroquinolones. (They impair bacterial DNA synthesis.) 14. Nucleotide excision repair. 15. During nucleotide repair, the entire nucleotide structure is removed and replaced. During base excision repair, the base is clipped off of the sugar and repaired without the entire backbone of the DNA being taken apart. 16. The mismatch repair system. 17. mRNA is the longest type, tRNA is the smallest, and rRNA is the most abundant type of RNA. 18. A significant decrease in gene transcription. 19. a-Amanitin. (When consumed, it causes liver failure.) 20. RNA polymerase II. 21. RNA polymerase III. 22. Lupus. 23. Alternative splicing. (In this case, the alternative splicing yields a pathologic protein.) 24. Tetracycline binds to the 30S subunit of the ribosome, blocking attachment of the aminoacyl tRNA. 25. Aminoglycosides inhibit formation of the initiation complex, causing misreading of mRNA. 26. They affect the 50S subunit of the ribosome, which is found in prokaryotic cells. Eukaryotes have 60S and 40S subunits. 27. (1) Aminoacyl-tRNA binds to the A site. (2) Peptidyltransferase adds a peptide to the amino acid chain at site A. (3) The ribosome advances three nucleotides in the 3' direction, thereby moving the peptidyl tRNA to the P site. CELLULAR 28.

file:///D|/USMLE%20Vids/Workbooks%202012/Chapter%202%20-%20Biochemistry%202012.txt[8/26/2013 2:13:07 AM]

29. Transition from G1 to S phase. (Defective cells are not allowed to undergo DNA synthesis.) 30. A-2, B-3, C-1. 31. Goblet cells (secrete mucus) and plasma cells (secrete antibodies). 32. Liver hepatocytes and steroid-hormoneproducing cells of the adrenal cortex. 33. I-cell disease. 34. Chdiak-Higashi syndrome. 35. Paclitaxel. 36. Griseofulvin. 37. Mebendazole and thiabendazole. 38. Kartagener's syndrome. 39. Microtubules. 40. Intermediate filaments. 41. It increases cardiac contractility by increasing intracellular calcium concentration. 42. A-4, B-1, C-2, D-3. 43. Scurvy; the limes supplied the sailors with the vitamin C they needed during their long voyage (and earned them the nickname "limeys"). 44. Osteogenesis imperfecta. 45. Ehlers-Danlos syndrome. 46. Blue sclerae. 47. Both are diseases of type IV collagen. (Alport's is genetic; Goodpasture's is autoimmune.) 48. Fibrillin. 49. Emphysema. (a1-Antitrypsin inhibits elastase, which degrades elastin; therefore, lack of a1-antitrypsin can lead to loss of elastin in the lungs, thereby resulting in emphysema.) LABORATORY TECHNIQUES 50. The DNA Polymerase in PCR needs to be stable at high temperatures, since you need to repeatedly melt the DNA into a single-stranded form. Therefore, the heat stable polymerase from the thermophilic bacteria Thermos aquaticus (Taq polymerase) is used. 51. A fragment of 10 kD; smaller molecules travel further since they are less hindered by the gel. 52. Southern: DNA is run on an electrophoresis gel and transferred to a filter; the DNA on the filter is denatured and exposed to a labeled DNA probe; the double-stranded DNA is visualized when the filter is exposed to film. Northern: RNA is run on an electrophoresis gel and transferred to a filter; the RNA on the filter is exposed to a labeled DNA probe; the hybrid DNA-RNA molecule is visualized when the filter is exposed to film. Western: Sample protein is separated via gel electrophoresis and transferred to a filter; labeled antibody is used to bind and detect the protein of interest. 53. Microarrays. 54. An ELISA using a test antibody can determine whether a particular antigen is present in the patient's blood. 55. FISH allows for identification of anomalies at a molecular level, including deletions that are too small to see on a karyotype. 56. Sequencing. 57. Karyotype. GENETICS 58. When a tumor suppressor gene has only one functional allele due to an inherited mutation or deletion of the other, and then a cell loses the remaining allele as well. Consequently, having no functional alleles permits a neoplastic transformation. 59. Locus heterogeneity. 60. 2pq. 61. Prader-Willi syndrome is inherited via deactivation of the paternal copies of genes on chromosome 15, or can occur via uniparental disomy. Symptoms include mental retardation, hyperphagia, obesity, hypogonadism, and hypotonia. 62. Angelmans syndrome is inherited via deactivation of the maternal copies of genes on chromosome 15, or can occur via uniparental disomy. Symptoms include mental retardation, seizures, ataxia, and inappropriate laughter (happy puppet). 63. A-5, B-1, C-2, D-3, E-4. 64. 50%.
file:///D|/USMLE%20Vids/Workbooks%202012/Chapter%202%20-%20Biochemistry%202012.txt[8/26/2013 2:13:07 AM]

65. False. (Her sons and daughters may be affected.) 66. Fibroblast growth factor receptor 3. 67. Dominant; recessive. 68. Arteriovenous malformations. (This constellation of symptoms is typical of hereditary hemorrhagic telangiectasia.) 69. Refractory hereditary spherocytosis can be cured by splenectomy. 70. Marfan's syndrome. (It is associated with cystic medial necrosis of the aorta [which may lead to a dissection] as well as a floppy mitral valve prone to prolapse.) 71. von Hippel-Lindau disease. 72. Although both will have renal cysts, tuberous sclerosis is associated with renal angiomyolipomas. 73. CF is due to a defect in the CFTR gene on chromosome 7 that affects the chloride channel. 74. Pseudomonas species and Staphylococcus aureus. 75. Vitamins A, D, E, and K (all of which are fat soluble). 76. Gowers maneuver. (This action is necessary due to weakness of the proximal muscles.) 77. Fragile X syndrome. (Remember: Fragile X = eXtra large testes, jaw, and ears.) 78. The patient has classic symptoms of Huntingtons disease, which is caused by an expansion of CAG trinucleotide repeats on chromosome 4. 79. Duodenal atresia. NUTRITION 80. A-10, B-5, C-8, D-1, E-7, F-6, G-12, H-13, I-11, J-2, K-9, L-3, M-4. 81. Zinc. METABOLISM 82. 83. A-10, B-7, C-3, D-4, E-1, F-9, G-13, H-5, I-8, J-6, K-11, L-12, M-2. 84. Aerobic glucose metabolism produces 32 ATP molecules per molecule of glucose in heart and liver and 30 in muscle. Anaerobic glucose metabolism products only 2 net ATP molecules per molecule of glucose. 85. Vitamins B1, B2, B3, and B5 and lipoic acid. 86. Pyruvate carboxylase, PEP carboxykinase, fructose-1,6-bisphosphatase, and glucose-6-phosphatase. 87. Heinz bodies are clumps of denatured hemoglobin precipitates that stick to an RBCs membrane. When the RBC passes through the spleen, macrophages remove the Heinz bodies, which make the cell look as if a bite were taken out of it. 88.

89. 90. X-linked recessive. 91. Disease Deficient Enzyme Accumulated Substance(s)
file:///D|/USMLE%20Vids/Workbooks%202012/Chapter%202%20-%20Biochemistry%202012.txt[8/26/2013 2:13:07 AM]

Fabrys disease a-galactosidase A Ceramide trihexoside Gauchers disease -glucocerebrosiderase Glucocereboside Hunters syndrome Iduronate sulfatase Heparan sulfate, dermatan sulfate Hurlers syndrome a-L-iduronidase Heparan sulfate, dermatan sulfate Krabbes disease Galactocerebrosidase Galactocerebroside Metachromatic leukodystrophy Arylsulfatase A Cerebroside sulfate Niemann-Pick disease Sphingomyelinase Sphingomyelin Tay-Sachs disease Hexosaminidase A GM2 ganglioside

92. Autosomal-recessive; Hunter's disease. 93. Autosomal-recessive; Fabry's disease. 94. Erythrocytes, because they have no mitochondria. 95. B-100; VLDL, IDL, LDL, HDL 96. HMG-CoA reductase 97. A-1, B-4, C-5, D-3, E-2. 98. Liver and intestine.

file:///D|/USMLE%20Vids/Workbooks%202012/Chapter%202%20-%20Biochemistry%202012.txt[8/26/2013 2:13:07 AM]

Embryology

How to Use the Workbook with the Videos

Video Part Questions Embryology part 1 17 Embryology part 2 816 Embryology part 3 1718 Embryology part 4 1936 Embryology part 5 3741 Embryology part 6 4254

file:///D|/USMLE%20Vids/Workbooks%202012/Chapter%203%20-%20Embryology%202012.txt[8/26/2013 2:13:09 AM]

Questions

1. At what time during embryonic development does organogenesis occur? (p. 124) _____________ 2. During what week of development does the heart of an embryo begin to beat? (p. 124) _________ 3. During embryonic development, when does implantation occur? (p. 124) ____________________ 4. The alar plate is on the _______________ (dorsal/ventral) side of the neural tube, whereas the basal plate is on the _______________ (dorsal/ventral) side. (p. 124) 5. What are the functions of neurons derived from the alar and basal plates? (p. 124) ___________ ______________________________________________________________________________ 6. During fetal development, bones generally are derived from the _______________ (ectoderm/ endoderm/mesoderm), except for bones of the skull, which arise from _______________. (p. 126) 7. An infant is born with vertebral defects, anal atresia, and limb defects. What other defects is this patient most likely to experience? (p. 126) _____________________________________________ 8. A 2-week-old embryo is exposed to a teratogen; what is the likely effect on development? (p. 127) ______________________________________________________________________________ 9. Match each teratogen with its most common effect(s). (p. 127) _____ A. ACE inhibitors 1. Abnormal fetal development, fetal addiction _____ B. Alcohol 2. Absence of digits, multiple anomalies _____ C. Alkylating agents 3. Bone deformities, fetal hemorrhage, abortion _____ D. Aminoglycosides 4. Caudal regression syndrome _____ E. Carbamazepine 5. Cleft palate, cardiac abnormalities _____ F. Cocaine 6. CN VIII toxicity _____ G. Diethylstilbestrol (DES) 7. Congenital goiter or hypothyroidism _____ H. Folate antagonists 8. Discolored teeth _____ I. Iodine (lack or excess) 9. Ebsteins anomaly _____ J. Lithium 10. Flipper limb _____ K. Maternal diabetes 11. Inhibited intestinal folate absorption _____ L. Phenytoin 12. Leading cause of birth defects in U.S. _____ M. Smoking (nicotine, CO) 13. Microcephaly, mental retardation _____ N. Tetracyclines 14. Neural tube defects _____ O. Thalidomide 15. Preterm labor, placental problems _____ P. Valproate 16. Renal damage _____ Q. Vitamin A excess 17. Vaginal clear cell carcinoma _____ R. Warfarin 18. Fetal hydantoin syndrome _____ S. X-rays 19. Neural tube defects, craniofacial defects, fingernail hypoplasia, developmental delay, IUGR 10. What congenital abnormalities are associated with fetal alcohol syndrome? (p. 127) ___________ ______________________________________________________________________________ 11. How many chorions and amniotic sacs do conjoined twins have? (p. 128) ____________________ 12. If a zygote splits between day 3 and day 8, what type of twins will result? (p. 128) ______________ 13. How many arteries and how many veins are contained in the umbilical cord? (p. 129) ___________ 14. The umbilical arteries carry _______________ (oxygenated/deoxygenated) blood. (p. 129) 15. What structure connects the fetal bladder with the yolk sac? (p. 129) ________________________ 16. What is the term for a fistula between the umbilicus and terminal ileum? (p. 129) ______________ 17. In the embryonic heart, the right common cardinal vein and the right anterior cardinal vein jointly give rise to which vein in the adult? (p. 130) ___________________________________________
file:///D|/USMLE%20Vids/Workbooks%202012/Chapter%203%20-%20Embryology%202012.txt[8/26/2013 2:13:09 AM]

18. Which two cardiac structures are created by migration of neural crest cells? (p. 130) __________ ______________________________________________________________________________ 19. Which embryonic shunt diverts oxygenated blood from the inferior vena cava into the left atrium? (p. 132) ________________________________________________________________________ 20. Which embryonic shunt directs oxygenated blood around the hepatic circulation? (p. 132) ______ ______________________________________________________________________________ 21. Which embryonic shunt bypasses the high-resistance pulmonary circulation? (p. 132) _________ ______________________________________________________________________________ 22. What is the approximate oxygen saturation (%) of the blood returning from the placenta in the umbilical vein? (p. 132) ___________________________________________________________ 23. Which drug can close the ductus arteriosus? Which drug can keep it open? (p. 132) ___________ ______________________________________________________________________________ 24. What adult structure is derived from the cavity in the embryonic metencephalon and myelencephalon? (p. 133) _________________________________________________________ 25. Neural tube defects are associated with low intake of what substance during pregnancy? (p. 133) ______________________________________________________________________________ 26. Neural tube defects are associated with elevated levels of what substance in amniotic fluid and maternal serum? (p. 133) __________________________________________________________ 27. In the fetus, failure of the bony spinal canal to close (without structural herniation) will result in what type of neural tube defect? (p.133) __________________________________________________ 28. A ___________ is characterized by the meninges herniating through a spinal canal defect. (p.133) 29. A _______________ is characterized by the meninges and spinal cord herniating though a spinal canal defect. (p. 133) 30. What is the term for a malformation of the anterior neural tube that causes agenesis of the brain and calvarium? (p. 134) ___________________________________________________________ 31. What happens to amniotic fluid if a fetus has anencephaly? (p. 134) ________________________ 32. An infant has cyclopia. What other defects is this patient most likely to experience? (p. 134) _____ ______________________________________________________________________________ 33. An MRI scan of the cervical spine of an infant shows an enlargement of the central canal of the spinal cord. What is the term for this lesion? (p. 134) ____________________________________ 34. A patient presents with syringomyelia, thoracolumbar myelomeningocele, and hydrocephalus. What is the most likely diagnosis? (p. 134) ____________________________________________ 35. What is the Dandy-Walker malformation? Describe the clinical presentation. (p. 134) ______________________________________________________________________________ 36. A patient develops loss of pain and temperature sensation in the hands, but touch is intact. Where is the lesion most likely located? (p. 134) ______________________________________________ 37. Fill in the chart below to identify which arteries are derived from which aortic arch derivatives. (p. 135) Aortic Arch Arteries First aortic arch

Second aortic arch

Third aortic arch

file:///D|/USMLE%20Vids/Workbooks%202012/Chapter%203%20-%20Embryology%202012.txt[8/26/2013 2:13:09 AM]

Fourth aortic arch

Sixth aortic arch

38. Fill in the chart below to identify which structures are derived from which branchial arch. (p. 136) Branchial Arch Derivative Cartilage Muscles Nerves First branchial arch

Second branchial arch

Third branchial arch

file:///D|/USMLE%20Vids/Workbooks%202012/Chapter%203%20-%20Embryology%202012.txt[8/26/2013 2:13:09 AM]

Fourth branchial arch

Sixth branchial arch

39. Which cranial nerve mediates taste and sensation in the extreme posterior portion of the tongue? (p. 137) ________________________________________________________________________ 40. Which structure connects the thyroid gland with the tongue? (p. 138) ________________________ 41. How do the etiologies of cleft lip and cleft palate differ? (p. 138) ____________________________ ______________________________________________________________________________ 42. What is the term for the persistence of herniated abdominal contents into the umbilical cord? (p. 138) __________________________________________________________________________ 43. A newborn is noted to choke and vomit with routine feedings. The mother had polyhydramnios during pregnancy. What is the most likely diagnosis? (p. 139) _____________________________ 44. In congenital pyloric stenosis, hypertrophy of the pylorus leads to what problem? (p. 139) _______ ______________________________________________________________________________ 45. What is the treatment for congenital pyloric stenosis? (p. 139) _____________________________ 46. The head of the pancreas is derived from the _______________ (ventral/dorsal) pancreatic bud, the body is derived from the _______________ (ventral/dorsal) pancreatic bud, and the tail is derived from the _______________ (ventral/dorsal) pancreatic bud. (p. 139) 47. The main pancreatic duct is derived from the _______________ (ventral/dorsal) pancreatic bud, whereas the accessory pancreatic duct is derived from the _______________ (ventral/dorsal) pancreatic bud. (p. 139) 48. Which fetal structure functions as the interim kidney for the first trimester? (p. 140) _____________ 49. Which hormone inhibits development of the paramesonephric duct in males? (p. 141) __________ ______________________________________________________________________________ 50. In male development, the _______________ gene on chromosome _______________ codes for testis-determining factor. (p. 141) 51. In male development, the mesonephric duct develops into all genitourinary internal structures except the _______________. (p. 141) 52. Which cells produce the androgens that promote development of the mesonephric ducts? (p. 141) ______________________________________________________________________________ 53. What condition results from an abnormal opening of the penile urethra on the inferior (ventral) side of the penis as a result of failure of the urethral folds to close? (p. 143) ______________________ 54. What condition results from an abnormal opening of the penile urethra on the superior (dorsal) side of the penis as a result of faulty positioning of the genital tubercle? (p. 143) __________________
file:///D|/USMLE%20Vids/Workbooks%202012/Chapter%203%20-%20Embryology%202012.txt[8/26/2013 2:13:09 AM]

Answers

1. Weeks 3 through 8. 2. Week 4. 3. Day 6. 4. Dorsal; ventral. 5. Neurons derived from the alar plate are sensory, whereas neurons derived from the basal plate are motor. 6. Mesoderm; neural crest cells. 7. Cardiac, renal, and tracheoesophageal defects. (Remember: VACTERL = Vertebral defects, Anal atresia, Cardiac defects, TracheoEsophageal fistula, Renal defects, Limb defects) 8. Before week 3, there is usually an all-or-none effect: either embryonal death or no abnormalities. 9. A-16, B-12, C-2, D-6, E-19, F-1, G-17, H-14, I-7, J-9, K-4, L-18, M-15, N-8, O-10, P-11, Q-5, R-3, S-13. 10. Developmental retardation (both pre- and postnatal), microcephaly, holoprosencephaly, facial abnormalities, limb dislocation, and heart and lung fistulas. 11. One chorion and one amniotic sac. 12. The twins will be monochorionic and diamniotic. 13. Two arteries and one vein. 14. Deoxygenated. 15. Urachus. 16. Vitelline fistula. 17. Superior vena cava. 18. The ascending aorta and pulmonary trunk are created from the truncus arteriosus. 19. Foramen ovale. 20. Ductus venosus. 21. Ductus arteriosus. 22. 80%. 23. Indomethacin closes a PDA, while prostaglandins can keep it open. 24. Fourth ventricle. 25. Folic acid. 26. a-Fetoprotein. 27. Spina bifida occulta. 28. Meningocele. 29. Myelomeningocele. 30. Anencephaly. 31. There is polyhydramnios due to the lack of a swallowing center in the brain, so the fetus cannot swallow amniotic fluid. 32. Holoprosencephaly is associated with cyclopia, Pataus syndrome, severe fetal alcohol syndrome, and cleft lip/palate. 33. Syringomyelia. (Remember: syrinx is Greek for tube, as in syringe) 34. A Chiari type II malformation. 35. A posterior fossa malformation that is associated with a large posterior fossa and an absent cerebellum, with a cyst in its place. Presents in infancy with motor retardation and increasing enlargement of the skull. 36. Most commonly at C8-T1. (This describes the "cape-like" distribution of syringomyelia.) 37. Aortic Arch Arteries First aortic arch
file:///D|/USMLE%20Vids/Workbooks%202012/Chapter%203%20-%20Embryology%202012.txt[8/26/2013 2:13:09 AM]

Part of maxillary artery Second aortic arch Stapedial artery Hyoid artery Third aortic arch Common carotid artery Proximal part of ICA Fourth aortic arch Aortic arch (on left) Proximal part of RCA (on right) Sixth aortic arch Proximal part of pulmonary arteries Ductus arteriosus (on left only)

38. Branchial Arch Derivative Cartilage Muscles Nerves First branchial arch Meckels cartilage: mandible, malleus, and sphenomandibular ligament Muscles of mastication Mylohyoid Anterior belly of digastric Tensor tympani Tensor veli palatini CN V2 and V3 Second branchial arch Reicherts cartilage: stapes, styloid process, lesser horn of hyoid Muscles of facial
file:///D|/USMLE%20Vids/Workbooks%202012/Chapter%203%20-%20Embryology%202012.txt[8/26/2013 2:13:09 AM]

expression Stapedius Stylohyoid Posterior belly of digastric CN VII (facial expression) Third branchial arch Cartilage: greater horn of hyoid Stylopharyngeus CN IX Fourth branchial arch Cartilage: thyroid, cricoids, arytenoids, corniculate, cuneiform Most pharyngeal constrictors Cricothyroid Levator veli palatini CN X (superior laryngeal branch) Sixth branchial arch Cartilage: thyroid, cricoids, arytenoids, corniculate, cuneiform All intrinsic muscles of larynx except cricothyroid CN X (recurrent laryngeal branch)

39. CN X. 40. The thyroglossal duct (a common site of ectopic thyroid tissue). 41. Cleft lip occurs when the maxillary and medial nasal processes fail to fuse. Cleft palate occurs when the lateral palatine processes, the nasal septum, and/or the median palatine process fail to fuse. 42. Omphalocele. 43. Tracheoesophageal fistula. 44. Gastric outlet obstruction. (A classic sign is projectile vomiting.) 45. Surgical incision to relax the pyloric muscle (pyloromytomy). 46. Ventral; dorsal; dorsal. 47. Ventral; dorsal. 48. Mesonephros. 49. Mllerian inhibiting substance (secreted by Sertoli cells). (In females, it prevents development of the fallopian tubes, uterus, and upper third of the vagina.) 50. SRY; Y. 51. Prostate.
file:///D|/USMLE%20Vids/Workbooks%202012/Chapter%203%20-%20Embryology%202012.txt[8/26/2013 2:13:09 AM]

52. Leydig cells. 53. Hypospadias. (Patients have an increased risk of urinary tract infection). 54. Epispadias.

file:///D|/USMLE%20Vids/Workbooks%202012/Chapter%203%20-%20Embryology%202012.txt[8/26/2013 2:13:09 AM]

Microbiology

How to Use the Workbook with the Videos

Video Part Questions Microbiology Clinical Bacteriology part 1 17 Microbiology Clinical Bacteriology part 2 812 Microbiology Clinical Bacteriology part 3 1316 Microbiology Bacteriology part 1 1720 Microbiology Bacteriology part 2 2124 Microbiology Bacteriology part 3 2526 Microbiology Bacteriology part 4 2730 Microbiology Bacteriology part 5 3, 3133 Microbiology Mycology 3437 Microbiology Parasitology part 1 3841 Microbiology Parasitology part 2 4246 Microbiology Virology part 1 4748 Microbiology Virology part 2 4951 Microbiology Virology part 3 5255

file:///D|/USMLE%20Vids/Workbooks%202012/Chapter%204%20-%20Microbiology%202012.txt[8/26/2013 2:13:11 AM]

Microbiology Virology part 4 5660 Microbiology Systems part 1 6169 Microbiology Systems part 2 7071 Microbiology Antimicrobials part 1 7278 Microbiology Antimicrobials part 2 75, 7882 Microbiology Antimicrobials part 3 75, 8394

Questions CLINICAL BACTERIOLOGY 1. Which structure of the bacterial cell wall protects against phagocytosis? (p. 146) _______________ 2. Which feature of the cell wall is unique to gram-positive organisms? Which is unique to gram-negative organisms? (p. 146) _______________________________________________________ 3. Which bacterial species does not have a cell wall and therefore cannot be stained? How can it be identified? (pp. 147, 170) __________________________________________________________ 4. What feature of the cell wall makes it possible for an acid-fast stain to identify Mycobacterium and some Nocardia species? (p. 147) ____________________________________________________ 5. A 19-year-old Asian immigrant comes to the clinic because of blood in his sputum. He says that he has been losing weight and having night sweats. The patient has a fever, and physical examination reveals bronchial breath sounds with crepitant rales. Laboratory tests show lymphocytosis and an increased erythrocyte sedimentation rate. X-ray of the chest shows a calcified lung lesion and hilar lymphadenopathy. A sputum sample is obtained. Which stain should be used to identify the most likely infectious organism? (p. 148) A. Congo red B. Giemsa C. India ink D. Periodic acid-Schiff E. Ziehl-Neelson 6. What are the unique staining and culture requirements of fungi? (p. 148) _____________________ ______________________________________________________________________________ 7. True or False: Aminoglycosides are effective against obligate anaerobe infections. (p. 149) ______ 8. Group A streptococci produce _______________, which helps them avoid phagocytosis. (p. 150) 9. Exotoxins are heat-_______________ (stable/labile), have _______________ (high/low) toxicity, and _______________ (can/cannot) be vaccinated against. Endotoxins are heat-_______________ (stable/labile), have _______________ (high/low) toxicity, and _______________ (can/cannot) be vaccinated against. (p. 150) 10. A 4-year-old boy is brought to a rural clinic. He has a 2-day history of intense coughing spells that last 1-2 minutes and end with a loud gasp when he inhales. The patients mother mentions that he has had episodes of vomiting after the coughing spells. She also notes that he had a cold about 2 weeks earlier and that he has not received
file:///D|/USMLE%20Vids/Workbooks%202012/Chapter%204%20-%20Microbiology%202012.txt[8/26/2013 2:13:11 AM]

any immunizations. Laboratory tests show a WBC count of 22,000/mm3 with 60% lymphocytes. The rest of the workup (including x-ray of the chest) is unremarkable. Which organism is the most likely culprit? (p. 152) A. Bordetella pertussis B. Clostridium botulinum C. Corynebacterium diptheriae D. Staphylococcus aureus E. Vibrio cholera 11. Which four bacteria produce toxins that induce cAMP? (p. 152) ____________________________ ______________________________________________________________________________ 12. How do superantigens work? (p. 153) ________________________________________________ ______________________________________________________________________________ 13. Endotoxins are made of _______________ and are responsible for the release of the cytokines ______________ and _______________. (p. 153) 14. The transfer of DNA from one bacterium to another through viruses is known as _______________. (p. 154) 15. The ability of a bacterium to take up DNA from the environment is known as ___________. (p. 154) 16. Transferring genetic information from one bacterium to another in the form of plasmids is known as _______________ (p. 154) BACTERIOLOGY 17. Which organism is a gram-positive, catalase-positive, coagulase-positive coccus? (p. 156) ______ ______________________________________________________________________________ 18. How have MRSA developed their resistance to antibiotics? (p. 156) _________________________ ______________________________________________________________________________ 19. An 85-year-old woman presents from a nursing home with fever, chills, pleuritic pain, and rusty-greenish sputum. Physical examination reveals bronchial breath sounds over the right lower lobe with dullness to percussion, and increased tactile fremitus without tracheal deviation. The patients respiratory distress worsens until she is intubated and admitted to the intensive care unit, where she dies. Which organism is the most likely culprit? (p. 156) A. Chlamydia pneumonia B. Klebsiella pneumoniae C. Mycoplasma pneumonia D. Staphylococcus aureus E. Streptococcus pneumoniae 20. A 12-year-old boy is experiencing pain localized to the joints of the extremities. The mother recalls that he had a sore throat about a month earlier, but recovered completely without medical attention. Which of the following is the most likely etiology? (p. 157) A. Calcification of a thickened mitral valve B. Degeneration of synovial joints C. Hepatic failure D. Kidney failure

21. A 76-year-old man is hospitalized because of acute exacerbation of chronic obstructive pulmonary disease. After 5 days in the hospital, he develops a fever. Urine cultures show enterococci. An antibiotic is administered, and 10 days later the patient experiences watery stools. Cytotoxic assay of stool culture shows Clostridium difficile. Which antibiotic was most likely administered to treat the Enterococcus infection? (p. 158) A. Ampicillin B. Ciprofloxacin C. Clindamycin D. Metronidazole
file:///D|/USMLE%20Vids/Workbooks%202012/Chapter%204%20-%20Microbiology%202012.txt[8/26/2013 2:13:11 AM]

E. Trimethoprim-sulfamethoxazole 22. Actinomyces israelii is a gram-positive _______________ (aerobe/anaerobe) that causes _______________ infections, while Nocardia asteroides is a gram-positive _______________ (aerobe/anaerobe) that causes __________ infections. Both have what kind of structure? (p. 159). 23. How are the results of a PPD for tuberculosis interpreted? (p. 159) _________________________ ______________________________________________________________________________ 24. What are the two forms of Hansens disease? (p. 160) ___________________________________ 25. Public-health investigators looking into several cases of pneumonia that have occurred in a community are able to trace the outbreak to a water-mist machine used in the produce section of a supermarket. Patients exposed to the water-mist machine have presented with cough, high fever, headache, and abdominal pain. Elderly patients present with more severe disease. Which organism is most likely responsible for this outbreak? (p. 162) A. Haemophilus influenzae type B B. Legionella pneumophila C. Mycobacterium tuberculosis D. Streptococcus pneumoniae 26. Five soldiers living in the same military barrack present to the infirmary with high fever, headache, stiff neck, and a rash on the trunk. CSF analysis shows increased polymorphonuclear leukocytes, increased protein, and decreased glucose in all five patients. A bacterial culture of CSF is most likely to show which of the following? (p. 162) A. Gram-negative bacilli B. Gram-negative cocci C. Gram-negative coccobacilli 27. What are three virulence factors for Escherichia coli and with what pathologies are they associated? (p. 163) _____________________________________________________________ 28. A 28-year-old man presents to his physician because of worsening muscle weakness that began in his legs and feet 3 days earlier, but now involves his arms and hands. Other than having a self-limited episode of gastroenteritis 2 weeks earlier, the patient has been in good health. CSF analysis shows a highly elevated protein level, a normal cell count, and a normal glucose level. An infection with which organism is associated with this patients neurologic symptoms? (p. 164) A. Candida albicans B. Campylobacter jejuni C. Pseudomonas aeruginosa D. Streptococcus pneumoniae E. Streptococcus pyogenes 29. Which is more virulent, Salmonella or Shigella? (p. 164) __________________________________ 30. Describe the three stages of Lyme disease. (p. 165) ____________________________________ ______________________________________________________________________________ 31. Describe the three stages of syphilis. (p. 166) _________________________________________ ______________________________________________________________________________ 32. Which zoonotic bacteria are transmitted by tick bites? (p. 167) ____________________________ 33. How do the rashes of Rocky Mountain spotted fever and typhus differ? (p. 168) ______________ ______________________________________________________________________________ MYCOLOGY 34. Match the four fungi that cause systemic disease resembling tuberculosis with the area(s) to which they are endemic. (p. 171) _____ A. Blastomyces 1. East of Mississippi River and in Central America _____ B. Coccidioides 2. Mississippi and Ohio River Valleys _____ C. Histoplasma 3. Rural Latin America _____ D. Paracoccidioides 4. Southwestern U.S., California 35. An African-American teenager presents to the clinic in June with patches of skin on her face that are white and hypopigmented. Laboratory tests show a spaghetti and meatball appearance on KOH. What is the most likely diagnosis? (p. 172) _______________________________________
file:///D|/USMLE%20Vids/Workbooks%202012/Chapter%204%20-%20Microbiology%202012.txt[8/26/2013 2:13:11 AM]

36. What is the diagnostic form of Candida albicans? (p. 173) ________________________________ 37. A 35-year-old man who is an HIV-positive, long-term abuser of intravenous drugs presents to the emergency department with a cough and fever. X-ray of the chest shows faint bilateral interstitial infiltrates. Histologic analysis of induced sputum is likely to show trophozoite forms of which fungus? (p. 174) _________________________________________________________________ PARASITOLOGY 38. Which protozoa are most likely to cause gastrointestinal infections? Which are most likely to cause visceral infections? (pp. 175-177) ___________________________________________________ ______________________________________________________________________________ 39. Specify the common treatment(s) for each protozoan disease. (One number is used twice.) (pp. 175-177) _____ A. Giardia lamblia 1. Amphotericin _____ B. Leishmania donovani 2. Chloroquine _____ C. Naegleria fowleri 3. Metronidazole _____ D. Plasmodium 4. Nifurtimox _____ E. Toxoplasma 5. Sodium stibogluconate _____ F. Trichomonas vaginalis 6. Sulfadiazine and pyrimethamine _____ G. Trypanosoma cruzi 40. Which two plasmodium species should be treated with primaquine? Why? (p. 176) ______________________________________________________________________________ 41. A patient presents with trouble breathing, chest pain, dysphagia, and weight loss. Which protozoan species is the most likely cause? (p. 177) _____________________________________________ 42. A 66-year-old woman who recently emigrated from Mexico comes to the clinic because she has begun to have seizures. MRI of her head is shown in the image. Which organ or tissue is most likely to have similar lesions? (p. 179)
Image courtesy of Carlton Evans, Hector H. Garcia, Robert H. Gilman, and Jon S. Friedland A. Bladder B. Bone C. Kidney D. Skeletal muscle E. Small bowel 43. What drugs are used to treat intestinal nematode disease? Which drug is used to treat disease caused by trematodes? (pp. 178-179) ________________________________________________ 44. Which three worms penetrate into the human host cutaneously? (p. 178) ____________________ ______________________________________________________________________________ 45. Specify the common treatment(s) for each helminthic disease. (Numbers may be used more than once.) (pp. 178179) _____ A. Ascaris lumbricoides 1. -bendazoles _____ B. Diphyllobothrium latum 2. -bendazoles and pyrantel pamoate _____ C. Enterobius 3. Ivermectin _____ D. Onchocerca volvulus 4. Praziquantel _____ E. Schistosoma _____ F. Taenia solium _____ G. Trichinella spiralis 46. Which helminth is associated with squamous cell bladder cancer? (p. 179) ___________________ VIROLOGY 47. A child who suffers from an immunosuppressive disease is brought to his pediatrician for vaccinations. This patient should be given which vaccine? (p. 181) A. Measles B. Mumps C. Rubella D. Salk E. Varicella 48. Most DNA viruses replicate in the _______________ (cytoplasm/nucleus), while most RNA viruses replicate in
file:///D|/USMLE%20Vids/Workbooks%202012/Chapter%204%20-%20Microbiology%202012.txt[8/26/2013 2:13:11 AM]

the _______________ (cytoplasm/nucleus). (p. 181) 49. A 4-year-old child is brought to his physicians office because of a parvovirus infection. Which sign or symptom is most likely to be seen? (p. 183) A. Barking cough B. Erythema of cheeks C. Parotid gland swelling D. Upper respiratory infection E. Vesicular rash that appears in crops 50. A homeless 37-year-old woman with HIV infection comes to the clinic with a 4-week history of worsening hemiparesis, visual field deficits, and cognitive impairment. She has gone 2 years without antiretroviral therapy and her CD4+ count is 22/mm. MRI shows several hyperintensities on T2-weighted images that do not enhance with contrast and are not surrounded by edema. CSF analysis shows a normal opening pressure, a mildly elevated protein level, and the presence of myelin basic protein, with a mild mononuclear pleocytosis. What is the most likely etiology? (p. 183) A. Cortical tuberculoma B. Cytomegalovirus encephalitis C. JC virus D. Primary CNS lymphoma E. Toxoplasmosis 51. Which herpesviruses are transmitted by respiratory secretions? Which are transmitted via sexual contact? (p. 184) ________________________________________________________________ 52. Which RNA virus is the leading cause of fatal diarrhea in children? (p. 186) ___________________ 53. A 6-year-old boy is brought to the emergency department by his parents after they all returned from a trip to East Africa. The parents report that approximately 2 weeks earlier, the patient had a fever and diarrhea that resolved. However, he now has a fever and weakness of his left leg. On further questioning, his parents state that he is homeschooled and has never received vaccinations. Which sequel is most likely to occur in this patient? (p. 187) A. Neuron loss in posterior horns B. Respiratory muscle paralysis C. Sensory loss in affected limbs D. Short-term memory loss 54. A family who recently emigrated from Romania brings their 7-year-old son to the clinic because of conjunctivitis and periorbital swelling. The child has had coughing, a runny nose, and high fever for 3 days. Physical examination reveals small lesions with blue-white centers in his oral cavity. What is the most likely cause of his symptoms? (p. 189) A. Diptheria B. Pertussis C. Roseola D. Rubella E. Rubeola 55. A 17-year-old boy comes to the office complaining of fever and painful, swollen cheeks. He says one of his relatives had a similar illness several weeks earlier. He finds it difficult to talk, eat, and swallow. Physical examination is most likely to reveal which of the following? (p. 189) A. Hepatomegaly B. Orchitis C. Papilledema D. Peripheral edema E. Splenomegaly 56. What three signs/symptoms are common to all hepatitis virus infections? (p. 190) ______________ ______________________________________________________________________________ 57. Which serologic markers would most likely be seen in a patient with chronic HBV of low infectivity? (p. 190) ________________________________________________________________________ 58. How is HIV diagnosed? (p. 192) _____________________________________________________
file:///D|/USMLE%20Vids/Workbooks%202012/Chapter%204%20-%20Microbiology%202012.txt[8/26/2013 2:13:11 AM]

59. A 51-year-old man presents to the clinic with a 4-month history of increasing cognitive decline characterized by increasing apathy and mental slowing. Physical examination reveals impaired saccadic eye movements, impaired ability to perform rapidly alternating movements, diffuse hyperreflexia, and frontal release signs. CSF analysis shows a total protein level of 72 mg/dL and an elevated IgG level. MRI of the brain shows global cerebral atrophy with multiple ill-defined areas of white matter enhancement. What is the most likely etiology? (p. 194) A. CMV encephalitis B. CNS lymphoma C. Disseminated Mycobacterium avium-intracellulare infection D. HIV-associated dementia E. Toxoplasmosis 60. In a patient with HIV infection, at what CD4 count does prophylaxis against Pneumocystis jiroveci become prudent? At what CD4 count does prophylaxis against CMV retinitis become prudent? (p.194) ________________________________________________________________________ SYSTEMS 61. Why does food poisoning due to Streptococcus aureus have such a quick onset? (p. 195) _______ ______________________________________________________________________________ 62. In the chart below, checkmark the type of diarrhea caused by each infectious agent. (p. 196) Bacterium Bloody diarrhea Watery diarrhea Campylobacter

Clostridium difficile

Clostridium perfringens

Entamoeba histolytica

Enterohemorrhagic E. coli

Enteroinvasive E. coli

Enterotoxigenic E. coli

Salmonella

file:///D|/USMLE%20Vids/Workbooks%202012/Chapter%204%20-%20Microbiology%202012.txt[8/26/2013 2:13:11 AM]

Shigella

Protozoa

Vibrio cholera

Viruses

Yersinia enterocolitica

63. A 70-year-old man living in a nursing home presents with neck stiffness, fever, headache, and photophobia. What is the most likely diagnosis, and what is the most likely causative organism? If the patient were 55 years old, would you suspect a different organism? Why or why not? (p. 196) ______________________________________________________________________________ 64. A 3-month-old girl is brought to the hospital with concerns from the mother that she is irritable, will not feed or stop crying, and has a fever. After ruling out pneumonia and urinary tract infection, meningitis is suspected as the source of the fever. Which three organisms are highest on the differential? (p. 197) ______________________________________________________________ 65. Fill n the chart below with the typical CSF findings in meningitis. (p. 197) Type of Infection Opening Pressure Predominant Cell Type Protein Level Sugar Level Bacterial

Fungal/TB

Viral
file:///D|/USMLE%20Vids/Workbooks%202012/Chapter%204%20-%20Microbiology%202012.txt[8/26/2013 2:13:11 AM]

66. A 10-year-old boy presents with fever, bone pain, and tenderness of the leg. There is localized redness and swelling of the site. What is the most likely diagnosis, and what is the most likely causative organism? (p. 197) _______________________________________________________ 67. A woman presents to her physician because of pain during urination. She also says that she has to urinate more often than usual and sometimes has trouble holding it in. What is the most likely diagnosis? What test would confirm the diagnosis? (pp. 197, 198, 520) ______________________ ______________________________________________________________________________ 68. A woman presents to her physician because of pain during urination. She also mentions that she has been experiencing fever and chills. She complains that her back has been hurting, and when the physician presses where the patient is pointing, she writhes in pain. What is the most likely diagnosis? What test would confirm the diagnosis? (pp. 197, 198, 520) ______________________ ______________________________________________________________________________ 69. List the ToRCHeS infections. (p. 199) ________________________________________________ ______________________________________________________________________________ 70. A child presents to the office with a rash. His mother says that the rash began on his head and has slowly moved downward, and is now located on his trunk. What two diagnoses should be at the top of the differential? How can these two infections be distinguished? (p. 200) ___________________ ______________________________________________________________________________ 71. What are the two most common causes of nosocomial infection? (p. 202) ____________________ ANTIMICROBIALS 72. Match each antimicrobial with its mechanism of action. (Numbers may be used more than once). (p. 204) _____ A. Aminoglycosides 1. Block bacterial cell wall synthesis _____ B. Ampicillin 2. Block DNA topoisomerases _____ C. Aztreonam 3. Block mRNA synthesis _____ D. Bacitracin 4. Block nucleotide synthesis _____ E. Cephalosporins 5. Block peptidoglycan synthesis _____ F. Chloramphenicol 6. Block protein synthesis at 30S subunit _____ G. Clindamycin 7. Block protein synthesis at 50S subunit _____ H. Fluoroquinolones _____ I. Imipenem _____ J. Linezolid _____ K. Macrolides _____ L. Penicillin _____ M. Rifampin _____ N. Streptogramins _____ O. Sulfonamides _____ P. Tetracyclines _____ Q. Trimethoprim _____ R. Vancomycin 73. What are the two main clinical uses of penicillin? (p. 204) _________________________________ ______________________________________________________________________________ 74. Which antimicrobials are effective against Pseudomonas? (p. 205) _________________________ 75. Match the antimicrobial agent (or class) and its associated adverse effects. (p. 205-214) _____ A. Acute cholestatic hepatitis 1. Azoles
file:///D|/USMLE%20Vids/Workbooks%202012/Chapter%204%20-%20Microbiology%202012.txt[8/26/2013 2:13:11 AM]

_____ B. Discoloration of teeth 2. Chloramphenicol _____ C. Disulfiram-like reaction 3. Ethambutol _____ D. Gray baby syndrome 4. Isoniazid _____ E. Gynecomastia 5. Macrolides _____ F. Interstitial nephritis 6. Methicillin _____ G. Orange body fluids 7. Metronidazole _____ H. Lupus 8. Rifampin _____ I. Red-green color blindness 9. Tetracyclines 76. Match the generation of cephalosporin with its antimicrobial profile. (p. 206) _____ A. First generation (cefazolin, 1. Gram-negative infections cephalexin) 2. Gram-positive cocci, H. influenzae, _____ B. Second generation (cefoxitin, Enterobacter, Neisseria, Proteus, Klebsiella, cefaclor, cefuroxime) Serratia _____ C. Third generation (ceftriaxone, 3. Gram-positive cocci, Proteus, E. coli, cefotaxime, ceftazidime) Klebsiella _____ D. Fourth generation (cefepime) 4. Pseudomonas, gram-positive organisms 77. Which antimicrobial is associated with nephrotoxicity, ototoxicity, and red man syndrome? How is this syndrome prevented? (p. 207) __________________________________________________ 78. In the chart below, checkmark whether the agents are bacteriostatic or bactericidal. (pp. 207-211) Antibiotic Bactericidal Bacteriostatic Aminoglycosides

Cephalosporins

Chloramphenicol

Clindamycin

Erythromycin

Fluoroquinolones

Metronidazole

Penicillin

file:///D|/USMLE%20Vids/Workbooks%202012/Chapter%204%20-%20Microbiology%202012.txt[8/26/2013 2:13:11 AM]

Sulfamethoxazole

Tetracyclines

Trimethoprim

Vancomycin

79. Which tetracycline also has utility in treating SIADH? Why? (p. 209) ________________________ 80. Which antimicrobials are used to treat atypical pneumonias? (p. 209) _______________________ 81. Which antimicrobial forms free-radical toxic metabolites in bacterial cells that damage DNA? (p. 211) ________________________________________________________________________ 82. Fill in the chart below to identify which drugs are used as prophylaxis and as treatment for mycobacterial infections. (p. 212) Bacterium Prophylaxis Treatment M. avium-intracellulare

M. leprae

M. tuberculosis

83. Which antifungal agents form membrane pores that allow leakage of electrolytes? To what, specifically, do these antifungals bind? (p. 214) _________________________________________ 84. What is the topical form of amphotericin? (p. 214) _______________________________________ 85. What is the mechanism of action of azoles? (p. 214) _____________________________________ 86. Fill in the diagram below, noting the antiviral agent(s) that interfere with the various steps. (p. 216) (Adapted, with permission, from Katzung BG, Trevor AJ. USMLE Road Map: Pharmacology. New York: McGraw-Hill, 2003: 120.) 87. Which antiviral agent is also used to treat Parkinsons disease? Why? (p. 216) _______________ ______________________________________________________________________________ 88. What is the mechanism of action of zanamivir and oseltamivir? (p. 216) _____________________
file:///D|/USMLE%20Vids/Workbooks%202012/Chapter%204%20-%20Microbiology%202012.txt[8/26/2013 2:13:11 AM]

______________________________________________________________________________ 89. Specify the common treatment(s) for each viral infection. (pp. 216-217) _____ A. Chronic hepatitis C 1. Acyclovir _____ B. Cytomegalovirus 2. Foscarnet _____ C. CMV retinitis 3. Ganciclovir _____ D. Herpes simplex virus 4. Ribavirin _____ E. Influenza B 5. Zanamivir 90. What are the three indications for HAART? (p. 218) _____________________________________ 91. Why are multiple drugs used in HAART? (p. 218) _______________________________________ 92. List the five most commonly used protease inhibitors. (p. 218) ____________________________ ______________________________________________________________________________ 93. List the four most commonly used nucleoside reverse transcriptase inhibitors. (p. 218) _________ ______________________________________________________________________________ 94. Match the agent with the reason why it should be avoided in pregnant women. (One number is used twice.) (p. 219) _____ A. Aminoglycosides 1. Embryotoxic _____ B. Chloramphenicol 2. Cartilage damage _____ C. Clarithromycin 3. Discoloration of teeth _____ D. Fluoroquinolones 4. Gray baby syndrome _____ E. Griseofulvin 5. Kernicterus _____ F. Metronidazole 6. Mutagenesis _____ G. Ribavirin 7. Ototoxicity _____ H. Sulfonamides 8. Teratogenic _____ I. Tetracyclines

Answers CLINICAL BACTERIOLOGY 1. The capsule. 2. LipoTeichoic acid is found in the cell wall of gram-positive organisms, while endotoxin/LPS is found in the cell wall of gram-negative organisms. 3. Mycoplasma; it can be grown on Eatons agar. 4. Mycolic acid. 5. E. The Ziehl-Neelsen stain is used to detect acid-fast bacteria. This patient most likely has tuberculosis, which is an acid-fast, gram-positive aerobic bacillus. 6. Fungi require a silver stain and are cultured on Sabourads agar. 7. False; aminoglycosides require oxygen to enter the bacterial cell wall. 8. M protein. 9. Exotoxins are heat-labile, have high toxicity, and can be vaccinated against. In contrast, endotoxins are heat-stable, have low toxicity, and no vaccines are available. 10. A. This is a typical picture of pertussis, or whooping cough. The patient has not been immunized and has characteristic coughing paroxysms interspersed with a loud "whooping" sound caused by inspiration against a narrowed airway. This paroxysmal phase is preceded by an initial phase that is indistinguishable from common upper respiratory infections. Post-tussive emesis (vomiting after a coughing spell) also is common with pertussis. A prominent lymphocytosis is often present as well. This is presumably caused by the ability of pertussis toxin to inhibit
file:///D|/USMLE%20Vids/Workbooks%202012/Chapter%204%20-%20Microbiology%202012.txt[8/26/2013 2:13:11 AM]

chemokine receptors. A culture of Bordetella pertussis takes 7-10 days; therefore it is important to make a presumptive diagnosis based on the clinical picture. 11. Vibrio cholerae, Bordetella pertussis, Escherichia coli (heat-labile toxin), and Bacillus anthracis. 12. Superantigens work by binding directly and simultaneously to MHC II and T-cell receptors; this activates T cells to stimulate the release of INF-. and IL-2. 13. Lipopolysaccharides; IL-1 and TNF. 14. Transduction. 15. Transformation. 16. Conjugation. BACTERIOLOGY 17. Staphylococcus aureus. 18. MRSA have developed an altered penicillin-binding protein that makes them resistant to -lactam antibiotics. 19. E. Streptococcus pneumoniae is the most common cause of lobar pneumonia. 20. A. This patient had Streptococcus pyogenes pharyngitis, which caused rheumatic fever. 21. A. Clostridium difficile produces an endotoxin that is cytotoxic to the colonic mucosa, which leads to pseudomembrane formation. It also releases a toxin that causes watery diarrhea. Classically, ampicillin and clindamycin lead to C. difficile overgrowth and pseudomembranous colitis, but almost any antibiotic can cause this disease. This particular patient had an enterococcal infection, which is treated with ampicillin. 22. A. israelii is an anaerobe that causes oral/facial abscesses; N. asteroids is an aerobe that causes pulmonary infection in immunocompromised patients. Both have a branching filament structure. 23. A positive PPD indicates current infection, past exposure, or vaccination against BCG; a negative PPD indicates no infection or that the patient is anergic. 24. Lepromatous and tuberculoid. 25. B. Legionella pneumophila is a gram-negative rod that causes Legionnaires' disease, a condition in which pneumonia and high fever occur. Other signs include hyponatremia and CNS changes. The organism is present only in water sources (e.g., air conditioning systems, whirlpools, mist sprayers) and can cause infection when aerosolized water droplets are inhaled. The organism is not transmitted by person-to-person contact. 26. B. These patients likely have bacterial meningitis. Bacterial meningitis is characterized by increased polymorphonuclear leukocytes, increased protein, and decreased glucose in the CSF. The most likely cause in this setting (young adults) is Neisseria meningitidis, which is a gram-negative coccus that ferments maltose and glucose. (N. gonorrhoeae ferments only glucose.) 27. Fimbriae-cystitis and pyelonephritis, K capsule-pneumonia, neonatal meningitis, and LPS endotoxin-septic shock. 28. B. This patient has Guillain-Barr syndrome, an acute peripheral neuropathy that causes progressive weakness over several days. Approximately two thirds of these patients have an antecedent gastrointestinal or flulike illness. The most common involves Campylobacter jejuni infections. 29. Shigella. 30. Stage 1 is characterized by ECM (bulls eye rash) and flulike symptoms. Stage 2 is characterized by neurologic and cardiac symptoms (including Bells palsy). Stage 3 is characterized by chronic monoarthritis and migratory polyarthritis. 31. Primary syphilis is localized and presents with a painless chancre. Secondary syphilis is disseminated and causes constitutional symptoms, particularly condylomata lata. Tertiary syphilis is associated with tabes dorsalis and aortic involvement. 32. Lyme disease (Borrelia burgdorferi) and tularemia (Francisella tularensis). 33. The rash of RMSF starts on the palms and soles, then migrates to trunk; the rash of typhus starts on the trunk and spreads outward but spares the palms and soles. MYCOLOGY 34. A-1, B-4, C-2, D-3. 35. Tinea versicolor. 36. Germ tube formation. 37. Pneumocystis jiroveci pneumonia. PARASITOLOGY 38. Giardia lamblia, Entamoeba histolytica, and Cryptosporidium are likely to cause GI infections, while Trypanosoma cruzi and Leishmania donovani are likely to cause visceral infections.
file:///D|/USMLE%20Vids/Workbooks%202012/Chapter%204%20-%20Microbiology%202012.txt[8/26/2013 2:13:11 AM]

39. A-3, B-5, C-1, D-2, E-6, F-3, G-4. 40. P. ovale and P. vivax. Because they have the ability to lie dormant in the liver. 41. Trypanosoma cruzi, which causes Chagas disease, megacolon, and dilated cardiomyopathy. 42. D. Although this patient's presentation is highly suspicious for malignancy, the image confirms neurocysticercosis, which is caused by infection with Taenia solium, a pork tapeworm. It is the most common parasitic infection of the CNS worldwide, and is particularly endemic to Central and South America, Eastern Europe, and some parts of Asia. After humans ingest the tapeworm's eggs, the eggs hatch and the larvae invade the wall of the small intestine and disseminate hematogenously. Cysticerci may be found in any organ, but most commonly appear in the brain, muscles, skin, and heart. Because this patient is already suffering from cysts in her brain, the most likely additional location would be her muscles. 43. Bendazoles are typically used to kill intestinal nematodes, while praziquantel is typically used to kill flukes. 44. Strongyloides, Ancylostoma, and Necator. 45. A-2, B-4, C-2, D-3, E-4, F-4, G-1. 46. Schistosoma haematobium.

VIROLOGY 47. D. The Salk polio vaccine is a killed vaccine, meaning that it induces humoral immunity only and it does not have the ability to replicate. Live attenuated vaccines induce both humoral and cell-mediated immunity but are more dangerous for immunosuppressed patients because they can cause illness. 48. Nucleus; cytoplasm. 49. B. Parvovirus B19 is associated with red cheeks or a "slapped cheek" appearance. 50. C. The clinical picture and imaging are consistent with progressive multifocal leukoencephalopathy (PML) secondary to reactivation of latent JC virus infection, which can occur with CD4 counts <50/mm. It typically presents with rapidly progressive focal neurologic deficits without signs of increased intracranial pressure. Ataxia, aphasia, and cranial nerve deficits also may occur. Lumbar puncture is nondiagnostic and frequently shows mild elevations in protein and WBCs. CSF analysis can show myelin basic protein, which is due to demyelination caused by the JC virus. PML is characterized by multiple nonenhancing T2-hyperintense lesions on MR. When PML is suspected, a stereotactic biopsy is required for definitive diagnosis; however, a positive CSF polymerase chain reaction for JC virus is diagnostic in the appropriate clinical setting. 51. HSV-1, VZV, and EBV are transmitted via respiratory secretions; HSV-2, CMV, and HHV-8 are transmitted via sexual contact. 52. Rotavirus. 53. B. This child has classic symptoms of polio. Poliovirus infects Peyer's patches of the intestine and the motor neurons. It is passed by the fecal-oral route and can present as a spectrum of severity, depending on the patients age. Younger children and infants often have a nonclinical infection or mild fever with diarrhea. In older children who have not previously been infected, meningitic signs can develop. The most severe complications are respiratory muscle failure, paraplegia, and quadriplegia. 54. E. Rubeola (measles) is relatively uncommon in the United States due to the MMR vaccine. The rash that spreads from head to toe develops 1-2 days after the appearance of Koplik spots, which are red oral lesions with blue-white centers. 55. B. Mumps is an infectious disease that can cause swollen cheeks. While not often seen in the United States because of the MMR vaccine, mumps occasionally presents in those who have not been vaccinated. Mumps is caused by an RNA paramyxovirus that replicates in the upper respiratory tract and causes parotiditis and, frequently, orchitis. Pancreatitis and meningitis can also be present. 56. Episodic fevers, jaundice, and elevated ALT and AST levels. 57. HbsAg, anti-HbeAg antibody, and anti-HbcAg antibody. 58. ELISA first, then a positive ELISA is confirmed with a Western blot. 59. D. HIV-associated dementia (also known as AIDS dementia) presents with memory loss, gait disorder, and spasticity. It generally occurs later in the course of illness. Early symptoms may be subtle and include depressive symptoms and apathetic withdrawal; later symptoms include global dementia and motor deficits. As the dementia progresses, patients experience difficulty with smooth limb movement, dysdiadochokinesia (impairment in performing rapid, alternating movements), impaired saccadic eye movements, hyperreflexia, and frontal release signs. Imaging
file:///D|/USMLE%20Vids/Workbooks%202012/Chapter%204%20-%20Microbiology%202012.txt[8/26/2013 2:13:11 AM]

studies are imperative to rule out mass lesions; 20%-40% of patients demonstrate nonenhancing, poorly demarcated areas of increased T2 signal intensity in the deep white matter. The symptoms must be distinguished from typical focal neurologic signs and symptoms that may be evident in patients with mass lesions. Elevated levels of protein and IgG on CSF analysis are present in approximately 45% and 80% of cases, respectively. 60. A CD4 count <200 mm3; a CD4 count <50 mm3. SYSTEMS 61. Because the toxin is preformed when ingested. 62. Bacterium Bloody diarrhea Watery diarrhea Campylobacter v Clostridium difficile v v Clostridium perfringens v Entamoeba histolytica v Enterohemorrhagic E. coli v Enteroinvasive E. coli v Enterotoxigenic E. coli v Salmonella v Shigella v Protozoa v Vibrio cholera

file:///D|/USMLE%20Vids/Workbooks%202012/Chapter%204%20-%20Microbiology%202012.txt[8/26/2013 2:13:11 AM]

v Viruses v Yersinia enterocolitica v

63. Streptococcus pneumoniae. Yes, Neisseria meningitidis. Remember that this is the most likely cause of meningitis for any patient who is 6-60 years old. 64. Group B Strepococcus, Eschericia coli, and Listeria. 65. Type of Infection Opening Pressure Predominant Cell Type Protein Level Sugar Level Bacterial . PMNs . . Fungal/TB . Lymphocytes . . Viral Normal to . Lymphocytes Normal to . Normal

66. Osteomyelitis; Staphylococcus aureus. 67. This patient has a UTI. Urinalysis will show a high WBC count in the urine, positive leukocyte esterase, and nitrites. The most likely organism is E. coli. 68. This patient has pyelonephritis. You should know the symptoms that differentiate a lower UTI from an infection that has ascended into the kidneys. On urinalysis, pyelonephritis is characterized by WBC casts in addition to WBCs, nitrites, and leukocyte esterase. The most likely cause is E. coli. 69. The ToRCHes infections are Toxoplasma gondii, Rubella, Cytomegalovirus, HIV/Herpes simplex virus, and Syphilis. 70. Measles (rubeola) and rubella. Measles: look for cough, coryza, conjunctivitis, and Koplik spots. Rubella: look for
file:///D|/USMLE%20Vids/Workbooks%202012/Chapter%204%20-%20Microbiology%202012.txt[8/26/2013 2:13:11 AM]

postauricular lymphadenopathy. 71. E. coli (UTI) and S. aureus (wound infection). ANTIMICROBIALS 72. A-6, B-1, C-1, D-5, E-1, F-7, G-7, H-2, I-1, J-7, K-7, L-1, M-3, N-7, O-4, P-6, Q-4, R-5. 73. Gram-positive organisms, syphilis. 74. Ticarcillin, carbenicillin, piperacillin, and fourth-generation cephalosporins. 75. A-5, B-9, C-7, D-2, E-1, F-6, G-8, H-4, I-3. 76. A-3, B-2, C-1, D-4. 77. Vancomycin; by the slow infusion of antihistamine.

78. Antibiotic Bactericidal Bacteriostatic Aminoglycosides v Cephalosporins v Chloramphenicol v Clindamycin v Erythromycin v Fluoroquinolones v

file:///D|/USMLE%20Vids/Workbooks%202012/Chapter%204%20-%20Microbiology%202012.txt[8/26/2013 2:13:11 AM]

Metronidazole v Penicillin v Sulfamethoxazole v Tetracyclines v Trimethoprim v Vancomycin v

79. Demecyocycline; it acts as a diuretic in SIADH. 80. Macrolides. 81. Metronidazole. 82. Bacterium Prophylaxis Treatment M. avium-intracellulare Azithromycin Azithromycin M. leprae None Dapsone Rifampin Clofazimine M. tuberculosis Isoniazid Rifampin Isoniazid Pyrazinamide Ethambutol

83. Amphotericin binds to ergosterol.


file:///D|/USMLE%20Vids/Workbooks%202012/Chapter%204%20-%20Microbiology%202012.txt[8/26/2013 2:13:11 AM]

84. Nystatin. 85. Inhibit ergosterol synthesis. 86. (Adapted, with permission, from Katzung BG, Trevor AJ. USMLE Road Map: Pharmacology. New York: McGraw-Hill, 2003: 120.) 87. Amantadine; increases dopamine release. 88. Neuraminidase inhibition. 89. CD4 counts <350 mm3, AIDS-defining illness, and a high viral load. 90. To prevent resistance. 91. A-4, B-3, C-2, D-1, E-5. 92. Saquinavir, ritonavir, indinavir, nelfinavir, and amprenavir. 93. Zidovudine, didanosine, zalcitabine, and stavudine. 94. A-7, B-4, C-1, D-2, E-8, F-6, G-8, H-5, I-3.

file:///D|/USMLE%20Vids/Workbooks%202012/Chapter%204%20-%20Microbiology%202012.txt[8/26/2013 2:13:11 AM]

Immunology

How to Use the Workbook with the Videos

Video Part Questions Immunology Lymphoid Structures 110 Immunology Lymphocytes 1122 Immunology Immune Responses part 1 2334 Immunology Immune Responses part 2 3540 Immunology Immunosuppressants 4143

file:///D|/USMLE%20Vids/Workbooks%202012/Chapter%205%20-%20Immunology%202012.txt[8/26/2013 2:13:13 AM]

Questions LYMPHOID STRUCTURES 1. Which lymph node area contains cords of closely packed lymphocytes, plasma cells, and sinuses? (p. 222) ________________________________________________________________________ 2. Which lymph node structures communicate with efferent lymphatics and contain reticular cells and macrophages? (p. 222) ________________________________________________________ 3. Which lymph node area contains endothelial venules through which T and B cells enter the nodes from the blood? (p. 222) ___________________________________________________________ 4. Which part of the lymph node contains B cells? (p. 222) __________________________________ 5. In which part of the lymph node are follicles located? (p. 222) _____________________________ 6. Which lymph node area becomes greatly enlarged during an extreme cellular immune response? (p. 222) ________________________________________________________________________ 7. Match the area of the body with its primary lymph node drainage site. (Numbers may be used more than once). (p. 222) _____ A. Anal canal (below pectinate line) 1. Axillary _____ B. Duodenum 2. Celiac _____ C. Jejenum 3. Colic to inferior mesenteric _____ D. Lateral breast 4. Internal iliac _____ E. Lateral side of dorsum of foot 5. Popliteal _____ F. Rectum (above pectinate line) 6. Superficial and deep plexuses to para-aortic _____ G. Scrotum 7. Superficial inguinal _____ H. Sigmoid colon 8. Superficial mesenteric _____ I. Stomach _____ J. Superficial thigh _____ K. Testes _____ L. Upper limbs 8. What three findings are typically seen on peripheral blood smears of postsplenectomy patients? (p. 223) __________________________________________________________________________ 9. Postsplenectomy patients are most susceptible to which four pathogens? (p. 223) ____________ ______________________________________________________________________________ 10. B cells mature in the _______________ (bone marrow/thymus), while T cells mature in the _______________ (bone marrow/thymus). (p. 223) LYMPHOCYTES 11. In the chart below, checkmark which components are part of the adaptive vs. the innate immune system. (p. 223) Component Adaptive Immune System Innate Immune System Antibody

B cells
file:///D|/USMLE%20Vids/Workbooks%202012/Chapter%205%20-%20Immunology%202012.txt[8/26/2013 2:13:13 AM]

Complement

Dendritic cells

Macrophages

Natural killer cells

Neutrophils

T cells

12. What are the three MHC class I genetic loci? (p. 224) ____________________________________ 13. What are the three MHC class II genetic loci? (p. 224) ___________________________________ 14. Match the HLA subtype with its associated disease. (Numbers may be used more than once.) (p. 224) _____ A. Ankylosing spondylitis 1. A3 _____ B. Diabetes mellitus type 1 2. B8 _____ C. Goodpastures syndrome 3. B27 _____ D. Graves disease 4. DR2 _____ E. Hay fever 5. DR3 _____ F. Hashimotos thyroiditis 6. DR4 _____ G. Hemochromatosis 7. DR5 _____ H. Inflammatory bowel disease 8. DR7 _____ I. Multiple sclerosis _____ J. Pernicious anemia _____ K. Psoriasis _____ L. Reiters syndrome _____ M. Rheumatoid arthritis _____ N. Steroid-responsive nephrotic syndrome _____ O. Systemic lupus erythematosus 15. Natural killer cell activity is enhanced by what three cytokines? (p. 224) _____________________ 16. _______________ (B cells/T cells) are involved in hyperacute organ rejection, while _______________ (B cells/T cells) are involved in chronic organ rejection. (p. 225) 17. _______________ (Th1/Th2) cells are involved in the cell-mediated immune response, while _______________ (Th1/Th2) cells are involved in the humoral response. (p. 226). 18. In the chart below, checkmark whether the functions below are mediated by Th1 cells or Th2 cells. (p. 226)
file:///D|/USMLE%20Vids/Workbooks%202012/Chapter%205%20-%20Immunology%202012.txt[8/26/2013 2:13:13 AM]

Function Th1 Cells Th2 Cells Antibody production (via B cells)

CD8 T-cell activation

Interferon-. secretion

IL-2 secretion

IL-4 secretion

IL-5 secretion

IL-10 secretion

Macrophage activation

19. Name the three enzymes used by CD8+ T cells to kill infected, neoplastic, and donor graft cells. (p. 227) _______________________________________________________________________ 20. What are the two major anti-inflammatory cytokines produced by regulatory T cells? (p. 227) ______________________________________________________________________________ 21. By which four mechanisms is antibody diversity generated? (p. 227) ________________________ ______________________________________________________________________________ 22. Match each immunoglobulin isotype with its characteristic. (Numbers may be used more than once.) (p. 228) _____ A. Activates eosinophils 1. IgA _____ B. Antigen receptor on B cell surface 2. IgD _____ C. Binds mast cells and basophils 3. IgG _____ D. Fixes complement and crosses the placenta 4. IgE _____ E. Fixes complement but does not cross the placenta 5. IgM _____ F. Function is unclear _____ G. Main antibody in primary immune response
file:///D|/USMLE%20Vids/Workbooks%202012/Chapter%205%20-%20Immunology%202012.txt[8/26/2013 2:13:13 AM]

_____ H. Main antibody in secondary immune response _____ I. Mediates type I hypersensitivity _____ J. Neutralizes bacterial toxins and viruses _____ K. Opsonizes bacteria _____ L. Prevents attachment of bacteria and viruses to mucous membranes

IMMUNE RESPONSES 23. In the chart below, identify the component(s) of the complement system associated with each action. (p. 229) Function Complement Anaphylaxis Cytolysis by MAC Opsonization Neutrophil chemotaxis Viral neutralization Primary opsonins in bacterial defense

24. What activates the alternative, lectin, and classic pathways? (p. 229) ________________________ ______________________________________________________________________________ 25. Which cytokines are secreted by macrophages? (p. 230) _________________________________ 26. Which cytokines are secreted by T cells? (p. 230) _______________________________________ 27. Match the cytokine with its action(s). (p. 230) _____ A. Activates macrophages and Th1 cells, suppresses Th2 cells 1. IL-1 _____ B. Causes fever and acute inflammation, induces chemokine 2. IL-2 secretion 3. IL-3 _____ C. Causes fever and stimulates production of acute-phase proteins 4. IL-4 _____ D. Induces differentiation of T cells to Th1 cells, activates NK cells 5. IL-5 _____ E. Induces differentiation of T cells to Th2 cells, enhances class 6. IL-6 switching to IgE and IgG 7. IL-8 _____ F. Major chemotactic factor for neutrophils 8. IL-10 _____ G. Modulates inflammatory response, inhibits Th1 cells 9. IL-12 _____ H. Mediates septic shock, recruits leukocytes 10. INF-. _____ I. Promotes B cell differentiation, enhances class switching to IgA 11. TNF-a _____ J. Stimulates growth of Th and Tc cells _____ K. Supports growth and differentiation of bone marrow stem cells

file:///D|/USMLE%20Vids/Workbooks%202012/Chapter%205%20-%20Immunology%202012.txt[8/26/2013 2:13:13 AM]

28. In the chart below, checkmark which cell-surface proteins and receptors are expressed by which type of cell. (p. 231) Protein/Receptor B cells T cells Macrophages NK cells B7

C3b receptor

CD3

CD4

CD8

CD14

CD16 receptor

file:///D|/USMLE%20Vids/Workbooks%202012/Chapter%205%20-%20Immunology%202012.txt[8/26/2013 2:13:13 AM]

CD19

CD20

CD21

CD28

CD40

CD40L

CD56 receptor

Fc receptor

IgG

file:///D|/USMLE%20Vids/Workbooks%202012/Chapter%205%20-%20Immunology%202012.txt[8/26/2013 2:13:13 AM]

MHC I receptor

MHC II

TCR

29. Which type of vaccination induces humoral immunity? (p. 232) ____________________________ 30. Which type of hypersensitivity reaction is mediated by T cells? (p. 233) ______________________ 31. Which type of hypersensitivity reaction is mediated by antibody? (p. 233) ____________________ 32. Which type of hypersensitivity reaction is mediated by an immune complex? (p. 233) ___________ 33. Which type of hypersensitivity reaction occurs rapidly due to preformed antibody? (p. 233) _______

34. Match the type of hypersensitivity reaction with the conditions it mediates. (Numbers can be used more than once.) (p. 234) _____ A. Allergic rhinitis 1. Type I _____ B. Anaphylaxis 2. Type II _____ C. Anemia (pernicious, hemolytic) 3. Type III _____ D. Arthus reaction 4. Type IV _____ E. Asthma _____ F. Bullous phemphigoid _____ G. Contact dermatitis _____ H. Diabetes mellitus type 1 _____ I. Erythroblastosis fetalis _____ J. Hashimotos thyroiditis _____ K. Hypersensitivity pneumonitis _____ L. Goodpastures syndrome _____ M. Graft-vs-host disease _____ N. Graves disease _____ O. Gullain-Barr syndrome _____ P. Multiple sclerosis
file:///D|/USMLE%20Vids/Workbooks%202012/Chapter%205%20-%20Immunology%202012.txt[8/26/2013 2:13:13 AM]

_____ Q. Myasthenia gravis _____ R. Polyarteritis nodosum _____ S. Poststreptococcal glomerulonephritis _____ T. Rheumatic fever _____ U. Rheumatoid arthritis _____ V. Serum sickness _____ W. Systemic lupus erythematosus 35 . Match the autoantibody with its associated disorder. (p. 235) _____ A. Autoimmune hepatitis 1. Antibasement membrane _____ B. Celiac disease 2. Anticentromere _____ C. Dermatomyositis, polymyositis 3. Anti-desmoglein _____ D. Diabetes mellitus type 1 4. Anti-dsDNA, anti-Smith _____ E. Drug-induced lupus 5. Antigliadin, antiendomyosial _____ F. Goodpastures syndrome 6. Antiglutamate decarboxylase _____ G. Hashimotos thyroiditis 7. Antihistone _____ H. Mixed connective tissue disease 8. Anti-IgG _____ I. Vasculitides (other than Wegeners) 9. Anti-Jo-1 _____ J. Pemphigus vulgaris 10. Anti-microsomal, antithyroglobulin _____ K. Primary biliary cirrhosis 11. Antimitochondrial _____ L. Rheumatoid arthritis 12. Antinuclear antibodies _____ M. Scleroderma (CREST) 13. Anti-Scl-70 _____ N. Scleroderma (diffuse) 14. Antismooth muscle _____ O. Sjgrens syndrome 15. Anti-SS-A, anti-SS-B _____ P. SLE 16. Anti-U1 RNP _____ Q. SLE, nonspecific 17. c-ANCA _____ R. Wegeners granulomatosis 18. p-ANCA

36. What immune deficiency is most closely associated with each clinical scenario? (pp.237-238) A. A 9-year-old boy presents with recurrent sinusitis, otitis media, and erythematous skin infections. _________________________________________________________________ B. An infant is brought to his pediatrician for the sixth time in several months. Upper respiratory infection had been diagnosed previously, and he underwent incision and drainage of several buttock abscesses. Today he is fussy and febrile, and has rhinorrhea and demarcated erythema in the skin folds._____________________________________________________ C. An infant arrives for her 2-month well-child visit. Her abdomen is soft and nontender, but her umbilical remnant is still present. A red, firm area is present on the back of her thigh, with no evidence of fluctuance. _______________________________________________________ D. A 5-year-old presents because of recurrent skin infections. In the past, she has been treated for an E. coli urinary tract infection as well as numerous skin infections. Results of todays nitroblue tetrazolium test are negative. ___________________________________________ E. A 9-year-old boy presents with coarse facies and a rash. Physical examination reveals he has two sets of teeth where his adult dentitia have erupted. ___________________________ 37. Which autosomal-recessive immune deficiency presents with recurrent pyogenic staphylococcal and streptococcal infections, partial albinism, and peripheral neuropathy? (p. 238) ____________ ______________________________________________________________________________ 38. Which immune deficiency presents with a triad of symptoms that include recurrent pyogenic infections, thrombocytopenic purpura, and eczema? (p. 238) _____________________________ 39. Name three possible causes of severe combined immunodeficiency. (pp 237-238) ____________ ______________________________________________________________________________
file:///D|/USMLE%20Vids/Workbooks%202012/Chapter%205%20-%20Immunology%202012.txt[8/26/2013 2:13:13 AM]

40. What are the four signs and symptoms of graft-vs-host disease? (p. 239) ____________________ ______________________________________________________________________________ IMMUNOSUPPRESSANTS 41. What is the mechanism of action of cyclosporine? (p. 240) ________________________________ 42. What is the mechanism of action of sirolimus, and what effect does this have on immunity? (p. 240) ______________________________________________________________________________ 43. Which two antibodies are used to treat Crohns disease, rheumatoid arthritis, and psoriatic arthritis? (p. 241) ________________________________________________________________________

Answers LYMPHOID STRUCTURES 1. Medulla. 2. Medullary sinuses. 3. Paracortex. 4. Follicles. 5. Outer cortex. 6. Paracortex. 7. A-7, B-8, C-8, D-1, E-5, F-4, G-7, H-3, I-2, J-7, K-6, L-1. 8. Howell-Jolly bodies, target cells, and thrombocytosis. 9. Salmonella, S. pneumonia, H. influenza, and N. meningitides. 10. Bone marrow; thymus. LYMPHOCYTES 11. Component Adaptive Immune System Innate Immune System Antibody v B cells v Complement v Dendritic cells v Macrophages v Natural killer cells v
file:///D|/USMLE%20Vids/Workbooks%202012/Chapter%205%20-%20Immunology%202012.txt[8/26/2013 2:13:13 AM]

Neutrophils v T cells v

12. HLA-A, HLA-B, HLA-C. 13. HLA-DR, HLA-DP, HLA-DQ. 14. A-3, B-5, C-4, D-2, E-4, F-7, G-1, H-3, I-4, J-7, K-3, L-3, M-6, N-8, O-4. 15. IL-12, IFN-, IFN-a. 16. B cells; T cells. 17. Th1; Th2. 18. Function Th1 Cells Th2 Cells Antibody production (via B cells) v CD8 T-cell activation v Interferon-. secretion v IL-2 secretion v IL-4 secretion v IL-5 secretion v IL-10 secretion v Macrophage activation v

file:///D|/USMLE%20Vids/Workbooks%202012/Chapter%205%20-%20Immunology%202012.txt[8/26/2013 2:13:13 AM]

19. Perforins, granzymes, and granulysins. 20. IL-10 and TGF-. 21. Random recombination of VJ or VDJ genes; random combination of heavy and light chains; somatic hypermutation; and addition of nucleotides to DNA during genetic recombination by terminal deoxynucleotidyl transferase 22. A-4, B-5, C-4, D-3, E-5, F-2, G-5, H-3, I-4, J-3, K-3, L-1. IMMUNE RESPONSES 23. Function Complement Anaphylaxis C3a, C5a Cytolysis by MAC C5b-9 Opsonization C3b Neutrophil chemotaxis C5a Viral neutralization C1, C2, C3, C4 Primary opsonins in bacterial defense IgG and C3b

24. The alternative and lectin pathways are activated by substances (e.g., endotoxin) on microbial surfaces, and the classic pathway is activated by antigen-antibody complexes. 25. IL-1, IL-6, IL-8, IL-12, and TNF-... 26..IL-3, IL-2, interferon-., IL-4, IL-5, and IL-10. 27. A-10, B-1, C-6, D-9, E-4, F-7, G-8, H-11, I-5, J-2, K-3. 28. Protein/Receptor B cells T cells Macrophages NK cells B7 v v C3b receptor

file:///D|/USMLE%20Vids/Workbooks%202012/Chapter%205%20-%20Immunology%202012.txt[8/26/2013 2:13:13 AM]

v CD3 v

CD4 v

CD8 v

CD14 v CD16 receptor

v CD19 v

CD20 v

CD21 v

CD28

file:///D|/USMLE%20Vids/Workbooks%202012/Chapter%205%20-%20Immunology%202012.txt[8/26/2013 2:13:13 AM]

CD40 v v CD40L v

CD56 receptor

v Fc receptor v IgG v

MHC I receptor

v MHC II v v TCR v

file:///D|/USMLE%20Vids/Workbooks%202012/Chapter%205%20-%20Immunology%202012.txt[8/26/2013 2:13:13 AM]

29. Inactivated or killed vaccine. 30. Type IV. 31. Type II. 32. Type I. 33. Type III. 34. A-1, B-1, C-2, D-3, E-1, F-2, G-4, H-4, I-2, J-4, K-3, L-2, M-4, N-2, O-4, P-4, Q-2, R-3, S-3, T-2, U-3, V-3, W3. 35. A-14, B-5, C-9, D-6, E-7, F-1, G-10, H-16, I-18, J-3, K-11, L-8, M-2, N-13, O-15, P-4, Q-12, R-17. 36. A = Brutons agammaglobulinemia; B = severe combined immunodeficiency; C = leukocyte adhesion deficiency; D = chronic granulomatous disease; E = Jobs syndrome. 37. Chdiak-Higashi syndrome. 38. Wiskott-Aldrich syndrome. 39. Defective IL-2 receptor, adenosine deaminase deficiency, and failure to synthesize MHC II. 40. Maculopapular rash, jaundice, hepatosplenomegaly, and diarrhea. IMMUNOSUPPRESSANTS 41. Inhibits calcineurin and prevents production of IL-2 and the IL-2 receptor. 42. Binds mammalian target of rapamycin (mTOR) and inhibits T-cell proliferation in response to IL-2. 43. Infliximab and adalimumab.

file:///D|/USMLE%20Vids/Workbooks%202012/Chapter%205%20-%20Immunology%202012.txt[8/26/2013 2:13:13 AM]

Pathology

How to Use the Workbook with the Videos

Video Part Questions Pathology Inflammation part 1 13 Pathology Inflammation part 2 47 Pathology Inflammation part 3 89 Pathology Neoplasia part 1 1011 Pathology Neoplasia part 2 1218 Pathology Neoplasia part 3 1922

Questions INFLAMMATION 1. Describe the fundamental differences between the intrinsic and extrinsic pathways of apoptosis. Name two important similarities between the pathways. (p. 244) ___________________________ ______________________________________________________________________________ ______________________________________________________________________________ ______________________________________________________________________________ 2. Describe the fundamental differences between apoptosis and necrosis. What are the six types of necrosis? List an example of each. (p. 244) ___________________________________________ ______________________________________________________________________________ ______________________________________________________________________________ ______________________________________________________________________________ 3. Name three organs that manifest irreversible ischemia with red infarcts. Name three that show pale infarcts. (p. 245) _________________________________________________________________ ______________________________________________________________________________ 4. In the chart below, compare and contrast the characteristics of hypovolemic/cardiogenic shock vs. septic shock. (p. 245) Hypovolemic/

file:///D|/USMLE%20Vids/Workbooks%202012/Chapter%206%20-%20Pathology%202012.txt[8/26/2013 2:13:15 AM]

Cardiogenic Shock Septic Shock Cardiac output

Clinical appearance

Total peripheral resistance

5. What are the five signs and symptoms of inflammation? (p. 246) ___________________________ ______________________________________________________________________________

6. In the chart below, compare and contrast the ligand-receptor interactions required for each step of leukocyte extravasation. (p. 247) Ligand-Receptor Interaction Endothelial Cells Leukocytes Rolling

Tight binding

Diapedesis

Migration

7. What are the three ways that free radicals can be eliminated? (p. 247) ______________________ ______________________________________________________________________________
file:///D|/USMLE%20Vids/Workbooks%202012/Chapter%206%20-%20Pathology%202012.txt[8/26/2013 2:13:15 AM]

8. In the chart below, compare and contrast the characteristics of transudates and exudates. (p. 248) Transudate Exudate Causes

Cellularity

Protein level

Specific gravity

9. What three conditions are associated with a low erythrocyte sedimentation rate? (p. 248) _______ ______________________________________________________________________________ NEOPLASIA 10. Define the following terms and provide an example of each. (pp. 250-251) A. Hyperplasia_________________________________________________________________ __________________________________________________________________________ B. Metaplasia _________________________________________________________________ __________________________________________________________________________ C. Dysplasia __________________________________________________________________ __________________________________________________________________________ D. Anaplasia __________________________________________________________________ __________________________________________________________________________ E. Neoplasia _________________________________________________________________ __________________________________________________________________________ 11. Describe the differences between tumor grade and tumor stage. (p. 251) ____________________ ______________________________________________________________________________ ______________________________________________________________________________ 12. Compare and contrast the characteristics of benign vs. malignant tumors. (p. 251) Characteristic Benign Tumor Metastatic Tumor Differentiated?

Growth

file:///D|/USMLE%20Vids/Workbooks%202012/Chapter%206%20-%20Pathology%202012.txt[8/26/2013 2:13:15 AM]

Distinct boundaries?

Metastatic potential?

13. Match the neoplasm(s) to the condition(s) with which it is most commonly associated. (p. 252) _____ A. Acute lymphoblastic leukemia 1. Acanthosis nigricans _____ B. Astrocytoma, angiomyolipoma, cardiac 2. Actinic keratosis rhabdomyoma 3. AIDS _____ C. Colonic adenocarcinoma 4. Autoimmune diseases _____ D. Gastric adenocarcinoma 5. Barretts esophagus _____ E. Esophageal adenocarcinoma 6. Chronic atrophic gastritis, pernicious _____ F. Hepatocellular carcinoma anemia, postsurgical gastric _____ G. Lymphoma remnants _____ H. Malignant lymphoma 7. Cirrhosis _____ I. Malignant melanoma 8. Down syndrome _____ J. Melanoma, basal cell carcinoma, and 9. Dysplastic nevus squamous cell carcinoma of skin 10. Immunodeficiencies _____ K. Non-Hodgkins lymphoma and Kaposis 11. Pagets disease of bone sarcoma 12. Plummer-Vinson syndrome _____ L. Sarcoma and papillary thyroid cancer 13. Radiation exposure _____ M. Secondary osteosarcoma and fibrosarcoma 14. Tuberous sclerosis _____ N. Squamous cell carcinoma of esophagus 15. Ulcerative colitis _____ O. Squamous cell carcinoma of skin 16. Xeroderma pigmentosum, albinism _____ P. Visceral malignancy 14. Oncogenes are associated with a ______ (gain/loss) of function and require damage to ______ (one/both) allele(s); examples include _________________________. In contrast, tumor suppressor genes are associated with a ______ (gain/loss) of function and require damage to ______ (one/both) allele(s); examples include _________________________. (p. 253) 15. How are tumor markers best used? (p. 254) ___________________________________________ ______________________________________________________________________________ 16. A 40-year-old otherwise healthy man is diagnosed with nasopharyngeal carcinoma. He does not smoke or drink. What is the most likely cause of his cancer? (p. 254) ________________________ 17. An IV drug abuser who is being monitored for cirrhosis shows a sudden increase in his a-fetoprotein level. For which disease is he at increased risk? (p. 254) ________________________ 18. The vaccine Gardasil protects against which viruses that can cause which cancer? (p. 254) ____ ______________________________________________________________________________ 19. A 70-year-old who eats smoked salmon every day presents with abdominal pain and loss of appetite. Which diagnosis should be high on the differential? (p. 255) _______________________ 20. A 55-year-old woman with a 40-pack-year history of cigarette smoking presents with new-onset cough, hemoptysis, and polyuria. What diagnosis should be high on the differential? (p. 255) ______________________________________________________________________________ 21. Which cancers are most common in men? In women? (p. 256) ____________________________ ______________________________________________________________________________ 22. In the chart below, checkmark which primary tumors can metastasize to which organ(s). (p. 256) Primary Tumor Metastasize to Brain Metastasize to Liver
file:///D|/USMLE%20Vids/Workbooks%202012/Chapter%206%20-%20Pathology%202012.txt[8/26/2013 2:13:15 AM]

Metastasize to Bone Breast

Colon

Gastrointestinal

Kidney

Lung

Pancreas

Prostate

Skin

Stomach

Testes

Thyroid

file:///D|/USMLE%20Vids/Workbooks%202012/Chapter%206%20-%20Pathology%202012.txt[8/26/2013 2:13:15 AM]

Answers INFLAMMATION 1. The intrinsic pathway begins with changes in the levels of anti- and pro-apoptotic factors, leading to increased mitochondrial permeability. The extrinsic pathway begins with ligand-receptor interactions or perforin/granzyme release. Similarities: both require ATP, and both ultimately activate caspases. 2. Apoptosis occurs without any inflammation, whereas necrosis causes local inflammation. The six types of necrosis are coagulative (as occurs in the heart), liquefactive (bacterial abscess), caseous (e.g., due to systemic fungal infection), fatty (saponification of pancreas), fibrinoid (as occurs in blood vessels), and gangrenous (limb gangrene). 3. Red infarct: liver, lungs, and intestine. Pale infarcts: heart, kidney, and spleen. 4. Hypovolemic/ Cardiogenic Shock Septic Shock Cardiac output Low High Clinical appearance Cold and clammy Hot Total peripheral resistance Increased Decreased

5. Rubor (redness), dolor (pain), calor (heat), tumor (swelling), and functio laesa (loss of function). 6. Ligand-Receptor Interaction Endothelial Cells Leukocytes Rolling E-selection P-selectin Sialyl Lewis Tight binding ICAM-1 LFA-1 Diapedesis
file:///D|/USMLE%20Vids/Workbooks%202012/Chapter%206%20-%20Pathology%202012.txt[8/26/2013 2:13:15 AM]

PECAM-1 PECAM-1 Migration C5a IL-8 LTB4 Kallikrein Various

7. By enzymes, by spontaneous decay, and by antioxidants. 8. Transudate Exudate Causes Increased hydrostatic pressure Decreased oncotic pressure Sodium retention Lymphatic obstruction Inflammation Cellularity Hypocellular Cellular Protein level Low High Specific gravity Low High

9. Congestive heart failure, sickle cell anemia, and polycythemia. NEOPLASIA 10. A. Hyperplasia: increase in the number of cells. Examples: parathyroid hyperplasia, benign prostatic hyperplasia, congenital adrenal hyperplasia. B. Metaplasia: one type of cell is replaced by another. Examples: Barretts esophagus, squamous metaplasia of the airways. C. Dysplasia: abnormal cells. Examples: cervical dysplasia, fibrous dysplasia of bone, congenital retinal dysplasia. D. Anaplasia: lacking differentiation. Examples: anaplastic oligodendroglioma, anaplastic thyroid tumors, many other subtypes of tumors. E. Neoplasia: new growth, either benign or malignant. Examples: uterine fibroids, nevi, malignant cancers. 11. Characteristic Benign Tumor
file:///D|/USMLE%20Vids/Workbooks%202012/Chapter%206%20-%20Pathology%202012.txt[8/26/2013 2:13:15 AM]

Metastatic Tumor Differentiated? Well-differentiated May be poorly differentiated Growth Slow Erratic Distinct boundaries? Yes Diffuse or locally invasive Metastatic potential? No Yes

12. Tumor grade is the degree of cellular differentiation within the tumor, and is a characteristic of the tumor itself. In contrast, tumor stage describes the extent of tumor spread within a patient, and thus is a better indication of a patients prognosis than tumor grade. 13. A-8, B-14, C-15, D-6, E-5, F-7, G-4, H-10, I-9, J-16, K-3, L-13, M-11, N-12, O-2, P-1. 14. Oncogenes are associated with a gain of function and require damage to only one allele for expression; examples include c-myc (Burkitts lymphoma) and ras (colon carcinoma). Tumor suppressor genes are associated with a loss of function and require damage to both alleles for expression; examples include NF1 (neurofibromatosis type 1) and BRCA2 (breast cancer). 15. To confirm a diagnosis, to check for tumor recurrent, and to monitor response to therapy. 16. Epstein-Barr Virus (EBV). 17. Hepatocellular carcinoma. IV drug use and cirrhosis are associated with HCV, and HCV is associated with hepatocellular carcinoma. 18. HPV 6, 11, 16, and 18. HPV 16 and 18 have been associated with cervical cancer. 19. Gastric cancer. Smoked foods contain large amounts of nitrosamine. 20. Small cell lung carcinoma with paraneoplastic SIADH secretion. 21. Prostate, lung, and colon/rectal cancers are the most common cancers in men; breast, lung, and colon/rectal cancers are the most common in men. 22. Primary Tumor Metastasize to Brain Metastasize to Liver Metastasize to Bone Breast v v v Colon v Gastrointestinal
file:///D|/USMLE%20Vids/Workbooks%202012/Chapter%206%20-%20Pathology%202012.txt[8/26/2013 2:13:15 AM]

Kidney v v Lung v v v Pancreas v Prostate v Skin v

Stomach v Testes v Thyroid v

file:///D|/USMLE%20Vids/Workbooks%202012/Chapter%206%20-%20Pathology%202012.txt[8/26/2013 2:13:15 AM]

Pharmacology

How to Use the Workbook with the Videos

Video Part Questions Pharmacology Pharmacodynamics 130 Pharmacology Autonomic Drugs part 1 3144 Pharmacology Autonomic Drugs part 2 4563 Pharmacology Toxicities & Side Effects 6478 Pharmacology Miscellaneous 79

Questions

PHARMACODYNAMICS 1. Competitive inhibitors ________ (do/do not) resemble the substrate, while noncompetitive inhibitors ________ (do/do not) resemble the substrate. (p. 258) 2. The value of Km reflects the _______________ of the enzyme for its substrate. (p. 258) 3. True or False: In enzyme kinetics, the lower the Km, the higher the affinity. (p. 258) _____________ 4. Vmax is directly proportional to the _______________ _______________. (p. 258) 5. A graph of substrate concentration on the x-axis and velocity of the reaction on the y-axis has _______________ (increasing/decreasing) velocity as substrate is increased, although it will plateau when the enzyme is saturated. (p. 258) 6. When velocity is equal to one half of its maximum (Vmax), the corresponding concentration of substrate is equal to what value? (p. 258) _____________________________________________ 7. Use the graph below to answer the following questions. (p. 258) A. What pharmacodynamic term describes the x-intercept of the line? _____________________ B. What pharmacodynamic term describes the y-intercept? ____________________________

file:///D|/USMLE%20Vids/Workbooks%202012/Chapter%207%20-%20Pharmacology%202012.txt[8/26/2013 2:13:16 AM]

C. If the y-intercept increases, how is the maximum reaction rate affected? _________________ D. If the x-intercept moves to the right (increases), how is the Km affected? _________________ 8. In enzyme kinetics, a competitive inhibitor ________ (can/cannot) be overcome by increasing the concentration of substrate; a noncompetitive inhibitor ________ (can/cannot) be overcome by increasing the concentration of substrate. (p. 258) 9. Competitive inhibitors _______________ (increase/decrease/do not change) the Vmax of the reaction, whereas noncompetitive inhibitors _______________ (increase/decrease/do not change) the Vmax of the reaction. (p. 258) 10. Competitive inhibitors _______________ (increase/decrease/do not change) the Km of the reaction, whereas noncompetitive inhibitors _______________ (increase/decrease/do not change) the Km of the reaction. (p. 258) 11. What is the formula for calculating a drugs volume of distribution? (p. 259) ___________________ 12. Drugs with a low volume of distribution are found in the _______________ (blood/tissue/extracellular space). Drugs with a high volume of distribution are most likely found in the _______________ (blood/tissue/extracellular space). (p. 259) 13. What is the formula for calculating a drugs clearance? (p. 259) ____________________________ 14. What is the definition of the half-life of a drug? (p. 259) ___________________________________ 15. For a drug that is infused at a constant rate, how many half-lives must pass before the drug reaches approximately 94% of steady-state concentration? (p. 259) ________________________ 16. Given the volume of distribution and clearance of a drug, how is the drugs half-life calculated? (p. 259) __________________________________________________________________________ 17. After one half-life, given constant intravenous infusion of a drug, how close to steady state is the drugs concentration? How close is it after three half-lives? (p. 259) _________________________ 18. What is the formula for calculating a drugs loading dose? (p. 259) __________________________ 19. What is the formula for calculating the maintenance dose of a drug administered intravenously? (p. 259) __________________________________________________________________________ 20. How do the loading and maintenance doses of drugs differ for patients with renal or liver disease? (p. 259) ________________________________________________________________________ 21. What is the bioavailability (%) of a drug if it is administered intravenously? (p. 259) _____________ 22. In zero-order elimination of drugs from the body, what is the relationship between the rate of elimination and the drug concentration? (p. 260) ________________________________________ 23. Name three drugs that exhibit zero-order elimination. (p. 260) _____________________________ 24. In first-order drug elimination, what is the relationship between the rate of elimination and the drug concentration? (p. 260) ___________________________________________________________ 25. A 24-year-old man attempts suicide by consuming the contents of a small bottle of aspirin. Three hours later he is brought to the emergency room, where he is administered intravenous saline with bicarbonate. By what mechanism does this help him? (p. 260) _____________________________ ______________________________________________________________________________ 26. A drug that requires a very low dose to achieve its desired effect is _______________ (effective/potent). (p. 261)

27. The graph below shows the effects of two types of antagonists on an agonist. What type of antagonist is represented by curve A? By curve B? (p. 261) _______________________________ 28. The addition of a noncompetitive agonist _______________ (increases/decreases/does not change) the efficacy of the agonist. (p. 261) 29. How does the efficacy of a partial agonist relate to the efficacy of a full agonist of the same receptor? (p. 261) ________________________________________________________________ 30. How does the potency of a partial agonist relate to the potency of a full agonist of the same receptor? (p. 261) ________________________________________________________________ AUTONOMIC DRUGS 31. How does botulinum toxin work? (p. 262) _____________________________________________ 32. Identify the G-protein class for each receptor. (Numbers may be used more than once.) (p. 263) _____ A. a1 1. Gi _____ B. a2 2. Gq
file:///D|/USMLE%20Vids/Workbooks%202012/Chapter%207%20-%20Pharmacology%202012.txt[8/26/2013 2:13:16 AM]

_____ C. 1 3. Gs _____ D. 2 _____ E. D1 _____ F. D2 _____ G. H1 _____ H. H2 _____ I. M1 _____ J. M2 _____ K. M3 _____ L. V1 _____ M. V2 33. What are the major effects of a1-receptor activation? (p. 263) ______________________________ ______________________________________________________________________________ 34. What are the major functions of a2-receptor activation? (p. 263) ___________________________ ______________________________________________________________________________ 35. What are the major functions of 1-receptor activation? (p. 263) ____________________________ ______________________________________________________________________________ 36. What is the major effect of 2-receptor activation on the body's vasculature? What is the effect on the respiratory system? (p. 263) _____________________________________________________ 37. How does 2-receptor activation affect insulin release? (p. 263) ____________________________ 38. In the images below, identify which autonomic drugs work at which site of action. (p. 264) (Adapted, with permission, from Katzung BG, Trevor AJ. Pharmacology: Examination & Board Review, 5th ed. Stamford, CT: Appleton & Lange, 1998: 42.) 39. Name five indirect cholinergic agonists. (p. 265) ________________________________________ ______________________________________________________________________________ 40. What symptoms are likely in patients taking cholinomimetic agents? (p. 265) _________________ ______________________________________________________________________________ 41. Which pharmacologic agent is used to treat atropine overdose? (p. 265) _____________________ 42. What is a methacholine challenge test? (p. 265) ________________________________________ ______________________________________________________________________________ 43. A farmer presents with diarrhea, abdominal pain, wheezing, pinpoint pupils, copious tears, and salivation. What medications should be prescribed? (p. 265) ______________________________ 44. Why is pyridostigmine used to treat myasthenia gravis? (p. 265) ___________________________ ______________________________________________________________________________ 45. A patient recently began taking haloperidol to treat schizophrenia, but visits his physician because of new-onset Parkinson's-like motor symptoms. What drug could be used to treat these symptoms? (p. 266) ________________________________________________________________________ 46. What are the two effects of atropine on the eye? (p. 266) _________________________________ 47. True or False: Diarrhea is a sign of atropine toxicity. (p. 266) ______________________________ 48. Low doses of epinephrine are selective for _______ (a1, a2, 1, 2) adrenergic receptors. (p. 266) 49. Isoproterenol is an agonist for which receptors? (p. 266) _________________________________ 50. Dopamine __________ (is/is not) inotropic and __________ (is/is not) chronotropic, whereas dobutamine __________ (is/is not) inotropic and __________ (is/is not) chronotropic. (p. 266) 51. What are the clinical applications of epinephrine? (p. 266) ________________________________ ______________________________________________________________________________ 52. What role does dopamine have in treating shock? (p. 266) ________________________________ ______________________________________________________________________________ 53. What are the clinical applications of phenylephrine? (p. 266) ______________________________ ______________________________________________________________________________ 54. What is the clinical application for albuterol? (p. 267) ____________________________________ ______________________________________________________________________________ 55. Which sympathomimetics can reduce premature uterine contractions? (p. 267) _______________ 56. What effect does isoproterenol have on pulse pressure and heart rate? (p. 267) _______________
file:///D|/USMLE%20Vids/Workbooks%202012/Chapter%207%20-%20Pharmacology%202012.txt[8/26/2013 2:13:16 AM]

______________________________________________________________________________ 57. What is the effect of clonidine on central adrenergic outflow? On which receptor does it act? (p. 267) __________________________________________________________________________ 58. What is the clinical application and mechanism of action of phentolamine? (p. 268) _____________ ______________________________________________________________________________ 59. What is the net effect of epinephrine on blood pressure before and after nonselective a-blockade? Why? (p. 268) ___________________________________________________________________ ______________________________________________________________________________ 60. A 63-year-old man is referred long-term care after his first myocardial infarction. Is a -blocker suggested or contraindicated for this patient? Why? (p. 269) ______________________________ ______________________________________________________________________________ 61. How do -blockers work in the setting of angina pectoris? What type of respiratory disease is a major contraindication for their use? (p. 269) __________________________________________ ______________________________________________________________________________ 62. Which -blockers have partial agonist activity? (p. 269) __________________________________ 63. Name two nonselective a- and -antagonists. (p. 269) ___________________________________ TOXICITIES AND SIDE EFFECTS 64. Match the specific antidote(s) with each of the toxicities. (p. 270) _____ A. Acetaminophen 1. 100% oxygen, hyperbaric oxygen _____ B. Amphetamines 2. Aminocaproic acid _____ C. Antimuscarinics and 3. Atropine, pralidoxime anticholinergics 4. -Blocker _____ D. Benzodiazepines 5. CaEDTA, dimercaprol, succimer, penicillamine _____ E. -Blockers 6. Deferoxamine _____ F. Carbon monoxide 7. Dimercaprol, succimer _____ G. Copper, arsenic, gold 8. Ethanol, dialysis, fomepizole _____ H. Cyanide 9. Flumazenil _____ I. Digitalis 10. Glucagon _____ J. Heparin 11. Methylene blue, vitamin C _____ K. Iron 12. N-Acetylcysteine _____ L. Lead 13. NaHCO3 _____ M. Mercury, arsenic, gold 14. NaCHO3, dialysis _____ N. Methanol, antifreeze 15. NH4Cl _____ O. Methemoglobin 16. Nalaxone/naltrexone _____ P. Opioids 17. Nitrite, hydroxocobalamin, thiosulfate _____ Q. Organophosphates, 18. Phosphostigmine salicylate acetylcholinesterase inhibitors 19. Penicillamine _____ R. Salicylates 20. Protamine _____ S. TCAs 21. Stop the drug, normalize K+, lidocaine, anti-dig _____ T. Theophylline Fab fragments, Mg2+ _____ U. tPA, streptokinase 22. Vitamin K, fresh frozen plasma _____ V. Warfarin 65. Which medications can cause agranulocytosis? (p. 271) _________________________________ ______________________________________________________________________________ 66. OCPs can cause what kind of complications? (p. 271) ___________________________________ 67. Which medications can cause hemolysis in patients with G6PD deficiency? (p. 271) ___________ ______________________________________________________________________________ 68. Which medications can cause gynecomastia? (p. 271) __________________________________ ______________________________________________________________________________ 69. Which medications can cause pulmonary fibrosis? (p. 271) _______________________________ ______________________________________________________________________________ 70. Which medications can cause megaloblastic anemia? (p. 271) ____________________________ ______________________________________________________________________________ 71. Which medications can cause photosensitivity? (p. 272) _________________________________
file:///D|/USMLE%20Vids/Workbooks%202012/Chapter%207%20-%20Pharmacology%202012.txt[8/26/2013 2:13:16 AM]

______________________________________________________________________________ 72. Which medications can induce seizures? (p. 272) ______________________________________ ______________________________________________________________________________ 73. Which medications can cause a Parkinson-like syndrome? (p. 272) ________________________ ______________________________________________________________________________ 74. Which medications can cause a disulfiram-like reaction? (p. 272) __________________________ ______________________________________________________________________________ 75. Which medications can cause nephrotoxicity and ototoxicity? (p. 272) ______________________ ______________________________________________________________________________ 76. In the chart below, checkmark which of the substances are P-450 inducers vs. inhibitors. (p. 273) Substance P-450 Inducer P-450 Inhibitor Alcohol use, acute

Alcohol use, chronic

Barbiturates

Carbamazepine

Cimetidine

Erythromycin

Grapefruit juice

Griseofulvin

HIV protease inhibitors

Isoniazid

file:///D|/USMLE%20Vids/Workbooks%202012/Chapter%207%20-%20Pharmacology%202012.txt[8/26/2013 2:13:16 AM]

Ketoconazole

Phenytoin

Quinidine

Rifampin

St. Johns wort

Sulfonamides

77. How does disulfiram work to treat alcoholism? (p. 273) __________________________________ ______________________________________________________________________________ 78. Which drugs must be avoided in patients with sulfa allergy? (p. 273) ________________________ ______________________________________________________________________________ MISCELLANEOUS 79. Match the drug name suffix with its category or usage. (Numbers may be used more than once) (p. 274) _____ A. -afil 1. 5-HT1B/1D agonist _____ B. -ane 2. a1 antagonist _____ C. -azepam 3. ACE inhibitor _____ D. -azine 4. Antibiotic, protein synthesis inhibitor _____ E. -azole 5. Antifungal _____ F. -barbital 6. antagonist _____ G. -caine 7. 2 agonist _____ H. -cillin 8. Barbiturate _____ I. -cycline 9. Benzodiazepine _____ J. -etine 10. Butyrophenone (neuroleptic) _____ K. -ipramine 11. Cardiac glycoside (inotropic) _____ L. -navir 12. Erectile dysfunction _____ M. -olol 13. H2 antagonist _____ N. -operidol 14. Inhalational general anesthetic _____ O. -oxin 15. Local anesthetic _____ P. -pril 16. Penicillin _____ Q. -terol 17. Phenothiazine _____ R. -tidine 18. Pituitary hormone _____ S. -triptan 19. Protease inhibitor _____ T. -triptyline 20. SSRI _____ U. -tropin 21. TCA _____ V. -zolam
file:///D|/USMLE%20Vids/Workbooks%202012/Chapter%207%20-%20Pharmacology%202012.txt[8/26/2013 2:13:16 AM]

_____ W. -zosin Answers PHARMACODYNAMICS 1. Do; do not. 2. Affinity. 3. True. 4. Enzyme concentration. 5. Increasing. 6. Km. 7. A = 1/-Km; B = 1/Vmax; C = it decreases; D = it increases. 8. Can; cannot. This is because competitive inhibitors bind the active site of the enzyme, competing with the substrate, whereas noncompetitive inhibitors bind elsewhere on the enzyme and so are not affected by substrate concentration. 9. Do not change; decrease. 10. Increase; do not change. 11. Volume of distribution = amount of drug in the body / plasma drug concentration. 12. Blood alone (these drugs do not distribute outside the plasma); blood, tissue, and extracellular space (these drugs distribute throughout the body). 13. Clearance (L/min) = rate of elimination of drug (g/min) / plasma drug concentration (g/L). 14. The time required to reduce the amount of drug in the body by one half. 15. Four. 16. Half-life = (0.7 volume of distribution) / clearance. 17. 50% of steady-state concentration; 87.5% of steady-state concentration. 18. Loading dose = (target plasma concentration volume of distribution) / bioavailability. 19. Maintenance dose = (target plasma concentration clearance) / bioavailability. 20. For both diseases, the loading dose does not change, but the maintenance dose decreases. 21. 100%. 22. The rate of elimination is constant, regardless of the drug concentration. 23. Phenytoin, ethanol, and aspirin (at high or toxic concentrations). 24. The rate of elimination is directly proportional to the drug concentration. A constant fraction (rather than a constant amount) is eliminated. 25. Bicarbonate alkalinizes the lumen of the nephrons, which traps acetylsalicylic acid within the lumen because it is a weak acid and is ionized in a basic environment. 26. Potent. 27. A = noncompetitive antagonist; B = competitive antagonist. 28. Decreases. 29. A partial agonist has lower maximal efficacy than a full agonist. 30. A partial agonist may be more potent than, less potent than, or equally as potent as a full agonist. AUTONOMIC DRUGS 31. Botulinum toxin prevents the release of neurotransmitter at all cholinergic terminals by cleaving SNARE proteins critical for exocytosis. 32. A-2, B-1, C-3, D-3, E-3, F-1, G-2, H-3, I-2, J-1, K-2, L-2, M-3. 33. a1-Receptor activation increases vascular smooth muscle contraction, as well as pupillary dilator muscle contraction (mydriasis). 34. a2-Receptor activation decreases sympathetic outflow and insulin release, while increasing glucagon release. 35. 1-Receptor activation increases the following: heart rate and contractility; renin release from the kidneys; and lipolysis of adipose tissue. 36. Vasodilation; bronchodilation. 37. 2-Receptor activation increases glucagon release. 38. (Adapted, with permission, from Katzung BG, Trevor AJ. Pharmacology: Examination &
file:///D|/USMLE%20Vids/Workbooks%202012/Chapter%207%20-%20Pharmacology%202012.txt[8/26/2013 2:13:16 AM]

Board Review, 5th ed. Stamford, CT: Appleton & Lange, 1998: 42.) 39. Neostigmine, pyridostigmine, edrophonium, physostigmine, and echothiophate. 40. Exacerbation of COPD, asthma, and peptic ulcers. Also increasees urination, diarrhea, miosis, bradycardia, etc. 41. Physostigmine. It crosses the blood-brain barrier and is able to reverse effects on the CNS and the peripheral nervous system. 42. A test in which methacholine is inhaled to stimulate muscarinic receptors and induce broncho-constriction. The test is used to diagnose asthma. 43. This patient has the classic signs of organophosphate poisoning, which is treated with atropine and pralidoxime. 44. Pyridostigmine increases the amount of acetylcholine in the neuromuscular synapse, thereby increasing muscle strength. 45. Benztropine. 46. Pupil dilation and cycloplegia. 47. False. (Constipation is a sign of atropine toxicity.) 48. 1. 49. 1 and 2 (equally). 50. Dopamine is inotropic and chronotropic; dobutamine is also inotropic but is not chronotropic. 51. Anaphylaxis, open-angle glaucoma, asthma, and hypotension. 52. Dopamine increases blood pressure while maintaining renal perfusion. 53. Phenylephrine treats nasal decongestion, causes vasoconstriction, and dilates pupils. 54. Acute asthma. 55. Terbutaline and salmeterol. 56. Isoproterenol increases pulse pressure and heart rate. 57. Clonidine is an a2-agonist that decreases central adrenergic outflow. (Remember: the a2-receptor is responsible for negative feedback). 58. Phentolamine is a nonselective a-blocker used to treat pheochromocytoma. 59. Before a-blockade, epinephrine increases blood pressure. After a-blockade, it decreases blood pressure. This is because epinephrine also activates 2, which lowers blood pressure and is not blocked. 60. Suggested. After myocardial infarction, patients should receive -blockers to decrease risk of mortality. 61. They decrease heart rate and contractility thereby reducing myocardial oxygen consumption. Asthma. 62. Pindolol and acebutolol. (Remember: PA = Partial Agonist) 63. Carvedilol and labetalol. TOXICITIES AND SIDE EFFECTS 64. A-12, B-15, C-18, D-9, E-10, F-1, G-19, H-17, I-21, J-20, K-6, L-5, M-7, N-8, O-11, P-16, Q-3, R-14, S-13, T-4, U-2, V-22. 65. Carbamazepine, colchicine, clozapine, dapsone, methimazole, and propylthiouracil. 66. Thrombotic complications. 67. Isoniazid, sulfonamides, primaquine, aspirin, ibuprofen, and nitrofurantoin. 68. Spironolactone, digitalis, cimetidine, chronic alcohol use, estrogens, and ketoconazole. 69. Bleomycin, amiodarone, and busulfan. 70. Phenytoin, methotrexate, and sulfa drugs. 71. Sulfonamides, amiodarone, and tetracycline. 72. Bupropion, imipenem/cilastatin, and isoniazid. 73. Chlorpromazine, haloperidol, metoclopramide, and reserpine. 74. Certain cephalosporins, first-generation sulfonylureas, metronidazole, and procarbazine. 75. Aminoglycosides, cisplatin, loop diuretics, and vancomycin. 76. Substance P-450 Inducer P-450 Inhibitor Alcohol use, acute v

file:///D|/USMLE%20Vids/Workbooks%202012/Chapter%207%20-%20Pharmacology%202012.txt[8/26/2013 2:13:16 AM]

Alcohol use, chronic v Barbiturates v Carbamazepine v Cimetidine v Erythromycin v Grapefruit juice v Griseofulvin v HIV protease inhibitors v Isoniazid v Ketoconazole v Phenytoin v Quinidine v Rifampin v St. Johns wort v
file:///D|/USMLE%20Vids/Workbooks%202012/Chapter%207%20-%20Pharmacology%202012.txt[8/26/2013 2:13:16 AM]

Sulfonamides v

77. Disulfiram inhibits acetaldehyde dehydrogenase, which breaks down acetaldehyde. Thus alcohol consumption while taking disulfiram results in nausea, vomiting, headache, and hypotension. 78. Acetazolamide, celecoxib, furosemide, probenicid, thiazides, TMP-SMX, sulfasalazine, sulfonamide antibiotics, and sulfonylureas. MISCELLANEOUS 79. A-12, B-14, C-9, D-17, E-5, F-8, G-15, H-16, I-4, J-20, K-21, L-19, M-6, N-10, O-11, Q-3, R-7, S-13, T-1, U-21, V-18, W-9, X-2.

file:///D|/USMLE%20Vids/Workbooks%202012/Chapter%207%20-%20Pharmacology%202012.txt[8/26/2013 2:13:16 AM]

Cardiovascular

How to Use the Workbook with the Videos

Video Part Questions Cardiovascular Anatomy & Physiology part 1 16 Cardiovascular Anatomy & Physiology part 2 718 Cardiovascular Anatomy & Physiology part3 1928 Cardiovascular Anatomy & Physiology part 4 2944 Cardiovascular Pathology part 1 4553 Cardiovascular Pathology part 2 5462 Cardiovascular Pathology part 3 6369 Cardiovascular Pathology part 4 7086 Cardiovascular Pharmacology part 1 8792 Cardiovascular Pharmacology part 2 93109

file:///D|/USMLE%20Vids/Workbooks%202012/Chapter%208%20-%20Cardiovascular%202012.txt[8/26/2013 2:13:17 AM]

Questions

ANATOMY 1. If the right coronary artery supplies the inferior portion of the left ventricle via the posterior descending artery, is the heart right or left dominant? (p. 280) _____________________________ 2. A patient has a myocardial infarction that damages the anterior interventricular septum and the apex. Which coronary artery is occluded? (p. 280) ______________________________________ 3. The posterior descending artery arises from the circumflex artery in _____ (20%/50%/80%) of cases. Is this heart right or left dominant? (p. 280) ______________________________________ 4. Enlargement of the left atrium can compress the recurrent laryngeal nerve to cause ____________ or the esophageal nerve to cause __________. (p. 280) PHYSIOLOGY 5. Cardiac output = heart rate x _______________. (p. 280) 6. With an increase in stroke volume, the heart would be expected to _______________ (decrease/increase) in preload, to _______________ (decrease/increase) in afterload, and to _______________ (decrease/increase) in contractility. (p. 281) 7. A 60-year-old man mistakenly receives triple the amount of maintenance intravenous fluids for 24 hours. Would his preload increase or decrease? (p. 281) _________________________________ 8. The cardiac ejection fraction is normally greater than what percentage of the total end-diastolic volume? (p. 282) ________________________________________________________________ 9. Which blood vessels account for most of total peripheral resistance? (p. 282) ______________________________________________________________________________ 10. Which parameter does the viscosity of blood mostly depend on? (p. 282) ______________________________________________________________________________ 11. A 23-year-old man has significant blood loss after a motor vehicle accident. A decrease in blood volume leads to _______________ (increased/decreased) right atrial pressure and to _______________ (increased/decreased) cardiac output. (p. 282) 12. A 76-year-old man with congestive heart failure is given digoxin as a positive inotrope. An increase in inotropy leads to _______________ (increased/decreased) cardiac output and to _______________ (increased/decreased) right atrial pressure. (p. 282) 13. A 10-year-old boy presents with dehydration following acute diarrhea. He receives 2 L of normal saline. An increase in blood volume leads to _________________________ (increased/decreased) right atrial pressure and to __________________________ (increased/decreased) cardiac output. (p. 282) 14. On auscultation of a patient with an atrial septal defect during inspiration, does the time between pulmonic and aortic valvular closure increase, decrease, or stay the same? __________________
file:///D|/USMLE%20Vids/Workbooks%202012/Chapter%208%20-%20Cardiovascular%202012.txt[8/26/2013 2:13:17 AM]

(p.283)_________________________________________________________________________ 15. During physical examination, what diagnostic sign might be observed in the neck of a patient with right heart failure? (p. 283) _________________________________________________________ 16. In normal splitting of the S2 heart sound, the pulmonic valve closes later during inspiration due to _______________ (increased/decreased) blood flow in lungs, and the aortic valve closes earlier during inspiration due to _______________ (increased/decreased) venous return to the left heart. (p. 283) 17. Fill in the blanks A-D with the correct valvular event that occurs at each stage of the left ventricular cardiac cycle. Then fill in the blanks 1-5 with the correct phase of the left ventricular cardiac cycle. (p. 283) A. ____________________________ 1. ____________________________ B. ____________________________ 2. ____________________________ C. ____________________________ 3. ____________________________ D. ____________________________ 4. ____________________________ 5. ____________________________ 18. Fill in the blanks A-N with the correct heart sound, jugular venous pulse waveform, or ECG waveform. Specify the cause of each. (p. 283) A. ____________________________ H. ____________________________ B. ____________________________ I. _____________________________ C. ____________________________ J. _____________________________ D. ____________________________ K. _____________________________ E. ____________________________ L. _____________________________ F. ____________________________ M. _____________________________ G. ____________________________ N. _____________________________ 19. Fill in the blanks A-E with the correct auscultation site. (p. 284) A. ____________________________ D. ______________________________ B. ____________________________ E. ____________________________ C. ____________________________ 20. Name three pathological processes that can cause mitral regurgitation. (p. 285) _______________ ______________________________________________________________________________ 21. Name three pathological processes that can cause aortic regurgitation. (p. 285) _______________ ______________________________________________________________________________ 22. What is meant by pulsus parvus et tardus? (p. 285) ____________________________________ ______________________________________________________________________________ 23. Which murmur is often caused by age-related calcification? (p. 285) ________________________ 24. How is cardiac myocyte physiology different from that in skeletal muscle? (p. 285) _____________ ______________________________________________________________________________ 25. Fill in the blanks (A-E) with the correct phase of the ventricular action potential and the ionic current responsible for each phase. (p. 286) A. ____________________________ D. ______________________________ B. ____________________________ E. ____________________________ C. ____________________________ 26. Fill in the blanks A-C with the correct phase of the pacemaker action potential and the ionic current responsible for each phase. (p. 287) A. ____________________________ C. ______________________________ B. _____________________________ 27. As compared with the myocardial action potential, which phases are absent from the pacemaker potential? (p. 287) _______________________________________________________________ ______________________________________________________________________________
file:///D|/USMLE%20Vids/Workbooks%202012/Chapter%208%20-%20Cardiovascular%202012.txt[8/26/2013 2:13:17 AM]

28. What is the physiologic benefit of atrioventricular delay? (p. 288) ___________________________ ______________________________________________________________________________ 29. What are the most dangerous sequelae of torsades de pointes? (p. 288) _____________________ ______________________________________________________________________________ 30. Patients with Wolff-Parkinson-White syndrome have a higher risk for which type of arrhythmia? (p. 288) ________________________________________________________________________ 31. A 67-year-old man has an irregularly irregular ECG tracing during a routine visit to his doctor. Which drug can decrease his risk of stroke? Which drug can control his heart rate? (pp. 289) ______________________________________________________________________________ 32. The ECG tracing of a 73-year-old woman shows a "sawtooth" pattern. Which three classes of drugs could be given to treat her condition? (pp. 289) ____________________________________ ______________________________________________________________________________ 33. Progressive lengthening of the PR interval take place in _______________ (Mobitz type I/Mobitz type II/both Mobitz type I and type II) heart block. (p. 289) ________________________________ 34. An ECG shows no identifiable waves. What is the most likely diagnosis? (p. 290) ______________ ______________________________________________________________________________ 35. A 65-year-old man presents with an ECG tracing that displays P waves and QRS complexes that occur independently of each other. Which therapeutic intervention would be most appropriate? (p. 290) ________________________________________________________________________ 36. Which infectious disease can cause third-degree heart block? (p. 290) ______________________ 37. What chemical changes of blood elicit a response from peripheral chemoreceptors? How do central chemoreceptors differ? (p. 290) _____________________________________________________ 38. A 25-year-old athlete begins training for the Olympics. As she runs her standard 3 miles, is the increased oxygen demand of the heart met by increased coronary blood flow or by increased extraction of oxygen? (p. 291) ______________________________________________________ 39. In the lungs, what is the physiologic advantage of vasoconstriction in response to hypoxia? (p. 291) ________________________________________________________________________ 40. An 80-year-old man with a history of right-sided heart failure presents with bilateral ankle edema. In terms of capillary fluid exchange, what is the mechanism by which his edema developed? (p. 291) _______________________________________________________________________________ 41. A 55-year-old man with longstanding alcoholic cirrhosis presents with bilateral pedal edema and ascites. In terms of capillary fluid exchange, what is the mechanism by which his edema developed? (p. 291) ______________________________________________________________ 42. A 43-year-old woman presents with bilateral pitting leg edema and laboratory results remarkable for high lowdensity lipoprotein, low albumin, and proteinuria (likely nephrotic syndrome). In terms of capillary fluid exchange, what is the mechanism by which her edema developed? (p. 291) ______________________________________________________________________________ 43. A 50-year-old Ethiopian man presents with severe bilateral leg and scrotal edema due to elephantiasis (lymphatic obstruction caused by filarial nematodes). In terms of capillary fluid exchange, what is the mechanism by which his edema developed? (p. 291) ______________________________________________________________________________ 44. Describe the pressures in the left ventricle and aorta for a patient with aortic stenosis. (p. 291) _____________________________________________________________________________ PATHOLOGY 45. How do neonates with tricuspid atresia remain viable given their severely compromised circulation? (p. 292) ________________________________________________________________________ 46. What are the four clinical features of tetralogy of Fallot? (p. 292) ___________________________ ______________________________________________________________________________ 47. What must be present for a fetus with D-transposition of great vessels to remain viable? (p. 293) ________________________________________________________________________ 48. What physical exam findings are associated with adult-type coarctation of the aorta? (p. 293) ________________________________________________________________________ 49. Describe the murmur of patent ductus arteriosus. (p. 293) ________________________________ ______________________________________________________________________________
file:///D|/USMLE%20Vids/Workbooks%202012/Chapter%208%20-%20Cardiovascular%202012.txt[8/26/2013 2:13:17 AM]

50. Which three cardiac defects are associated with Down syndrome? (p. 294) ___________________ ______________________________________________________________________________ 51. List the five general risk factors for hypertension. (p. 294) _________________________________ ______________________________________________________________________________ 52. An 80-year-old veteran is told by his physician that he has calcification of his radial arteries and that the condition is relatively benign. What disease does he have? (p. 294) _______________ ______________________________________________________________________________ 53. What are the six common complications of atherosclerosis? (p. 295) ________________________ ______________________________________________________________________________ 54. A patient presents to the emergency department with tearing chest pain radiating to the back and dies soon after presentation. What vascular pathology would most likely be seen at autopsy? What would most likely be seen on x-ray of the chest? (p. 295) _________________________________ ______________________________________________________________________________ 55. At what point is ischemic heart disease given the term "myocardial infarction" rather than "unstable angina"? (p. 296) ________________________________________________________________ ______________________________________________________________________________ 56. List the nine common symptoms of myocardial infarction. (p. 297) __________________________ ______________________________________________________________________________ ______________________________________________________________________________ 57. Describe the time frame for events after a myocardial infarction. (p. 297) A. Coagulative necrosis becomes apparent _________________________________________ B. Contraction bands become apparent ____________________________________________ C. Tissue around infarct shows acute inflammation ___________________________________ D. Hyperemia develops _________________________________________________________ E. Granulation tissue appears ____________________________________________________ F. Contracted scar complete _____________________________________________________ 58 After a myocardial infarction, when is the risk for arrhythmia greatest? The risk for free wall rupture or interventricular septal rupture? The risk for ventricular aneurysm? (p. 298) _________________ ______________________________________________________________________________ 59. Six days after having a myocardial infarction, a patient presents with a new-onset murmur. Which type of murmur is the most likely? (p. 298) ____________________________________________ 60. ST-segment elevation on an ECG indicates _______________ (subendocardial/transmural) infarction of the myocardium, while ST-segment depression indicates _______________ (subendocardial/transmural) infarction. (p. 298) 61. Which ECG leads are best for diagnosing an infarct of the LAD? (p. 298) ____________________ ______________________________________________________________________________ 62. A 16-year-old boy presents for a school physical. Physical examination reveals a 2/6 systolic murmur at the left sternal border. Upon questioning, he mentions that he has had several fainting episodes. His father, a former Italian soccer player, had similar episodes and died suddenly at the age of 25 years. What is this patient's most likely diagnosis? What would a cardiac biopsy specimen reveal? (p. 299) _______________________________________________________ ______________________________________________________________________________ 63. In congestive heart failure, _______________ (increased/decreased) cardiac output leads to _______________ (increased decreased) activity of renin-angiotensin-aldosterone, which leads to _______________ (increased/decreased) systemic venous pressure, and ultimately the physical finding of ______________ (peripheral/pulmonary) edema. (p. 300) 64. In congestive heart failure, _______________ (increased/decreased) left ventricular contractility leads to _______________ (increased/decreased) pulmonary venous pressure, ultimately leading to _______________ (peripheral/pulmonary) edema. (p. 300) 65. With respect bacterial endocarditis, what symptoms and signs are represented by the mnemonic FROM JANE? (p. 301) ___________________________________________________________ ______________________________________________________________________________ 66. Rheumatic fever is caused by infection with which organism? (p. 302) _______________________
file:///D|/USMLE%20Vids/Workbooks%202012/Chapter%208%20-%20Cardiovascular%202012.txt[8/26/2013 2:13:17 AM]

67. List the components of the FEVERSS mnemonic for rheumatic heart disease. (p. 302) ______________________________________________________________________________ ______________________________________________________________________________ 68. What six physical findings are commonly associated with cardiac tamponade? (p. 302) ______________________________________________________________________________ ______________________________________________________________________________ 69. A 70-year-old former prostitute presents with chest pain radiating to the back and worsening shortness of breath on exertion. Her cardiac enzymes are negative and she has no ST changes on ECG. An echocardiogram shows aortic regurgitation and a dilated aortic root. Laboratory tests are significant for a positive rapid plasma reagin. What is the most likely cause of her pain and shortness of breath? (p. 302) ______________________________________________________ ______________________________________________________________________________ 70. Which cardiac tumor may present with multiple syncopal episodes? (p. 303) __________________ 71. A 60-year-old woman with varicose veins experiences chest pain and shortness of breath in the hospital after left knee replacement surgery. A CT angiogram shows multiple pulmonary emboli. What is the most likely cause of her pulmonary emboli? (p. 303) ______________________________________________________________________________ 72. Raynaud's disease may be associated with which rheumatologic diseases? (p. 303) ______________________________________________________________________________ 73. List eight signs or symptoms of Wegener's granulomatosis. (p. 304) ________________________ ______________________________________________________________________________ 74. In microscopic polyangiitis, the patient will test positively for __________ (p-ANCA/c-ANCA) in the serum. In Wegener's granulomatosis, the patient will test positively for __________ (p-ANCA/c-ANCA). (p. 304) 75. Patients with Churg-Strauss syndrome usually present with which signs or symptoms? (p. 304) ______________________________________________________________________________ 76. A patient is diagnosed with Sturge-Weber disease. What neurologic manifestation are most likely to be seen? (p. 304) ________________________________________________________________ ______________________________________________________________________________ 77. A 7-year-old boy has a viral urinary tract infection followed by worsening abdominal and joint pain. Purpura develops on his legs. What is the most likely diagnosis? (p. 304) ______________________________________________________________________________ 78. Which four clinical findings are associated with Buerger's disease? (p. 304) __________________ ______________________________________________________________________________ ______________________________________________________________________________ 79. A 7-year-old Japanese child has 1 week of fever, palpable lymph nodes, erythema of the conjunctiva and tongue, and desquamation of the palms of the hands. What is the most likely diagnosis? What is the preferred treatment? (p. 304) ____________________________________ ______________________________________________________________________________ 80. Which infectious disease is strongly associated with polyarteritis nodosa? (p. 304) _____________ ______________________________________________________________________________ 81. Which arteriographic findings are seen with polyarteritis nodosa? (p. 304) ____________________ ______________________________________________________________________________ 82. A 75-year-old woman presents with new-onset right jaw pain and headache at the right temple. What is the most likely diagnosis? (p. 304) ____________________________________________ ______________________________________________________________________________ 83. Temporal arteritis is associated with what laboratory finding? (p. 304) _______________________ ______________________________________________________________________________ 84. What is a strawberry hemangioma? (p. 305) ___________________________________________ 85. What is a cherry hemangioma? (p. 305) ______________________________________________ 86. What is Kaposis sarcoma? (p. 305) __________________________________________________ PHARMACOLOGY 87. Why are ACE inhibitors especially important for patients with diabetes mellitus? (p. 306)
file:///D|/USMLE%20Vids/Workbooks%202012/Chapter%208%20-%20Cardiovascular%202012.txt[8/26/2013 2:13:17 AM]

______________________________________________________________________________ 88. What two agents are first-line therapy for hypertension in pregnancy? (p. 306) ________________ ______________________________________________________________________________ 89. A patient is started on antihypertensive therapy. One week later he returns, complaining of swollen ankles and flushed skin. Which class of medication was he likely prescribed? (p. 306) ________________________________________________________________________ 90. List four adverse effects of nitroglycerin. (p. 306) _______________________________________ ______________________________________________________________________________ 91. What is an adverse effect of diazoxide? (p. 306) ________________________________________ 92. What is the mechanism by which medications can reduce angina? (p. 307) ___________________ ______________________________________________________________________________ 93. A 50-year-old man with hypercholesterolemia is deficient in vitamins A, D, E, and K. He also complains of gastrointestinal discomfort since starting a lipid-lowering agent. Which lipid-lowering agent is the most likely cause? (p. 308) _______________________________________________ 94. A patient who recently started taking lovastatin presents with diffuse muscle pain and weakness. Which laboratory test should be ordered? (p. 308) _____________________________________ ______________________________________________________________________________ 95. Digoxin inhibits which mechanism of transport in the cell membrane? (p. 309) _________________ 96. List six ECG findings characteristic of digoxin toxicity. (p. 309) _____________________________ ______________________________________________________________________________ 97. What are the mechanisms of action of cardiac glycosides? (p. 309) _________________________ ______________________________________________________________________________ 98. Facial rash and joint pain develop in a patient who is taking procainamide for an arrhythmia. Antihistone antibodies are present in her serum. What is the most likely diagnosis? (p. 310) ______________________________________________________________________________ 99. Quinidine causes symptoms of headache and tinnitus, which collectively are known as _______________. (p. 310) 100. Fill in the blanks A-F with the correct subclass of class I antiarrhythmics, and give examples of each subclass. (p. 310) (Adapted, with permission, from Katzung BG, Trevor AJ. Pharmacology: Examination & Board Review, 5th ed. New York: McGraw-Hill, 2011: 284.) A. ____________________________ C. ______________________________ B. ____________________________ D. ____________________________ 101. What are the toxicities of -blockers? (p. 311) __________________________________________ ______________________________________________________________________________ 102. What is the mechanism of action of -blockers? (p. 311) _________________________________ ______________________________________________________________________________ 103. What three types of testing must be performed periodically for patients who take amiodarone? (p. 311) ____________________________________________________________________ ______________________________________________________________________________ 104. What is a potentially fatal adverse effect of ibutilide? (p. 311) ______________________________ 105. What are the adverse effects of calcium channel blockers? (p. 312) _________________________ ______________________________________________________________________________ 106. What do calcium channel blockers work? (p. 312) _______________________________________ ______________________________________________________________________________ 107. Which antiarrhythmic is a first-line drug for diagnosing and abolishing supraventricular tachycardia? (p. 312) _____________________________________________________________________ 108. Which ion is infused to treat torsades de pointes and digoxin toxicity? (p. 312) ________________ ______________________________________________________________________________ 109. Name three toxicities of the antiarrhythmic drug adenosine. (p. 312) ________________________ ______________________________________________________________________________

file:///D|/USMLE%20Vids/Workbooks%202012/Chapter%208%20-%20Cardiovascular%202012.txt[8/26/2013 2:13:17 AM]

Answers ANATOMY 1. Right dominant. 2. The left anterior descending artery. 3. 20%; left dominant. 4. Hoarseness; dysphagia. PHYSIOLOGY 5. Stroke volume. 6. Increase; decrease; increase. 7. Increase. 8. 55%. 9. Arterioles. 10. Hematocrit. 11. Decreased; decreased. 12. Increased; increased. 13. Increased; increased. 14. Stays the same. (Because pressures can equalize across the atrial wall, there is no change in splitting during inspiration.) 15. Elevated jugular venous pressure. 16. Increased; decreased. 17. A = Aortic valve closes; B = aortic valve opens; C = mitral valve closes; D = mitral valve opens; 1 = isovolumetric contraction; 2 = systolic ejection; 3 = isovolumetric relaxation; 4 = rapid filling; 5 = reduced filling. 18. A = S4; atrial kick, caused by high atrial pressures and associated with ventricular hypertrophy and a stiff ventricle. B = S1; mitral and tricuspid valve closure. C = S2; aortic and pulmonary valve closure. D = S3; associated with increased filling pressures, and more common in dilated ventricles. E = a wave; atrial contraction. F = c wave; RV contraction (closed tricuspid valve bulging into right atrium). G = x wave; first down slope of the jugular venous pulse. H = v wave; increased right atrial pressure due to filling against a closed tricuspid valve. I = y wave; second down slope of the jugular venous pulse. J = P wave; atrial depolarization. K = QRS complex; ventricular depolarization. L = QRS complex; ventricular depolarization. M = QRS complex; ventricular depolarization. N = T wave; ventricular repolarization. 19. A = Aortic area; B = left sternal border; C = pulmonic area; D = tricuspid area; E = mitral area. 20. Ischemic heart disease, mitral valve prolapse, and left ventricular dilatation. 21. Aortic root dilatation, bicuspid aortic valve, and rheumatic fever. 22. Pulses are weak, and the strongest part of the peripheral pulse occurs late after the S1 is heard; this is because it takes a long time for blood to cross the stenotic aortic valve to fill the vessels. 23. Aortic stenosis. 24. The cardiac muscle action potential has a plateau due to calcium ion influx; cardiac nodal cells display automaticity by spontaneously depolarizing; and cardiac myocytes are electrically coupled via gap junctions. 25. A = Phase 0; Na+ current. B = Phase 1; K+ current. C = Phase 2; Ca2+ and K+ current. D = Phase 3; K+ current. E = Phase 4; K+ current. 26. A = Phase 4; Na+ current. B = Phase 0; Ca2+ current. C = Phase 3; K+ current. 27. Phases 1 and 2. 28. AV delay allows time for ventricular filling. 29. Ventricular fibrillation and death. 30. Supraventricular tachycardia. 31. Warfarin prevents thromboembolism. -blockers and calcium channel blockers control heart rate. 32. Class IA, IC, and III antiarrhythmics. 33. Type I. Type I involves progressive lengthening followed by a dropped beat. In type II, there are dropped beats without progressive lengthening. 34. Ventricular fibrillation. 35. An implantable pacemaker. 36. Lyme disease.
file:///D|/USMLE%20Vids/Workbooks%202012/Chapter%208%20-%20Cardiovascular%202012.txt[8/26/2013 2:13:17 AM]

37. Low PO2 (<60 mmHg), high PCO2, and low pH of blood; central chemoreceptors are not sensitive to oxygen. 38. Increased coronary blood flow (the heart always operates with maximal oxygen extraction). 39. This mechanism allows for only well-ventilated areas to remain perfused, optimizing gas exchange. 40. Heart failure results in increased capillary pressure, which causes fluid to move out of the capillaries and into the interstitial space. 41. Liver failure results in decreased plasma proteins, which decreases plasma colloid oncotic pressure, and in turn causes fluid to move out of the capillaries and into the interstitial space. 42. Nephrotic syndrome results in proteinuria and subsequent hypoalbuminemia, which decreases plasma colloid oncotic pressure, and in turn causes fluid to move out of the capillaries and into the interstitial space. 43. Lymphatic obstruction causes increased interstitial fluid colloid osmotic pressure, which causes fluid to move out of the capillaries and into the interstitial space. 44. In a patient with aortic stenosis, the pressure in the left ventricle is higher than that in the aorta; the ventricle squeezes blood past a stenotic valve. Thus, the pressure before the valve (the ventricle) is higher than the pressure after the valve (in the aorta). PATHOLOGY 45. To maintain viability, both an ASD and a VSD are required for babies with tricuspid atresia. 46. Pulmonary stenosis, Right ventricular hypertrophy, Overriding aorta, and Ventricular septal defect. (Remember: PROVe) 47. A shunt must be present, which allows adequate mixing of pulmonary and systemic blood (i.e., VSD, ASD, or patent foramen ovale). 48. Notched ribs due to increased collateral circulation, hypertension in the upper extremities, and weak pulses in the lower extremities. 49. Continuous "machine-like" murmur. 50. ASD, VSD, and atrioventricular septal defect. 51. Increased age, obesity, diabetes, smoking, and genetics. 52. Mnckeberg arteriosclerosis. 53. Infarcts, peripheral vascular disease, thrombi, emboli, aneurysms, and ischemia. 54. Longitudinal intraluminal tear forming a false lumen, indicative of aortic dissection. Mediastinal widening is suggestive of aortic dissection. 55. When acute thrombosis due to coronary artery atherosclerosis results in myocyte necrosis. 56. Severe retrosternal pain, nausea, vomiting, pain in the left arm, diaphoresis, jaw pain, shortness of breath, fatigue, and adrenergic symptoms. 57. A = 4 hours; B = 12-24 hours; C = 2-4 days; D = 5-10 days; E = 5-10 days; F = 7 weeks. 58. First 4 days; 5-10 days; 7 weeks. 59. Holosystolic murmur of mitral regurgitation, best heard over the apex of the heart. 60. Transmural infarct; subendocardial infarct. 61. Leads V1-V4. 62. Hypertrophic cardiomyopathy; biopsy shows disoriented, tangled, hypertrophied myocardial fibers. 63. Decreased; increased; increased; peripheral edema. 64. Decreased; increased; pulmonary edema. 65. FROM JANE = Fever, Roth spots, Osler's nodes, Murmur, Janeway lesions, Anemia, Nail-bed hemorrhages, and Emboli. 66. Group A -hemolytic streptococci. 67. FEVERSS = Fever, Erythema marginatum, Valvular damage (vegetation and fibrosis), ESR increase, Red-hot joints (migratory polyarthritis), Subcutaneous nodules (Aschoff bodies), and St. Vitus' dance (chorea). 68. Hypotension, jugular venous distention, distant heart sounds, increased heart rate, and pulsus paradoxus. 69. Ascending aortic aneurysm due to tertiary syphilis. 70. Myxoma; syncope can occur with ball-valve obstruction of the mitral valve. 71. Thromboembolism from stasis in a deep vein of the leg. Despite her varicose veins, thromboembolism from superficial veins is very rare. 72. Mixed connective tissue disease, SLE, and CREST syndrome. 73. Perforation of the nasal septum, chronic sinusitis, otitis media, mastoiditis, cough, dyspnea, hemoptysis, and hematuria. 74. p-ANCA; c-ANCA.
file:///D|/USMLE%20Vids/Workbooks%202012/Chapter%208%20-%20Cardiovascular%202012.txt[8/26/2013 2:13:17 AM]

75. Asthma, skin lesions, sinusitis, and peripheral neuropathy (e.g., wrist/foot drop). 76. Leptomeningeal angiomatosis (intracerebral arteriovenous malformations) and seizures. 77. Henoch-Schnlein purpura. 78. Intermittent claudication, superficial nodular phlebitis, Raynaud's phenomenon, and severe pain in the digits. 79. Kawasaki disease; treat with intravenous immunoglobulin and aspirin. 80. Hepatitis B. 81. Multiple aneurysms and constrictions. 82. Temporal arteritis. 83. Elevated erythrocyte sedimentation rate. 84. A benign capillary hemangioma of infancy, which initially grows and then spontaneously regresses. 85. A benign capillary hemangioma of the elderly; they become more numerous with age. 86. An endothelial malignancy of the skin associated with human herpes virus type 8 and AIDS. PHARMACOLOGY 87. ACE inhibitors can delay progression to diabetic nephropathy. 88. Hydralazine with methyldopa. 89. Calcium channel blockers. 90. Reflex tachycardia, hypotension, flushing, and headache. 91. Hyperglycemia, secondary to a reduction in insulin release. 92. Reduction of myocardial oxygen consumption by decreasing one or more of the determinants of oxygen consumption: end diastolic volume, blood pressure, heart rate, contractility, and ejection time. 93. Bile acid resin. 94. Creatine kinase to test for rhabdomyolysis. 95. Na+/K+/ATPase. 96. Prolonged PR interval, shortened QT interval, scooping, T-wave inversion, arrhythmia, and signs of hyperkalemia. 97. They increase intracellular calcium (thereby acting as a positive inotrope) and stimulate the vagus nerve. 98. Reversible SLE-like syndrome. 99. Cinchonism (which can occur with all quinine derivatives). 100. A = all class I drugs; B = class IA (quinidine procainamide, disopyramide); C = class IB (lidocaine, methilextine, tocainide); D = class IC (flecainide, encainide, propafenone). 101. Impotence, asthma exacerbation, cardiovascular effects (bradycardia, AV block, and CHF), and CNS effects (sedation and sleep alterations). 102. -Blockers decrease cAMP and calcium ion current, and suppress abnormal pacemakers by decreasing the slope of phase 4 of the pacemaker action potential. 103. Pulmonary function, liver function, and thyroid function. 104. Torsades de pointes. 105. Constipation, flushing, edema, and cardiovascular effects (CHF, AV block, sinus node depression). 106. They are used to prevent nodal arrhythmias, such as supraventricular tachycardia. 107. Adenosine. 108. Magnesium. 109. Flushing, hypotension, and chest pain.

file:///D|/USMLE%20Vids/Workbooks%202012/Chapter%208%20-%20Cardiovascular%202012.txt[8/26/2013 2:13:17 AM]

Endocrine

How to Use the Workbook with the Videos

Video Part Questions Endocrine Anatomy 16 Endocrine Physiology part 1 711 Endocrine Physiology part 2 1216 Endocrine Pathology part 1 1725, 49 Endocrine Pathology part 2 2634 Endocrine Pathology part 3 3538 Endocrine Pathology part 4 3944 Endocrine Pharmacology 4549

Questions

ANATOMY 1. What are the three major zones of the adrenal gland? List the major secretory product of each. ________________________________________________________________________

(p. 314)

2. What is the most common adrenal medulla tumor in children? In adults? (p. 314) ______________ ______________________________________________________________________________ 3. What are the two divisions of the pituitary gland? Which hormones does each secrete? (p. 314) ______________________________________________________________________________ 4. What cells produce the hormones released by the posterior pituitary gland? (p. 314) ___________ ______________________________________________________________________________ 5. Name the three major cell types of the Islets of Langerhans. What critical hormone is secreted by each cell? (p. 315)

file:///D|/USMLE%20Vids/Workbooks%202012/Chapter%209%20-%20Endocrine%202012.txt[8/26/2013 2:13:18 AM]

_______________________________________________________________ ______________________________________________________________________________ 6. Which tissues do not need insulin to facilitate glucose uptake? (p. 315) _____________________ ______________________________________________________________________________ PHYSIOLOGY 7. Which two molecules control the secretion of prolactin? Are they stimulatory or inhibitory? (p. 317) ______________________________________________________________________________ 8. Which two anatomical structures play a central role in the regulation of many endocrine functions? (p. 317) ________________________________________________________________________ 9. For each of the congenital bilateral adrenal hyperplasias, indicate whether blood pressure and sex hormone levels are increased or decreased. (p. 318) Disease Blood Pressure Sex Hormones 11-hydroxylase deficiency

17a-hydroxylase deficiency

21-hydroxylase deficiency

10. What enzyme modulates the conversion of testosterone to estradiol? What enzyme modulates the conversion of testosterone to DHT (the more potent form)? (p. 318) ______________________________________________________________________________ 11. How is cortisol produced? (p. 319) ___________________________________________________ 12. PTH _______________ (increases/decreases) serum calcium levels and _______________ (increases/decreases) serum phosphate levels. Vitamin D _______________ (increases/decreases) serum calcium level and _______________ (increases/decreases) serum phosphate levels. (p. 320) 13. Which kidney enzyme acts on vitamin D to activate it? (p. 320) ____________________________ ______________________________________________________________________________ 14. What condition can vitamin D deficiency cause in children? In adults? (p. 321) ________________ ______________________________________________________________________________ 15. Increased levels of sex hormone-binding globulins _______________ (increase/decrease) free testosterone levels in men and _______________ (increase/decrease) free testosterone levels in women. (p. 322) 16. How do T3 and T4 control the bodys metabolic rate? (p. 322) ______________________________ ______________________________________________________________________________ PATHOLOGY 17. What is the most common cause of Cushings syndrome? (p. 323) _________________________ 18. For a patient who is not taking steroids, what are the other potential causes of Cushings syndrome and the likelihood (% of risk) that the patient would have any one of them? (p. 323) ___________ ______________________________________________________________________________ 19. What are the common clinical manifestations of Cushings syndrome? (p. 323) ________________ ______________________________________________________________________________ 20. What is the primary test by which Cushings syndrome is diagnosed? (p. 323) _________________ ______________________________________________________________________________
file:///D|/USMLE%20Vids/Workbooks%202012/Chapter%209%20-%20Endocrine%202012.txt[8/26/2013 2:13:18 AM]

21. Primary hyperaldosteronism is associated with _______________ (increased/decreased) plasma renin activity, while secondary hyperaldosteronism is associated with _______________ (increased/decreased) plasma renin activity. (p. 323) 22. Is hypokalemia seen in primary or secondary adrenal insufficiency? (p. 323) __________________ ______________________________________________________________________________ 23. What are the three common causes of Waterhouse-Friderichsen syndrome? (p. 324) ___________ ______________________________________________________________________________ 24. What is the rule of 10s for pheochromocytoma? What is the cell of origin for this tumor? (p. 324) ______________________________________________________________________________ ______________________________________________________________________________ 25. What are the signs and symptoms of pheochromocytoma? (p. 324) _________________________ ______________________________________________________________________________ 26. Compare and contrast the characteristics of hypothyroidism vs. hyperthyroidism. (p. 325) Sign/Symptom Hypothyroidism Hyperthyroidism Activity level

Bowel movements

Cardiovascular changes

Edema

Free T4

Hair texture

Reflexes

Skin changes

T3 uptake

Temperature
file:///D|/USMLE%20Vids/Workbooks%202012/Chapter%209%20-%20Endocrine%202012.txt[8/26/2013 2:13:18 AM]

Total T4

TSH level

Weight

27. List four diseases characterized primarily by hypothyroidism. (p. 325) _______________________ ______________________________________________________________________________ 28. What is the difference between de Quervains thyroiditis and Riedels thyroiditis? (p. 325) _______ ______________________________________________________________________________ 29. List three diseases characterized primarily by hyperthyroidism. (p. 326) ______________________ ______________________________________________________________________________ 30. What are the four types of thyroid cancer? Which is most common? (p. 326) __________________ ______________________________________________________________________________ 31. Primary hyperparathyroidism is characterized by _______________ (increased/decreased) calcium levels and _______________ (increased/decreased) alkaline phosphatase, while secondary hyperparathyroidism is characterized by _______________ (increase/decreased) calcium levels and _______________ (increased/decreased) alkaline phosphatase. (p. 327) 32. What symptoms does the mnemonic stones, bones, and groans summarize? (p. 327) ______________________________________________________________________________ 33. What is Chvosteks sign? (p. 327) ___________________________________________________ 34. What is Trousseaus sign? (p. 327) __________________________________________________ 35. What are the symptoms of the most common form of pituitary adenoma? (p. 328) ______________________________________________________________________________ 36. How is diabetes insipidus diagnosed? (p. 328) _________________________________________ 37. What are the causes of SIADH? (p. 328) ______________________________________________ 38. What are the differences between DI and SIADH? (p. 328) ________________________________ ______________________________________________________________________________ 39. What are the acute manifestations of diabetes mellitus type 1? (p. 329) ______________________ ______________________________________________________________________________ 40. What are the chronic manifestations of poorly controlled diabetes? (p. 329) ___________________ ______________________________________________________________________________ 41. Compare and contrast the characteristics of type 1 vs type 2 diabetes. (p. 330) Type 1 Type 2 Associated with obesity

file:///D|/USMLE%20Vids/Workbooks%202012/Chapter%209%20-%20Endocrine%202012.txt[8/26/2013 2:13:18 AM]

Genetic predisposition

Glucose intolerance

Ketoacidosis

Need for insulin

Primary defect

Sensitivity to insulin

Typical age of onset

42. How can patients with diabetic ketoacidosis be recognized by their breath? (p. 330) ____________ ______________________________________________________________________________ 43. What are the signs and symptoms of carcinoid syndrome? (p. 331) _________________________ ______________________________________________________________________________ 44. Checkmark which components are involved in the multiple endocrine neoplasias. (p. 332) MEN 1 MEN 2A MEN 2B Pancreas

Parathyroid

Pituitary

file:///D|/USMLE%20Vids/Workbooks%202012/Chapter%209%20-%20Endocrine%202012.txt[8/26/2013 2:13:18 AM]

Pheochromocytoma

PHARMACOLOGY 45. What seven classes of drugs are used to treat diabetes mellitus? (p. 333) ____________________ ______________________________________________________________________________ ______________________________________________________________________________ 46. What is the most feared complication of metformin? (p. 333) ______________________________ 47. Which drugs are used to treat hyperthyroidism? (p. 334) _________________________________ 48. Which drugs are used to treat hypothyroidism? (p. 334) __________________________________ 49. What is the most feared complication of glucocorticoid overdose? (pp 323, 334) _______________ Answers ANATOMY 1. The zona glomerulosa produces aldosterone, the zona fasciculata produces cortisol, and the zona reticularis produces sex hormones. 2. Neuroblastoma; pheochromocytoma. 3. Posterior pituitary: ADH (vasopressin), oxytocin. Anterior pituitary: FSH, LH, ACTH, TSH, prolactin, GH, MSH (melanotropin). 4. Neurons located in the hypothalamus. 5. cells produce insulin. a cells produce glucagon, and d cells produce somatostatin. 6. Brain, RBCs, intestine, cornea, kidney, and liver. PHYSIOLOGY 7. Dopamine is inhibitory; TRH is stimulatory. 8. The hypothalamus and the pituitary 9. Disease Blood Pressure Sex Hormones 11-hydroxylase deficiency . . 17a-hydroxylase deficiency . . 21-hydroxylase deficiency . .

10. Aromatase; 5a-reductase.


file:///D|/USMLE%20Vids/Workbooks%202012/Chapter%209%20-%20Endocrine%202012.txt[8/26/2013 2:13:18 AM]

11. CRH stimulates release of ACTH in pituitary, which prompts coritsol production in the adrenal zona fasciculata. 12. Increases; decreases; increases; increases. 13. 1a-hydroxylase. 14. Rickets; osteomalacia. 15. Decrease; decrease. 16. By increasing Na+/K+-ATPase activity, which increases oxygen consumption, respiratory rate, and body temperature. PATHOLOGY 17. Iatrogenic steroids. 18. Cushings disease (pituitary adenoma): 70%. Ectopic ACTH (e.g., small cell lung cancer): 15%. Adrenal tumor: 15%. 19. Weight gain (truncal obesity), moon facies, buffalo hump, skin thinning, striae, hypertension, hyperglycemia, osteoporosis, amenorrhea, and immunosuppression. 20. Dexamethasone challenge. 21. Decreased; increased. 22. Primary adrenal insufficiency. 23. Septicemia due to N. meningitidis, DIC, and endotoxic shock. 24. 10% are malignant, 10% are bilateral, 10% are extra-adrenal, 10% calcify, 10% occur in kids, and 10% of cases are familial. Chromaffin cells are the cells of origin for pheochromocytoma. 25. Elevated blood Pressure, Pain (headache), Perspiration, Palpitations (tachycardia), and Pallor. These 5 Ps are episodic.

26. Sign/Symptom Hypothyroidism Hyperthyroidism Activity level . . Bowel movements Constipation Diarrhea Cardiovascular changes Bradycardia, dyspnea on exertion Chest pain, palpitations, arrhythmias Edema Myxedema Pretibial myxedema Free T4 .
file:///D|/USMLE%20Vids/Workbooks%202012/Chapter%209%20-%20Endocrine%202012.txt[8/26/2013 2:13:18 AM]

. Hair texture Coarse, brittle Fine Reflexes . . Skin changes Dry, cool skin Warm, most skin T3 uptake . . Temperature Cold intolerance Heat intolerance Total T4 . . TSH level . (if primary) . (if primary) Weight Weight gain (low appetite) Weight loss (high appetite)

27. Hashimotos thyroiditis, cretinism, de Quervains thyroiditis, and Reidels thyroiditis. 28. de Quervains (subacute) thyroiditis is a self-limited type of hypothyroidism that follows a flu-like illness. Riedels thyroiditis involves fibrous replacement of the thyroid. 29. Graves disease, thyroid storm, and toxic multinodular goiter. 30. Papillary thyroid carcinoma (most common), follicular, medullary, and anaplastic. 31. Increased; increased; decreased; increased. 32. Renal calcium stones, osteitis fibrosa cystic of bones, and abdominal complaints (groans due to constipation). 33. In hypoparathyroid patients, tapping of the facial nerve causes the facial muscles to contract. 34. In hypoparathyroid patients, occlusion of the brachial artery causes carpal spasm. 35. Prolactinoma: amenorrhea, galactorrhea, low libido, and infertility. 36. Water deprivation test: urine osmolality doesnt increase in response to water deprivation. Response to desmopressin can distinguish between central and nephrogenic DI. 37. Ectopic ADH (e.g., small cell lung cancer), CNS disorder/head trauma, pulmonary disease, and drugs such as cyclophosphamide. 38. DI denotes lack of ADH; SIADH denotes too much ADH. DI is characterized by intense thirst and polyuria, with high serum osmolarity and low urine concentration. SIADH is characterized by excessive water retention, with high urine osmolarity and low serum osmolarity. 39. Polydipsia, polyuria, polyphagia, weight loss, DKA, and exacerbation of hyperglycemia due to unopposed GH and
file:///D|/USMLE%20Vids/Workbooks%202012/Chapter%209%20-%20Endocrine%202012.txt[8/26/2013 2:13:18 AM]

epinephrine secretion. 40. Retinopathy, glaucoma, cataracts, nephropathy, atherosclerosis, CAD, peripheral vascular occlusive disease, neuropathy, and gangrene. 41. Type 1 Type 2 Associated with obesity No Yes Genetic predisposition Weak Strong Glucose intolerance Severe Mild to moderate Ketoacidosis Common Rare Need for insulin Always Sometimes Primary defect Destruction of cells Increased resistance to insulin Sensitivity to insulin High Low Typical age of onset <30 years >40 years

42. Increased ketogenesis results in accumulation of acetone and other ketones; when exhaled, acetone gives breath a fruity odor. 43. Diarrhea, cutaneous flushing, asthmatic wheezing, and right-sided valvular disease. 44. MEN 1 MEN 2A MEN 2B Pancreas v

file:///D|/USMLE%20Vids/Workbooks%202012/Chapter%209%20-%20Endocrine%202012.txt[8/26/2013 2:13:18 AM]

Parathyroid v v Pituitary v

Pheochromocytoma v v

PHARMACOLOGY 45. Insulin, sulfonylureas, biguanides, glitazones/thiazolidinediones, a-glucosidase inhibitors, amylin mimetics, and GLP-1 analogs. 46. Lactic acidosis. 47. Propylthiouracil and methimazole. 48. Levothyroxine and triiodothyronine. 49. Iatrogenic Cushings syndrome.

file:///D|/USMLE%20Vids/Workbooks%202012/Chapter%209%20-%20Endocrine%202012.txt[8/26/2013 2:13:18 AM]

Gastrointestinal

How to Use the Workbook with the Videos

Video Part Questions Gastrointestinal Anatomy part 1 16 Gastrointestinal Anatomy part 2 710 Gastrointestinal Physiology part 1 1113 Gastrointestinal Physiology part 2 1419 Gastrointestinal Pathology part 1 2026 Gastrointestinal Pathology part 2 2732 Gastrointestinal Pathology part 3 3338 Gastrointestinal Pathology part 4 3942 Gastrointestinal Pathology part 5 4344, 51 Gastrointestinal Pathology part 6 4552 Gastrointestinal Pharmacology 5359

file:///D|/USMLE%20Vids/Workbooks%202012/Chapter%2010%20-%20Gastrointestinal%202012.txt[8/26/2013 2:13:19 AM]

Questions

ANATOMY 1. Which GI ligament is sometimes cut during surgery, and why? (p. 337) ______________________ ______________________________________________________________________________ 2. What are the four layers of the GI tract, from inside to out? (p. 338) ________________________ ______________________________________________________________________________ 3. At what spinal cord level does the celiac trunk branch off the aorta? Where does the aorta bifurcate? Where do the renal arteries branch off? (p. 339) ________________________________ 4. What are the three branches of the celiac trunk? (p. 340) _________________________________ 5. Portal hypertension is characterizes by varices of which three structures? (p. 341) _____________ ______________________________________________________________________________ 6. Which type of hemorrhoid is painful? Why? (p. 342) _____________________________________ 7. Name the major Vascular, neurologic, and lymphatic structures that pass through the femoral triangle in order from lateral to medial: (p. 343) _________________________________________ 8. Direct hernias protrude through the _______________ (abdominal wall/internal inguinal ring), while indirect hernias protrude through the _________________ (abdominal wall/internal inguinal ring). (p. 344) 9. In the image below, identify Hesselbachs triangle. Identify the three structures that define this triangle. For each of the Xs, identify the type of hernia that can occur. (p. 345)

10. How does the course of a direct inguinal hernia differ from that of an indirect inguinal hernia? (p. 345) ________________________________________________________________________ ______________________________________________________________________________ PHYSIOLOGY 11. What are the three major salivary glands? (p. 347) ______________________________________ 12. What are the major functions of saliva? (p. 347) ________________________________________ ______________________________________________________________________________ 13. What cells are responsible for producing gastric acid? What hormones act on them to cause secretion? (p. 348) _______________________________________________________________ 14. Describe the three major steps in carbohydrate digestion. (p. 349) __________________________ ______________________________________________________________________________ 15. Where in the GI tract is iron absorbed? Vitamin B12? Folate? Which require cofactors to facilitate absorption? (p. 349) ______________________________________________________________ ______________________________________________________________________________ 16. Why are Peyers patches important in the immune response? (p. 349) ______________________ ______________________________________________________________________________ 17. What is the composition of bile? (p. 350) ______________________________________________ 18. Direct bilirubin is _______________ (conjugated/unconjugated) with glucuronic acid and is _______________
file:///D|/USMLE%20Vids/Workbooks%202012/Chapter%2010%20-%20Gastrointestinal%202012.txt[8/26/2013 2:13:19 AM]

(soluble/ insoluble) in water. Indirect bilirubin is _______________ (conjugated/ unconjugated) with glucuronic acid and is _______________ (soluble/ insoluble) in water. (p. 350) 19. How is urobilinogen removed from the body? (p. 350) ____________________________________ ______________________________________________________________________________ PATHOLOGY 20. How do the symptoms of achalasia differ from those of esophageal obstruction? (p. 351) ________ ______________________________________________________________________________ 21. What is the characteristic imaging finding in a patient with achalasia? (p. 351) _________________ 22. What is the most common type of esophageal cancer in the United States? Worldwide? Why? (pp. 351-352) ___________________________________________________________________ ______________________________________________________________________________ 23. What are the symptoms of Plummer-Vinson syndrome? (p. 351) ___________________________ ______________________________________________________________________________ 24. Name four symptoms common to all malabsorption syndromes. (p. 353) _____________________ ______________________________________________________________________________ 25. What are three causes of pancreatic insufficiency? What is a major consequence? (p. 353) ______ ______________________________________________________________________________ 26. Celiac sprue is characterized by antibodies to __________ and __________ __________, and is associated with a skin condition called _______________ _______________. (p. 353) 27. What is a risk factor for acute gastritis? For chronic gastritis? (p. 354) _______________________ ______________________________________________________________________________ 28. What are the four major risk factors for stomach cancer? (p. 354) __________________________ ______________________________________________________________________________ 29. What are three common stomach cancer metastases? (p. 354) ____________________________ ______________________________________________________________________________ 30. The pain of gastric ulcers is __________ (increased/decreased) with meals, whereas the pain of duodenal ulcers is __________ (increased/decreased) with meals. (p. 354) 31. What drug combination is most often used as triple therapy for peptic ulcers? (p. 354) __________ ______________________________________________________________________________ 32. Compare and contrast the characteristics of Crohns disease vs. ulcerative colitis. (p. 355) Characteristic Crohns Disease Ulcerative Colitis Associated with colorectal cancer?

Characteristic lesion

Extent of inflammation

Extraintestinal manifestations

Granulomas?

file:///D|/USMLE%20Vids/Workbooks%202012/Chapter%2010%20-%20Gastrointestinal%202012.txt[8/26/2013 2:13:19 AM]

Location of ulcers

Rectal involvement?

33. How is McBurneys point used to diagnose appendicitis? (p. 356) __________________________ ______________________________________________________________________________ 34. An older man presents with subacute onset of left lower quadrant pain. He has a fever. He reports a typical American diet without much fiber. What is the most likely diagnosis? (p. 356) __________ 35. What is the difference between a false diverticulum and a true diverticulum? (p. 356) ___________ ______________________________________________________________________________ 36. _______________ (Intussussception/Volvulus) occurs when a portion of the bowel twists around its mesentery; _______________ (intussusceptions/volvulus) occurs when a part of the one bowel telescopes into a more distal segment. (p. 357) 37. Which intestinal disorders are associated with Down syndrome? Cystic fibrosis? Premature neonates? (p. 358) _______________________________________________________________ 38. What surgical complication can lead to acute bowel obstruction? (p. 358) ____________________ 39. What three features of a colonic polyp are associated with increased risk of malignancy? (p. 358)______________________________________________________________________________ 40. A 55-year-old woman presents with colicky pain. Results of a fecal occult blood test are positive. Colonoscopy reveals literally thousands of polyps in the colon and rectum. What is the most likely diagnosis? What if the patient also has osteosarcoma? What if, instead, she has a glioma? (p. 359) ______________________________________________________________________________ 41. Why do some patients with a carcinoid tumor develop carcinoid syndrome and others dont? (p. 359) ________________________________________________________________________ 42. What are the signs and symptoms of liver disease? (p. 360) _______________________________ ______________________________________________________________________________ 43. In alcoholic hepatitis, the AST level is __________ (greater than/less than) the ALT level; in viral hepatitis, the AST level is __________ (greater than/less than) the ALT level. (p. 360) 44. Match the type of liver disease with its notable characteristic(s) (pp. 361-362) _____ A. a1-Antitripsin deficiency 1. Causes panacinar emphysema _____ B. Alcoholic cirrhosis 2. Common cause is right-sided heart failure _____ C. Alcoholic hepatitis 3. Due to occlusion of IVC or hepatic veins _____ D. Budd-Chiari syndrome 4. Fatty changes in macrovesicles; reversible _____ E. Hepatic steatosis 5. Mallory bodies _____ F. Hepatocellular carcinoma 6. Primary malignant liver tumor in adults _____ G. Nutmeg liver 7. Shrunken liver with irregular surface 45. Compare and contrast the characteristics of the hereditary hyperbilirubinemias. (p. 363) Characteristic Crigler-Najjar Syndrome Dubin-Johnson Syndrome Gilberts Syndrome Impairment

Prognosis
file:///D|/USMLE%20Vids/Workbooks%202012/Chapter%2010%20-%20Gastrointestinal%202012.txt[8/26/2013 2:13:19 AM]

Symptoms

46. What molecule accumulates to cause Wilsons disease? What molecule is not made (tests show low levels) because of this accumulation? (p. 364) ______________________________________ 47. What is the cause of bronze diabetes? (p. 364) _______________________________________ 48. List four extrahepatic causes of biliary obstruction. (pp. 365-366) ___________________________ ______________________________________________________________________________ 49. List two intrahepatic causes of biliary obstruction. (p. 365) ________________________________ 50. Match the term with its definition. (p. 366) _____ A. Cholangitis 1. Gallstones _____ B. Cholecystitis 3. Infection of the biliary tree _____ C. Cholelithiasis 3. Inflammation of the gallbladder 51. What enzymes are elevated in acute pancreatitis? (pp. 360, 366) __________________________ 52. Acute pancreatitis and pancreatic adenocarcinoma share two symptoms: _______________ and _________________________. What two maneuvers can differentiate between these two diseases? (pp. 366-367) ___________________________________________________________ ______________________________________________________________________________ PHARMACOLOGY 53. List the most common histamine blockers for the GI tract. Which histamine receptor do they affect? (p. 368) ________________________________________________________________________ 54. What are the proton pump inhibitors? Why are they such effective drugs? (p. 368) _____________ ______________________________________________________________________________ 55. How is misoprostol most commonly used as a GI agent? (p. 368) __________________________ 56. What is the most dangerous adverse effect of all antacids? (p. 369) _________________________ 57. Infliximab is a monoclonal antibody that targets which molecule? What is a major concern relating to its use? (p. 369) _______________________________________________________________ 58. What powerful medicine is used to control vomiting and nausea after surgery? What receptor does it target? (p. 369) ________________________________________________________________ 59. What drug can be used to treat gastroparesis? What is a worrisome adverse effect? (p. 370) _____ ______________________________________________________________________________

Answers ANATOMY 1. The gastrohepatic ligament may be cut during surgery to provide access to the lesser sac. 2. Mucosa, submucosa, muscularis externa, and serosa/adventitia. 3. T12; L4; L1. 4. Left gastric artery, splenic artery, and common hepatic artery. 5. Esophagus, umbilicus, and rectum. 6. External hemorrhoids are painful because they receive somatic innervation. 7. Femoral nerve, femoral artery, femoral vein, lymphatics (femoral canal). 8. Abdominal wall; internal inguinal ring.
file:///D|/USMLE%20Vids/Workbooks%202012/Chapter%2010%20-%20Gastrointestinal%202012.txt[8/26/2013 2:13:19 AM]

9. 10. A direct hernia goes through Hesselbachs triangle (medial to inferior epigastrics), whereas an indirect hernia passes into the spermatic cord (lateral to inferior epigastrics). PHYSIOLOGY 11. Parotid, submandibular, and sublingual. 12. Saliva begins starch digestion (amylase/ptyalin), lubricates food (mucins), and carries antibacterial secretory products and growth factors that promote epithelial renewal. 13. Parietal cells. Acetylcholine and gastrin contribute, but the most important stimulator of acid secretion is histamine (whose production is stimulated by circulating gastrin). 14. Carbohydrate (salivary amylase) . disaccharides. Carbohydrate (pancreatic amylase) . oligosaccharides, disaccharides. Oligo- and disaccharides (brush border hydrolases) . monosaccharides. 15. Iron is absorbed in the duodenum, vitamin B12 is absorbed in the ileum; and folate is absorbed in the jejunum. Vitamin B12 absorption requires a cofactor (intrinsic factor), whereas iron and folate absorption does not require a cofactor. 16. Peyers patches produce IgA-secreting plasma cells that combat intraluminal antigens. 17. Bile salts, phospholipids, cholesterol, bilirubin, water, and ions. 18. Conjugated; soluble; unconjugated; insoluble. 19. About 80% is excreted as stercobilin in the feces; of the other 20%, about 10% is excreted in the urine as urobilin and about 90% returns to the liver via enterohepatic circulation. PATHOLOGY 20. Esophageal obstruction causes dysphagia with solids, whereas achalasia causes dysphagia with both solids and liquids. 21. A birds beak, or narrowing of the LES at the gastroesophageal junction. 22. Adenocarcinoma; squamous cell carcinoma. GERD is common in the U.S. and GERD can lead to Barretts esophagus, which in turn can lead to adenocarcinoma. 23. Dysphagia, glossitis and iron deficiency anemia. 24. Diarrhea, steatorrhea, weight loss, and weakness. 25. Cystic fibrosis, obstructing cancer, and chronic pancreatitis. A major consequence is deficiency of the fat-soluble vitamins (A, D, E, and K). 26. Gliadin; tissue transglutaminase; dermatitis herpetiformis. 27. Daily NSAID use; Helicobacter pylori infection. 28. Diet high in smoked foods (nitrosamines), achlorhydria, chronic gastritis, and type A blood. 29. Virchows node (involvement of left supraclavicular node), Krukenbergs tumor (metastasis to ovaries), and Sister Mary Josephs nodule (subcutaneous periumbilical metastases). 30. Increased; decreased. 31. A proton pump inhibitor, clarithromycin, and amoxicillin or metronidazole.

32. Characteristic Crohns Disease Ulcerative Colitis Associated with colorectal cancer? Yes Yes Characteristic lesion Skip lesions on cobblestone mucosa Continuous lesion on friable mucosa
file:///D|/USMLE%20Vids/Workbooks%202012/Chapter%2010%20-%20Gastrointestinal%202012.txt[8/26/2013 2:13:19 AM]

Extent of inflammation Transmural Submucosa Extraintestinal manifestations Arthritis Erythema nodosum Immunologic disorders Kidney stones Ankylosing spondylitis Pyoderma gangrenosum Primary sclerosing cholangitis Uveitis Granulomas? Yes No Location of ulcers Any part of GI tract Colon Rectal involvement? No Yes

33. McBurneys point is one-third the distance from the anterior superior iliac spine to the umbilicus on the right side; pain at this point is pathognomonic for appendicitis. 34. This is a typical presentation of diverticulitis, which is also associated with fever and leukocytosis. 35. In a false diverticulum, only the mucosa and submucosa form the pouch; in a true diverticulum, all three gut wall layers form the pouch. 36. Volvulus; intussusception. 37. Hirschsprungs disease and duodenal atresia are associated with Down syndrome. Cystic fibrosis is associated with meconium ileus. Necrotizing enterocolitis is more common in premature neonates. 38. Adhesions. 39. Larger size, villous histology, and epithelial dysplasia. 40. Familial adenomatous polyposis; Gardners syndrome; Turcots syndrome. 41. Carcinoid tumors often produce 5-HT, which causes carcinoid syndrome (wheezing, diarrhea, flushing, right-sided heart murmur). If the tumor is confined to the GI tract, the liver metabolizes the 5-HT and the syndrome doesnt develop. 42. Portal hypertension can cause hematemesis (esophageal varices), melena (peptic ulcer), splenomegaly, caput medusa, ascites, gastropathy, and hemorrhoids. Liver cell failure can cause coma, scleral icterus, fetor hepaticus, spider nevi, gynecomastia, jaundice, testicular atrophy, asterixis, bleeding tendency, anemia, and ankle edema. 43. Greater than; less than. 44. A-1, B-7, C-5, D-3, E-4, F-6, G-2. 45. Characteristic Crigler-Najjar Syndrome Dubin-Johnson Syndrome
file:///D|/USMLE%20Vids/Workbooks%202012/Chapter%2010%20-%20Gastrointestinal%202012.txt[8/26/2013 2:13:19 AM]

Gilberts Syndrome Impairment Bilirubin conjugation Conjugated bilirubin excretion Bilirubin uptake Prognosis Fatal within a few years of diagnosis Benign disease Benign disease Symptoms Jaundice Kernicterus Jaundice Jaundice

46. Copper; ceruloplasmin; basal ganglia. 47. Deposition of hemosiderosin in the skin. 48. Gallstones, biliary strictures, chronic pancreatitis, and carcinoma of the pancreatic head. 49. Primary biliary cirrhosis and primary sclerosing cholangitis. 50. A-3, B-3, C-1. 51. Amylase and lipase. 52. Anorexia and abdominal pain radiating to back. Pancreatic adenocarcinoma is also associated with migratory thrombophlebitis and a positive Courvoisiers sign (obstructive jaundice with a palpable, nontender gallbladder). PHARMACOLOGY 53. Cimetidine, ranitidine, famotidine, nizatidine. These are H2 receptor blockers. 54. Omeprazole, lansoprazole. PPIs are very effective because they act directly on the H+/K+ ATPase, instead of blocking just one of several stimulatory receptors. 55. Preventing NSAID-induced peptic ulcers. 56. Hypokalemia. 57. TNF; reactivation of latent tuberculosis. 58. Ondansetron; 5-HT3. 59. Metoclopramide; parkinsonian effects (because it is a D2 antagonist).

file:///D|/USMLE%20Vids/Workbooks%202012/Chapter%2010%20-%20Gastrointestinal%202012.txt[8/26/2013 2:13:19 AM]

Hematology & Oncology

How to Use the Workbook with the Videos

Video Part Questions Hematology & Oncology Anatomy 110 Hematology & Oncology Physiology 1114 Hematology & Oncology Pathology part 1 1518 Hematology & Oncology Pathology part 2 16, 1922 Hematology & Oncology Pathology part 3 2324 Hematology & Oncology Pathology part 4 2528 Hematology & Oncology Pathology part 5 2830 Hematology & Oncology Pharmacology part 1 3134 Hematology & Oncology Pharmacology part 2 3435

Questions

ANATOMY

file:///D|/USMLE%20Vids/Workbooks%202012/Chapter%2011%20-%20Hematology%20and%20Oncology%202012.txt[8/26/2013 2:13:20 AM]

1. Define the follow terms. (pp. 372-373) A. Anisocytosis _______________________________________________________________ B. Leukocytosis _______________________________________________________________ C. Poikilocytosis ______________________________________________________________ D. Thrombocytopenia __________________________________________________________ 2. What do the dense granules of platelets contain? (p. 372) ________________________________ 3. What do the a-granules of platelets contain? (p. 372) ____________________________________ 4. What is the typical WBC count? (p. 372) ______________________________________________ 5. CD14 is a cell surface marker for which cell type? (p. 373) ________________________________ 6. What five conditions can cause eosinophilia? (p. 373) ___________________________________ ______________________________________________________________________________ 7. List the types of white blood cells in order of decreasing prevalence. (pp. 373-374) _____________ ______________________________________________________________________________ 8. B lymphocytes are produced in the _______________ (bone marrow/thymus) and mature in the _______________ (bone marrow/thymus). T lymphocytes are produced in the _______________ (bone marrow/thymus) and mature in the _______________ (bone marrow/thymus). (pp. 374-375) 9. What molecules are in the granules of mast cells? (p. 374) ________________________________ 10. What type of cell has an off-center nucleus, abundant RER, and a clock-face chromatin distribution? (p. 374) ______________________________________________________________ PHYSIOLOGY 11. Why are Rh-negative mothers given RhoGAM? (p. 375) _________________________________

12. In the chart below, checkmark which coagulation factors are in the intrinsic vs. extrinsic coagulation pathways. (p. 376) Factor Extrinsic Pathway Intrinsic Pathway Both Pathways I

II

VII

VIII
file:///D|/USMLE%20Vids/Workbooks%202012/Chapter%2011%20-%20Hematology%20and%20Oncology%202012.txt[8/26/2013 2:13:20 AM]

IX

XI

XII

13. What factors are vitamin K dependent? (p. 376) ________________________________________ 14. Describe the four steps of primary hemostasis and platelet plug formation. (p. 377) ____________ ______________________________________________________________________________ PATHOLOGY 15. Identify each cell type and its associated pathology. (pp. 378-379) A. B. C. D. E. F. A. _______________________________ D. ______________________________ B. _______________________________ E. ______________________________ C. _______________________________ F. ______________________________ 16. Indicate whether the lab findings in the chart are elevated, decreased, or normal. (pp. 380-385) Lab Value Anemia of Chronic Disease Hemochroma-tosis Iron Deficiency Pregnancy/ OCPs Thalassemia Ferritin

Serum iron
file:///D|/USMLE%20Vids/Workbooks%202012/Chapter%2011%20-%20Hematology%20and%20Oncology%202012.txt[8/26/2013 2:13:20 AM]

Transferrin

% Transferrin

17. In a-thalassemia, what is the condition called when all four a-globin genes are deleted? When three are deleted? When one or two are deleted? (p. 380) ________________________________ ______________________________________________________________________________ 18. What are the major history and physical exam findings of lead poisoning? (p. 381) _____________ ______________________________________________________________________________ 19. What clinical findings differentiate megaloblastic anemia due to folate deficiency versus that due to vitamin B12 deficiency? (p. 382) _____________________________________________________ 20. What are four causes of aplastic anemia? (p. 383) ______________________________________ ______________________________________________________________________________ 21. Match the intrinsic hemolytic normocytic anemia with its characteristic. (p. 384) _____ A. G6PD deficiency 1. GLU . LYS mutation _____ B. HbC defect 2. GLU . VAL mutation _____ C. Hereditary spherocytosis 3. Heinz bodies _____ D. Paroxysmal nocturnal hemoglobinuria 4. Howell-Jolly bodies _____ E. Pyruvate kinase deficiency 5. Increased hemosiderin in urine _____ F. Sickle cell anemia 6. Rigid RBCs 22. Which autoimmune hemolytic anemias are associated with warm agglutinins? With cold agglutinins? (p. 385) ______________________________________________________________ ______________________________________________________________________________ 23. What are the 5 Ps of acute intermittent porphyria? (p. 386) ______________________________ ______________________________________________________________________________ 24. Indicate whether the lab findings in the chart are elevated, decreased, or normal. (pp. 387-388) Disorder Platelet Count Bleeding Time PT PPT DIC

file:///D|/USMLE%20Vids/Workbooks%202012/Chapter%2011%20-%20Hematology%20and%20Oncology%202012.txt[8/26/2013 2:13:20 AM]

Glanzmanns thromboasthenia

Hemophilia

ITP/TTP

Vitamin K deficiency

von Willebrands disease

25. What type of cell is shown in the image? In what condition is this cell seen? (p. 389) ___________________________________ ___________________________________ ___________________________________ 26. Which type of Hodgkins lymphoma is more common in women? (p. 389) ____________________ 27. What will serum protein electrophoresis reveal in a patient with multiple myeloma? What will be seen in the urine? (p. 391) _________________________________________________________ 28. Match the disease with the genetic translocation most closely associated with it. (pp. 390-393) _____ A. Burkitts lymphoma 1. t(8;14) _____ B. Chronic myelogenous leukemia 2. t(9;22) _____ C. Ewings sarcoma 3. t(11;14) _____ D. Follicular lymphoma 4. t(11;22) _____ E. M3 type of AML 5. t(14;18) _____ F. Mantle cell lymphoma 6. t(15;17) 29. What are the four major groups of leukemias? Which type is associated with Auer rods? (pp. 392-393) __________________________________________________________________
file:///D|/USMLE%20Vids/Workbooks%202012/Chapter%2011%20-%20Hematology%20and%20Oncology%202012.txt[8/26/2013 2:13:20 AM]

30. Indicate whether the lab findings in the chart are elevated, decreased, or normal. (p. 394) Chronic Myeloproliferative Disorder Platelets RBCs WBCs CML

Essential thrombocytosis

Myelofibrosis

Polycythemia vera

PHARMACOLOGY 31. Indicate how the drugs in the chart affect the lab findings. (pp. 395-397) Drug Bleeding Time Platelet Count PT PTT Aspirin

Clopidogrel/abciximab

Heparin

Warfarin
file:///D|/USMLE%20Vids/Workbooks%202012/Chapter%2011%20-%20Hematology%20and%20Oncology%202012.txt[8/26/2013 2:13:20 AM]

32. What is the mechanism of action of heparin? How is overdose treated? (p. 396) ______________ ______________________________________________________________________________ 33. What is the mechanism of action of warfarin? How is overdose treated? (p. 396) ______________ ______________________________________________________________________________ 34. Match the drug with its target. (pp. 396-402) _____ A. Abciximab 1. ADP receptor (platelets) _____ B. Clopidogrel 2. B cells (CD20) _____ C. Etoposide 3. bcr-abl tyrosine kinase _____ D. 5-Fluorouracil 4. Estrogen receptor _____ E. Imatinib 5. Glycoprotein IIb/IIIa receptor _____ F. Rituximab 6. HER2 (erb-B2) _____ G. Tamoxifen 7. Plasminogen _____ H. tPA 8. Thymidylate synthase _____ I. Trastuzumab 9. Topoisomerase II _____ J. Vincristine 10. Tubulin 35. Match the patient with the drug he or she is most likely taking. (pp. 399-400) _____ A. Patient preparing for bone marrow transplantation has 1. Bleomycin PFTs consistent with restrictive lung disease 2. Busulfan _____ B. Patient with colon cancer has myelosuppression 3. Cyclophosphamide not reversible with leucovorin 4. Doxorubicin _____ C. Patient with non-Hodgkins lymphoma is having 5. 5-Fluorouracil hemorrhagic cystitis 6. Methotrexate _____ D. Patient with leukemia is having myelosuppression reversible with leucovorin _____ E. Patient with solid tumor is having dilated cardiomyopathy _____ F. Patient with testicular cancer has PFTs consistent with restrictive lung disease Answers ANATOMY 1. A. Anisocytosis: Cells vary in size. B. Leukocytosis: Increased number of leukocytes. C. Poikilocytosis: Cells vary in shape. D. Thrombocytopenia: Decreased number of platelets. 2. ADP and calcium. 3. Von Willebrand Factor and fibrinogen. 4. 4,000-10,000/mm3. 5. Macrophages. 6. Neoplasm, asthma, allergic processes, collagen vascular diseases, and parasites. 7. Neutrophils, lymphocytes, monocytes, eosinophils, and basophils. 8. Bone marrow; bone marrow; bone marrow; thymus. 9. Histamine, heparin, and eosinophil chemotactic factors. 10. Plasma cell. PHYSIOLOGY 11. To prevent the development of anti-Rh IgG, which can cause erythroblastosis fetalis in a subsequent fetus.
file:///D|/USMLE%20Vids/Workbooks%202012/Chapter%2011%20-%20Hematology%20and%20Oncology%202012.txt[8/26/2013 2:13:20 AM]

12. Factor Extrinsic Pathway Intrinsic Pathway Both Pathways I v II v V v VII v

VIII v IX v X v XI v XII v

13. Factors II, VII, IX, X, and proteins C and S.


file:///D|/USMLE%20Vids/Workbooks%202012/Chapter%2011%20-%20Hematology%20and%20Oncology%202012.txt[8/26/2013 2:13:20 AM]

14. Injury: Endothelial damage prompts vWF binding to exposed collagen. Adhesion: Platelets bind vWF and release ADP/Ca2+. Activation: GpIIb/IIIa is expressed. Aggregation: Fibrinogen binds GpIIb/IIIa and links platelets. PATHOLOGY 15. A = Bite cell: G6PD deficiency. B = Ringed sideroblast: sideroblastic anemia. C = Schistocyte: microangiopathic hemolytic anemia (DIC, TTP, HUS). D = Teardrop cell: bone marrow infiltration or myelofibrosis. E = Target cell: HbC disease, asplenia, liver disease, thalassemia. F = Heinz bodies: a-thalassemia, G6PD deficiency. 16. Lab Value Anemia of Chronic Disease Hemochroma-tosis Iron Deficiency Pregnancy/ OCPs Thalassemia Ferritin . . . normal Normal Serum iron . . . normal Normal Transferrin . . . . Normal % Transferrin normal to . . . . Normal 17. Four deletions: Hb Barts; three deletions: HbH disease; one or two deletions: asymptomatic. 18. History reveals abdominal colic and confusion. Physical exam shows wrist and foot drop and Burtons lines on gingiva. 19. Megaloblastic anemia due to vitamin B12 deficiency is associated with neurologic symptoms. 20. Idiopathic, radiation and drug exposure, viral infection, and Fanconis anemia. 21. A-3, B-1, C-4, D-5, E-6, F-2. 22. Warm agglutinins (IgG): SLE, chronic lymphocytic leukemia, and certain drug reactions. Cold agglutinins (IgM): chronic lymphocytic leukemia, Mycoplasma pneumoniae infection, and infectious mononucleosis. 23. Painful abdomen, Pink urine, Polyneuropathy, Psychological disturbances, and Precipitated by drugs (alcohol, barbiturates).
file:///D|/USMLE%20Vids/Workbooks%202012/Chapter%2011%20-%20Hematology%20and%20Oncology%202012.txt[8/26/2013 2:13:20 AM]

24. Disorder Platelet Count Bleeding Time PT PPT DIC . . . . Glanzmanns thromboasthenia normal . normal Normal Hemophilia normal normal normal . ITP/TTP . . normal Normal Vitamin K deficiency normal normal . . von Willebrands disease normal . normal .

25. Reed Sternberg cell; Hodgkins lymphoma. 26. Nodular sclerosing. 27. An M-spike, representing a monoclonal antibody; IgG light chains (Bence-Jones proteins). 28. A-1, B-2, C-4, D-5, E-6, F-3. 29. AML (associated with Auer rods), ALL, CML, and CLL. 30. Chronic Myeloproliferative Disorder Platelets RBCs
file:///D|/USMLE%20Vids/Workbooks%202012/Chapter%2011%20-%20Hematology%20and%20Oncology%202012.txt[8/26/2013 2:13:20 AM]

WBCs CML . . . Essential thrombocytosis . Myelofibrosis variable . variable Polycythemia vera . . .

PHARMACOLOGY 31. Drug Bleeding Time Platelet Count PT PTT Aspirin . normal normal normal Clopidogrel/abciximab . normal normal normal Heparin normal normal . . Warfarin normal normal .
file:///D|/USMLE%20Vids/Workbooks%202012/Chapter%2011%20-%20Hematology%20and%20Oncology%202012.txt[8/26/2013 2:13:20 AM]

32. Heparin activates antithrombin, decreasing thrombin and Xa. Treat heparin overdose with protamine sulfate. 33. Warfarin interferes with normal synthesis and .-carboxylation of vitamin K-dependent clotting factors. Treat warfarin overdose with intravenous vitamin K and fresh frozen plasma. 34. A-5, B-1, C-9, D-8, E-3, F-2, G-4, H-7, I-6, J-10. 35. A-2, B-5, C-3, D-6, E-4, F-1.

file:///D|/USMLE%20Vids/Workbooks%202012/Chapter%2011%20-%20Hematology%20and%20Oncology%202012.txt[8/26/2013 2:13:20 AM]

Musculoskeletal

How to Use the Workbook with the Videos

Video Part Questions Musculoskeletal Anatomy & Physiology part 1 12 Musculoskeletal Anatomy & Physiology part 2 35 Musculoskeletal Anatomy & Physiology part 3 6 Musculoskeletal Anatomy & Physiology part 4 711 Musculoskeletal Pathology part 1 1213 Musculoskeletal Pathology part 2 12, 1416 Musculoskeletal Pathology part 3 1722 Musculoskeletal Pathology part 4 2326 Musculoskeletal Pathology part 5 2728 Musculoskeletal Pathology part 6pending 2830 Musculoskeletal Pharmacology 3136

file:///D|/USMLE%20Vids/Workbooks%202012/Chapter%2012%20-%20Musculoskeletal%20and%20Connective%20Tissue%202012.txt[8/26/2013 2:13:22 AM]

Questions ANATOMY AND PHYSIOLOGY 1. Identify the structures on the image below. (p. 404)

2. In a case of possible knee injury, abnormal passive abduction indicates a torn __________ (ACL/MCL), and a positive anterior drawer sign indicates a torn __________ (ACL/MCL) (p. 405) 3. A 36-year-old man presents with difficulty abducting his left arm. His left shoulder appears flattened and asymmetric to his right arm. What sensory deficit is most likely? Which nerve is likely involved? (p. 407) _______________________________________________________________ ______________________________________________________________________________ 4. A 24-year-old woman was in a car accident and suffered a fracture of the midshaft of her humerus. When asked to hold up her arm, her wrist could not be extended and appeared floppy. What sensory deficits is she likely experiencing? Which nerve is most likely affected? (p. 407) ______________________________________________________________________________ 5. A 20-year-old man presents with a fracture of his medial epicondyle. When asked to flex his wrist, his hand is radially deviated. Why is this deviation happening? (p. 407) ______________________ ______________________________________________________________________________ 6. In the chart below, describe the characteristics of brachial plexus lesions. (pp. 408-410) Lesion Injured Nerve(s) Presentation Affected Muscle(s) Ape hand

Erb-Duchenne palsy

file:///D|/USMLE%20Vids/Workbooks%202012/Chapter%2012%20-%20Musculoskeletal%20and%20Connective%20Tissue%202012.txt[8/26/2013 2:13:22 AM]

Klumpkes palsy

Median claw

Ulnar claw

7. A 30-year-old man comes to the physician after being tackled in a football game with his friends. He says he was tackled below the knee. The patient is using a steppage gait. What is the most likely diagnosis, and what sensory deficits are likely? (p. 411) _____________________________ ______________________________________________________________________________ 8. A 42-year-old man presents to the clinic with difficulty adducting his thigh. He had been skiing the previous week. He mentions that his thigh is hurting and that hes not sure what he did to it. What type of injury is most likely? (p. 411) _________________________________________________ ______________________________________________________________________________ 9. A 22-year-old woman has difficulty climbing stairs. What type of mechanical injury would predispose her to this problem? (p. 411) ______________________________________________ ______________________________________________________________________________ 10. A 22-year-old man is brought to the emergency department after a motor vehicle accident. His blood alcohol level is 0.20. He had not been wearing a seatbelt. Physical examination reveals he has difficulty flexing his thigh and substantial difficulty extending his leg. What injury is most likely? (p. 411) ________________________________________________________________________ 11. A 23-year-old woman who was a passenger in a motor vehicle accident is brought to the emergency department. She had been wearing a seat belt. Physical examination reveals trauma to the lateral aspect of the knee. What motor deficit is most likely? (p. 411) ____________________ ______________________________________________________________________________ PATHOLOGY 12. Indicate whether the lab findings in the chart below are elevated, decreased, or normal. (pp. 414-415) Condition Alkaline Phosphatase Phosphate PTH Serum Ca2+ Osteitis fibrosa cystica

Osteomalacia

file:///D|/USMLE%20Vids/Workbooks%202012/Chapter%2012%20-%20Musculoskeletal%20and%20Connective%20Tissue%202012.txt[8/26/2013 2:13:22 AM]

Osteopetrosis

Osteoporosis

Pagets disease

13. What are the symptoms of a vertebral crush fracture? (p. 414) _____________________________ 14. In the image below, identify the type of tumor according to its location in the bone. (p. 416) 15. A 64-year-old man with no significant medical history has had increasing back pain and right hip pain for the past decade. The pain is worse at the end of the day. Physical examination shows enlargement of the distal interphalangeal joints. What is the most likely diagnosis? (p. 417) ______ ______________________________________________________________________________ 16. A 36-year old woman presents to the clinic with a new complaint of fatigue of several months duration. She also reports stiffness in both hands in the morning, which decreases after showering. Physical examination reveals a lowgrade fever, and subcutaneous nodules are palpated along her forearms bilaterally. What type of hypersensitivity reaction is occurring? (p. 418) ______________________________________________________________________________ 17. What are the three symptoms of Sjgrens syndrome? What are the two common autoantibodies found in these patients? (p. 418) ____________________________________________________ ______________________________________________________________________________ 18. A 50-year-old obese man comes to the emergency room at 3 a.m. because of a painful great toe. The pain began 5 hours earlier, after he walked home from a bar where he had steak and beer. He is allergic to NSAIDs. What is the most appropriate treatment? (p. 419) ______________________ 19. In the chart below, compare and contrast gout vs. pseudogout. (p. 419) Gout Pseudogout Sexual predilection

Joint most often affected

file:///D|/USMLE%20Vids/Workbooks%202012/Chapter%2012%20-%20Musculoskeletal%20and%20Connective%20Tissue%202012.txt[8/26/2013 2:13:22 AM]

Crystal composition

Crystal shape

Birefringence

Treatment

20. List three causes of septic arthritis. (p. 420) ____________________________________________ _______________________________________________________________________________ 21. A 27-year-old man presents with a 6-month history of low back pain and stiffness that awakens him during the night and is worse in the morning. He has tenderness over his sacroiliac joints bilaterally and decreased motion of his lumbar spine. What is the most likely diagnosis? (p. 420) _________ ______________________________________________________________________________ 22. A 31-year-old man comes to the emergency room because his eyes have been red and itchy for the past 8 hours. For the past month, he has experienced pain during urination and diffuse joint pain; however, 3 weeks earlier he tested negatively for gonorrhea and chlamydia. Tests are also negative for rheumatoid factor. What is the most likely diagnosis? (p. 420) ___________________ 23. A 17-year-old girl complains of fever and a painful swollen left elbow. In addition, she has had pain in her right knee for the past several days. Her cheeks are slightly red but not tender. Her VDRL test result is positive. She is shocked to learn that she has syphilis because she has no sexual history. What is the most likely explanation for this finding? (p. 421) _________________________ ______________________________________________________________________________ 24. In Lambert-Eaton syndrome, symptoms __________ (improve/worsen) with muscle use. In myasthenia gravis, symptoms __________ (improve/worsen) with muscle use. (p. 422) 25. Acetylcholinesterase inhibition __________ (does/does not) reverse symptoms in Lambert-Eaton syndrome and __________ (does/does not) reverse symptoms in myasthenia gravis. (p. 422) 26. What does CREST stand for? (p. 423) ________________________________________________

27. Match the dermatologic term with its definition. (p. 423) _____ A. Blister containing pus 1. Acantholysis _____ B. Dried exudates from vesicle, bulla, or pustule 2. Acanthosis _____ C. Elevated skin lesion <1 cm 3. Bulla _____ D. Epidermal hyperplasia 4. Crust _____ E. Flat discoloration <1 cm 5. Dermatitis
file:///D|/USMLE%20Vids/Workbooks%202012/Chapter%2012%20-%20Musculoskeletal%20and%20Connective%20Tissue%202012.txt[8/26/2013 2:13:22 AM]

_____ F. Inflammation of skin 6. Keloid _____ G. Irregular, raised lesion after scar 7. Macule _____ H. Large, fluid-filled blister 8. Papule _____ I. Macule >1 cm 9. Patch _____ J. Papule >1 cm 10. Plaque _____ K. Separation of epidermal cells 11. Pustule _____ L. Small, fluid-filled blister 12. Vesicle _____ M. Transient vesicle 13. Wheal 28. Match the skin disorder with its defining characteristic(s). (pp. 423-427) _____ A. Associated with hyperinsulinemia 1. Acanthosis nigricans _____ B. Auspitz sign 2. Actinic keratosis _____ C. Target lesion 3. Albinism _____ D. Decrease in melanocytes 4. Bullous pemphigoid _____ E. Genital warts 5. Cellulitis _____ F. Honey-colored crusts 6. Condylomata acuminata _____ G. Horn cysts 7. Erythema multiforme _____ H. Infection of dermis and subcutaneous tissues 8. Freckle _____ I. Infection of stratum granulosum 9. Hives _____ J. Intensely pruritic wheals 10. Impetigo _____ K. Nikolskys sign negative 11. Pemphigus vulgaris _____ L. Nikolskys sign positive 12. Psoriasis _____ M. Normal melanocyte number, . melanin 13. Seborrheic keratosis _____ N. Normal melanocyte number, . melanin 14. Staphylococcal scalded skin _____ O. Premalignant lesion 15. Vitiligo 29. In the chart below, compare and contrast the characteristics of bullous pemphigoid vs. pemphigus vulgaris. (p. 426) Characteristic Bullous Pemphigoid Pemphigus Vulgaris Pattern of immunofluorescence

Location of blisters

Oral involvement

Nikolskys sign

30. Actinic keratosis is a precursor to _____________ (melanoma/squamous cell carcinoma), while dysplastic nevus is a precursor to _____________ (melanoma/squamous cell carcinoma). (p. 428) PHARMACOLOGY 31. In the arachidonic acid pathway (p. 429):
file:///D|/USMLE%20Vids/Workbooks%202012/Chapter%2012%20-%20Musculoskeletal%20and%20Connective%20Tissue%202012.txt[8/26/2013 2:13:22 AM]

A. Phospholipase A2 facilitates the conversion of __________ into _______________________. B. Lipoxygenase facilitates the conversion of __________ into __________________________. C. Cyclooxygenase facilitates the conversion of __________ into ________________________. 32. What is the mechanism of action of aspirin? (p. 429) ____________________________________ 33. What is the mechanism of action of NSAIDs? (p. 430) ___________________________________ 34. Why should a person who takes NSAIDs consider switching to a COX-2 inhibitor? What is the risk of COX-2 inhibitors? (p. 430) _______________________________________________________ ______________________________________________________________________________ 35. What is the mechanism of action of acetaminophen? (p. 430) _____________________________ 36. Which TNF-a inhibitor can be used to treat Crohns disease? (p. 432) _______________________ Answers ANATOMY AND PHYSIOLOGY 1. 2. MCL; ACL. 3. Axillary nerve damage leads to loss of sensation over the shoulder. 4. Radial nerve damage leads to loss of sensation over the posterior arm, dorsal hand, and thumb. 5. This patient likely injured his ulnar nerve. Therefore, he has lost function of the flexors on the ulnar portion of his wrist, and has retained the flexors innervated by the median nerve, which are on the radial aspect of the hand. Hence, when these flexors flex, they cause radial deviation. 6. Lesion Injured Nerve(s) Presentation Affected Muscle(s) Ape hand Proximal median nerve Unopposable thumb Opponens pollicis Erb-Duchenne palsy Upper trunk, C5-6 nerve roots Limb hangs at side, medially rotated; forearm pronated Abductors, lateral rotators, and biceps are paralyzed Klumpkes palsy Lower trunk; C8-T1 nerve roots MCP extended; DIP/PIP flexed All lumbricals Median claw Distal median nerve Second/third fingers clawed Lateral lumbricals Ulnar claw Distal ulnar nerve Popes blessing when asked to extend fingers Medial lumbricals

file:///D|/USMLE%20Vids/Workbooks%202012/Chapter%2012%20-%20Musculoskeletal%20and%20Connective%20Tissue%202012.txt[8/26/2013 2:13:22 AM]

7. With injury to the common peroneal nerve, sensory deficit would occur at the anterolateral leg and dorsal aspect of the foot. 8. Anterior hip dislocation causing damage to the obturator nerve. 9. Posterior hip dislocation causing injury to the inferior gluteal nerve. 10. Pelvic fracture causing damage to the femoral nerve. 11. Difficulty with foot inversion and plantar/toe flexion due to damage to the tibial nerve. PATHOLOGY 12. Condition Alkaline Phosphatase Phosphate PTH Serum Ca2+ Osteitis fibrosa cystica . . . . Osteomalacia normal . . . Osteopetrosis normal normal Normal Normal Osteoporosis normal normal Normal Normal Pagets disease . normal Normal Normal

13. Acute back pain, loss of height, and kyphosis. 14. 15. Osteoarthritis.
file:///D|/USMLE%20Vids/Workbooks%202012/Chapter%2012%20-%20Musculoskeletal%20and%20Connective%20Tissue%202012.txt[8/26/2013 2:13:22 AM]

16. The patient has rheumatoid arthritis, which is associated with a type III hypersensitivity (immune complex) reaction. 17. Xerophthalmia, xerostomia, and arthritis. SS-A (Ro) and SS-B (La) 18. This patient has gout; treat with colchicine if NSAIDs are contraindicated. 19. Gout Pseudogout Sexual predilection Men None Joint most often affected MTP joint of big toe Knee Crystal composition Monosodium urate Calcium pyrophosphate Crystal shape Needle Rhomboid Birefringence Negative Weakly positive Treatment NSAIDs; if NSAIDs contraindicated, colchicines None

20. Staphylococcus aureus, Streptococcus, and Neisseria gonorrhoeae. 21. Ankylosing spondylitis. 22. Reiters syndrome. 23. She has lupus, the great imitator! VDRL results are false-positive due to cross-reaction between antiphospholipid antibodies and the cardiolipin used in the VDRL. 24. Improve; worsen. 25. Does not; does. 26. Calcinosis, Raynauds phenomenon, Esophageal dysmotility, Sclerodactyly, and Telangiectasia. 27. A-11, B-4, C-8, D-2, E-7, F-5, G-6, H-3, I-9, J-10, K-1, L-12, M-13. 28. A-1, B-12, C-7, D-15, E-6, F-10, G-13, H-5, I-14, J-9, K-4, L-11, M-3, N-8, O-2. 29. Characteristic Bullous Pemphigoid Pemphigus Vulgaris Pattern of immunofluorescence Linear Reticular or lacelike

file:///D|/USMLE%20Vids/Workbooks%202012/Chapter%2012%20-%20Musculoskeletal%20and%20Connective%20Tissue%202012.txt[8/26/2013 2:13:22 AM]

Location of blisters Subepidermal Intraepidermal Oral involvement No Yes Nikolskys sign Negative Positive

30. Squamous cell carcinoma; melanoma. PHARMACOLOGY 31. A = Phospholipase A2 facilitates the conversion of membrane lipids into arachidonic acid. B = Lipoxygenase facilitates the conversion of arachidonic acid into hydroperoxides (which then get converted into leukotrienes). C = Cyclooxygenase facilitates the conversion of arachidonic acid into endoperoxides (which then get converted into prostacyclin, prostaglandins, and thromboxane). 32. Aspirin causes irreversible inhibition of cyclooxygenase. 33. NSAIDs cause reversible inhibition of cyclooxygenase. 34. Many people who take NSAIDs suffer from gastrointestinal distress and ulcer formation, which can be avoided by using COX-2 inhibitors. COX-2 inhibitors do, however, increase the risk of thrombosis. 35. Acetaminophen causes reversible inhibition of cyclooxygenase. 36. Infliximab.

file:///D|/USMLE%20Vids/Workbooks%202012/Chapter%2012%20-%20Musculoskeletal%20and%20Connective%20Tissue%202012.txt[8/26/2013 2:13:22 AM]

Neurology

How to Use the Workbook with the Videos

Video Part Questions Neurology Anatomy & Physiology part 1 14 Neurology Anatomy & Physiology part 2 511 Neurology Anatomy & Physiology part 3 1215 Neurology Anatomy & Physiology part 4 14, 1618 Neurology Anatomy & Physiology part 5 1921 Neurology Anatomy & Physiology part 6 2225 Neurology Anatomy & Physiology part 7 2634 Neurology Anatomy & Physiology part 8 3536 Neurology Anatomy & Physiology part 9 3738 Neurology Pathology part 1 39 Neurology Pathology part 2 40 Neurology Pathology part 3 4143 Neurology Pathology part 4 42 Neurology Pharmacology part 1 44

file:///D|/USMLE%20Vids/Workbooks%202012/Chapter%2013%20=%20Neurology%202012.txt[8/26/2013 2:13:23 AM]

Neurology Pharmacology part 2 44 Neurology Pharmacology part 3 4546 Neurology Pharmacology part 4 47

Questions

ANATOMY AND PHYSIOLOGY 1. Match the cell type with its characteristic. (pp. 434-435) _____ A. Astrocytes 1. Destroyed in Gullain-Barr syndrome _____ B. Microglia 2. Destroyed in multiple sclerosis _____ C. Oligodendroglia 3. Form multinucleated giant cells _____ D. Schwann cells 4. Maintain blood-brain barrier 2. ____________ (Oligodendroglia / Schwann cells) are destroyed in Gullain-Barr syndrome; ____________ (Oligodendroglia / Schwann cells) are destroyed in multiple sclerosis. (pp. 434-435) 3. Match the disease with its alteration in neurotransmitter. (p. 435) _____ A. Alzheimers disease 1. Decrease in acetylcholine _____ B. Anxiety 2. Decrease in dopamine _____ C. Depression 3. Increase in dopamine _____ D. Schizophrenia 4. Increase in norepinephrine 4. Which substances cross the blood-brain barrier quickly? Which substances cross it slowly? (p. 436) ______________________________________________________________________________ 5. Match the area of the hypothalamus with its function. (p. 436) _____ A. Anterior hypothalamus 1. Circadian rhythm _____ B. Lateral area 2. Cooling _____ C. Paraventricular nucleus 3. Heating _____ D. Posterior hypothalamus 4. Hunger _____ E. Suprachiasmatic nucleus 5. Makes ADH _____ F. Supraoptic nucleus 6. Makes oxytocin _____ G. Ventromedial area 7. Satiety 6. Ascending sensory information from the body reaches the __________ (VPL / VPM) of the thalamus, while sensory information from the face reaches the ___________ (VPL / VPM). (p. 437) 7. The striatum consists of the __________ and the ____________, while the lentiform nucleus refers to the ____________ and the _________________. (p. 438) 8. How does loss of dopamine in Parkinsons disease affect the excitatory pathway? How does it affect the inhibitory pathway? (p. 438) ________________________________________________ ______________________________________________________________________________ ______________________________________________________________________________ 9. Which nucleus is affected in Parkinsons disease? In hemiballismus? In Huntingtons disease? (pp. 438-439) ___________________________________________________________________ 10. What are the cardinal features of Parkinsons disease? (p. 438) ____________________________ ______________________________________________________________________________ ______________________________________________________________________________
file:///D|/USMLE%20Vids/Workbooks%202012/Chapter%2013%20=%20Neurology%202012.txt[8/26/2013 2:13:23 AM]

11. Define the following terms. (p. 439) A. Athetosis __________________________________________________________________ B. Chorea ___________________________________________________________________ C. Dystonia __________________________________________________________________ D. Myoclonus ________________________________________________________________ 12. Match the area of a brain lesion with its clinical effect(s). (p. 441) _____ A. Anterograde amnesia 1. Amygdala _____ B. Contralateral hemiballismus 2. Basal ganglia _____ C. Deficits in concentration, orientation and judgment 3. Cerebellar hemisphere _____ D. Eyes look away from side of lesion 4. Cerebellar vermis _____ E. Eyes look toward side of lesion 5. Frontal eye fields _____ F. Intention tremor and limb ataxia 6. Frontal lobe _____ G. Klver-Bucy syndrome 7. Hippocampus _____ H. Reduced levels of arousal and wakefulness 8. Mamillary bodies _____ I. Spatial neglect syndrome 9. PPRF _____ J. Tremor at rest, chorea, or athetosis 10. Midbrain _____ K. Truncal ataxia and dysarthria 11. Right parietal lobe _____ L. Wernicke-Korsakoff syndrome 12. Subthalamic nucleus 13. Fluent aphasia with impaired comprehension describes _______________ (Brocas/Wernickes) aphasia, while nonfluent aphasia with intact comprehension describes _______________ (Brocas/Wernickes) aphasia. (p. 442) 14. What is the major vascular territory covered by the ACA? MCA? PCA? (pp. 443-444) 15. Label the Circle of Willis in the image below. (p. 443) 16. Lesions in PICA or AICA may result in vomiting, vertigo, nystagmus and ipsilateral Horners syndrome. The two can be distinguished because _________ lesions cause loss of pain and temperature in the face and limbs, while _________ lesions cause paralysis of the face and pain/temperature loss in the face only. The ________ lesions cause decreased corneal reflex, while _______ lesions cause loss of gag reflex, as well as dysphagia and hoarseness. (p. 444)
17. On CT, an epidural hematoma _______________ (does/does not) cross suture lines, while a subdural hematoma _______________ (does/does not) cross suture lines. (p. 446) 18. A 50-year-old woman presents to the emergency department with a headache and numbness on the right side. Her speech is difficult to understand, and her mouth droops when talking. Noncontrast CT of the head shows bright areas. Should tPA be administered? (p. 447) _________ ______________________________________________________________________________ 19. What are the symptoms of normal pressure hydrocephalus? (p. 448) ________________________ ______________________________________________________________________________ 20. Which spinal nerves exit the intervertebral foramina above the corresponding vertebra? Which spinal nerves exit the foramina below the corresponding vertebra? (p. 448) ______________________________________________________________________________ 21. Where is a lumbar puncture usually performed? (p. 448) _________________________________ 22. Compare/contrast the characteristics of upper vs. lower motor neuron lesions. (p. 450) Characteristic UMN Lesion LMN Lesion Atrophy

Babinski reflex

Clasp knife spasticity

file:///D|/USMLE%20Vids/Workbooks%202012/Chapter%2013%20=%20Neurology%202012.txt[8/26/2013 2:13:23 AM]

Fasciculation

Reflexes

Spastic paralysis

Tone

Weakness

23. For each lesion in the image, identify the motor deficit and associated diseases. (pp. 451-452) A. ____________________________________________________________________________ B. ____________________________________________________________________________ C. ____________________________________________________________________________ D. ____________________________________________________________________________ E. ____________________________________________________________________________ F. ____________________________________________________________________________ G. ____________________________________________________________________________ 24. Horners syndrome is associated with which three lesions? (p. 453) ________________________ ______________________________________________________________________________ 25. Match these commonly tested reflexes to their nerve roots: (p. 454) _____ A. Biceps 1. L4 _____ B. Patella 2. C5 _____ C. Triceps 3. S1 _____ D. Achilles 4. C7 26. Fill in the chart below describing the cranial nerves. (p. 456)
file:///D|/USMLE%20Vids/Workbooks%202012/Chapter%2013%20=%20Neurology%202012.txt[8/26/2013 2:13:23 AM]

CN Name Function Type (Sensory/ Motor/Both) Location in Brainstem I Olfactory

II Optic

III Oculomotor

IV Trochlear

V Trigeminal

VI Abducens

VII Facial

VIII Vestibulocochlear

file:///D|/USMLE%20Vids/Workbooks%202012/Chapter%2013%20=%20Neurology%202012.txt[8/26/2013 2:13:23 AM]

IX Glossopharyngeal

X Vagus

XI Accessory

XII Hypoglossal

27. Which CNs mediate the pupillary response? (p. 456) ____________________________________ 28. If there is a lesion in CNs V and VII, which reflex is impaired? (p. 456) _______________________ 29. Which CNs mediate the gag reflex? (p. 456) ___________________________________________ 30. With a lesion in CN X, the uvula deviates _______________ (toward/away from) the side of the lesion. (p. 458) 31. With a lesion in CN XI, the head turns ________ (toward/away from) the side of the lesion. (p. 458) 32. With a lesion in CN XII, the tongue deviates _______________ (toward/away from) the side of the lesion. (p. 458)
33. What structures pass through the superior orbital fissure? (p. 458) __________________________ 34. What structures pass through the cavernous sinus? (p. 458) ______________________________ 35. Where is the obstruction in open/wide angle glaucoma? Where is the obstruction in closed/narrow angle glaucoma? Which one is painful? (p. 461) ________________________________________ ______________________________________________________________________________ 36. Referring to the image, which cranial nerve and muscle are tested with each movement? (p. 462) Line A-B: _______________________________________________________________ Line C-D: _______________________________________________________________ Line E-F: _______________________________________________________________

37. Identify the type of visual field defects in the image below. (p. 463)
file:///D|/USMLE%20Vids/Workbooks%202012/Chapter%2013%20=%20Neurology%202012.txt[8/26/2013 2:13:23 AM]

38. Horizontal diplopia develops in a 26-year-old woman with multiple sclerosis. Examination reveals she cannot adduct her left eye past midline and has a left-beating nystagmus in her right eye when looking to the right. However, her left eye can adduct during convergence. Where is the lesion most likely located? (p. 464) ____________________________________________________________ PATHOLOGY 39. Match the type of dementia with its defining characteristic. (p. 465) _____ A. Alzheimers disease 1. a-Synuclein defect _____ B. Creutzfeld-Jakob disease 2. Neurofibrillary tangles _____ C. Frontotemporal dementia 3. Prions _____ D. Lewy body dementia 4. Tau protein 40. For each case, identify the type of seizure and its first-line treatment. (p. 466) A. A teenage boy suddenly stiffens, falls down, and experiences rhythmic jerking of his extremities lasting 1 minute. ___________________________________________________ B. A 7-year-old boy is referred to his primary physician for behavioral problems at school. He spaces out during class. EEG shows a 3-Hz spike-and-wave pattern. __________________ __________________________________________________________________________ C. A 1-year-old girl is brought to the emergency department because she had been shaking and unresponsive for 30 seconds. Her rectal temperature is 104F. ________________________ D. A 45-year-old man who suffered a concussion from a car accident has episodes of jerky movements of his left arm that he cannot control. He remembers the incident itself, but had blacked out afterward. ________________________________________________________

41. For each case, identify the most likely neurocutaneous disorder. (p. 467) A. A 6-month-old presents with her first seizure. Woods lamp examination shows several areas of hypopigmentation over her trunk and extremities. ________________________________ B. A 6-month-old has a port wine stain over his left eye and cheek, extending to the tip of his nose, with a sharp drop-off to normal-toned skin on the right side of his face. _____________ C. A 6-month-old has congestive heart failure. Imaging shows a cavernous hemangioma in the liver. ______________________________________________________________________ D. A 6-month-old has multiple hyperpigmented brown macules scattered over the trunk and upper extremities. ___________________________________________________________ 42. For each case, identify the most likely brain tumor. (p. 469-470) A. A 49-year-old man presents with a 2-month history of morning headaches. CT of the head shows a heterogeneousappearing mass with irregular borders crossing the corpus callosum. __________________________________________________________________________ B. A 40-year-old woman develops a small, well-circumscribed nodular-appearing lesion on her right frontal lobe. It appears to be attached to the skull. ______________________________ C. A 4-year-old boy presents with a 1-month history of morning headaches, abnormal gait, and dysmetria. Imaging shows a pilocytic appearance in the posterior fossa. _________________ D. A 7-year-old girl presents with bitemporal hemianopia. ______________________________ 43. On MRI, _______________ (glioblastoma multiforme/meningioma/metastasis) has heterogeneous enhancement, _______________ (glioblastoma multiforme/meningioma/metastasis) has uniform enhancement, and _______________ (glioblastoma multiforme/meningioma/metastasis) has ring-enhancing lesions. (p. 469) PHARMACOLOGY 44. Match the drug with its indication for use. (pp 471-475) _____ A. Acute status epilepticus 1. Diazepam _____ B. Absence seizures 2. Ethosuximide _____ C. Chronic pain 3. Methadone _____ D. Closed/narrow angle glaucoma 4. Phenobarbital
file:///D|/USMLE%20Vids/Workbooks%202012/Chapter%2013%20=%20Neurology%202012.txt[8/26/2013 2:13:23 AM]

_____ E. Induction of anesthesia 5. Pilocarpine _____ F. Insomnia 6. Thiopental _____ G. Opiate dependency 7. Tramadol _____ H. Seizure prophylaxis in pregnancy 8. Zolpidem 45. Anesthetics with low blood and lipid solubility have __________ (fast/low) induction, while anesthetics with high blood and lipid solubility have __________ (fast/low) induction. (p. 475) 46. What are the two clinical uses of dantrolene? (p. 477) ___________________________________ ______________________________________________________________________________ 47. Describe the mechanism of action for each drug commonly used to treat Parkinsons disease. (p. 478) A. Benztropine ________________________________________________________________ B. Bromocriptine ______________________________________________________________ C. L-dopa/carbidopa ___________________________________________________________ D. Selegiline __________________________________________________________________ Answers ANATOMY AND PHYSIOLOGY 1. A-4, B-3, C-2, D-1. 2. Schwann cells; oligodendroglia. 3. A-1, B-4, C-2 D-3. 4. Nonpolar/lipid-soluble substances cross rapidly (via diffusion); glucose and amino acids cross slowly (by carriermediated transport). 5. A-2, B-4, C-6, D-3, E-1, F-5, G-7. 6. Body-VPL, Face-VPM 7. Striatum = putamen + caudate; Lentiform = putamen + globus pallidus 8. Loss of dopamine inhibits the excitatory pathway and disinhibits (or excites) the inhibitory pathway. 9. Substantia nigra; caudate nucleus; subthalamic nucleus. 10. Tremor at rest, cogwheel Rigidity, Akinesia, and Postural instability. 11. A. Athetosis: Slow, writhing movements. B. Chorea: Sudden, jerky, purposeless movements. C. Dystonis: Sustained, involuntary muscle contractions. D. Myoclonus: Sudden, brief muscle contractions. 12. A-7, B-12, C-6, D-9, E-5, F-3, G-1, H-10, I-11, J-2, K-4, L-8. 13. Wernickes; Brocas. 14. The ACA supplies the medial surface of the brain, which covers the leg area of the motor and sensory cortices. The MCA supplies the motor and sensory cortex of the face and arm. The PCA supplies the occipital cortex. 15. 16. PICA; AICA; AICA; PICA 17. Does not; does. 18. This patient has had a stroke, and the bright areas on noncontrast CT indicate hemorrhage. Thus tPA should not be administered. 19. Ataxia, dementia, and urinary incontinence. 20. Nerves C1-C7 exit above the corresponding vertebra; the others exit below the corresponding vertebrae. 21. In the L3-L4 or L4-L5 interspace. 22. Characteristic
file:///D|/USMLE%20Vids/Workbooks%202012/Chapter%2013%20=%20Neurology%202012.txt[8/26/2013 2:13:23 AM]

UMN Lesion LMN Lesion Atrophy + Babinski reflex + Clasp knife spasticity + Fasciculation + Reflexes . . Spastic paralysis + Tone . . Weakness + +

23. A. Lower motor neuron lesions only, attributable to destruction of anterior horns; poliomyelitis and WerdingHoffmann disease. B. Random and asymmetric lesions due to demyelination; multiple sclerosis. C. Combination of upper and lower motor neuron deficits with no sensory deficit; amyotrophic lateral sclerosis. D. Complete occlusion of anterior spinal artery with sparing of dorsal columns and tract of Lissauer. E. Degeneration of dorsal roots and dorsal columns; tabes dorsalis. F. Damage to the crossing fibers of the corticospinal tract; syringomyelia. G. Demyelination of dorsal columns, lateral corticospinal tracts, and spinocerebellar tracts; vitamin B12 neuropathy, vitamin E deficiency, and Friedrichs ataxia. 24. Pancoasts tumor, Brown-Squard syndrome, and late-stage syringomyelia. 25. A-2, B-1, C-4, D-3. 26. CN Name

file:///D|/USMLE%20Vids/Workbooks%202012/Chapter%2013%20=%20Neurology%202012.txt[8/26/2013 2:13:23 AM]

Function Type (Sensory/ Motor/Both) Location in Brainstem I Olfactory Smell Sensory In cerebrum II Optic Sight Sensory Midbrain III Oculomotor Extraocular eye movements Pupillary constriction Eyelid opening Motor Midbrain IV Trochlear Extraocular eye movements Motor Midbrain V Trigeminal Facial sensation Muscles of mastication Both Pons VI Abducens Extraocular eye movements Motor Pons VII Facial Muscles of facial expression Taste from anterior two thirds of tongue Lacrimation, salivation Eyelid closing Stapedius muscle in ear Both Pons VIII
file:///D|/USMLE%20Vids/Workbooks%202012/Chapter%2013%20=%20Neurology%202012.txt[8/26/2013 2:13:23 AM]

Vestibulocochlear Hearing, balance Sensory Pons IX Glossopharyngeal Taste from posterior third of tongue Swallowing, palate elevation Salivation Chemo- and baroreceptors of the carotid body and sinus Both Medulla X Vagus Taste from epiglottic region Swallowing, palate elevation Midline uvula Muscles of pharynx and larynx Parasympathetics to visceral organs Aortic arch chemo- and baroreceptors Both Medulla XI Accessory Head turning Shoulder shrugging Motor Medulla XII Hypoglossal Tongue movement Motor Medulla

27. CNs II and III mediate the papillary response. 28. The corneal reflex is impaired if there is a lesion in CNs V and VII. 29. CNs IX and X mediate the gag reflex. 30. The uvula deviates away from the side of the CN X lesion. 31. There is weakness turning the head away from the CN XI lesion. 32. The tongue deviates toward the side of the CN XII lesion. 33. CN III, CN IV, CN V1, CN VI, ophthalmic vein, sympathetic fibers 34. CN III, CN IV, CN V1, CN V2, CN VI, sympathetic fibers, internal carotid artery 35. Open/wide angle glaucoma is due to obstructed outflow. Closed/narrow angle glaucoma is due to obstructed flow between iris and lens. Closed/narrow angle glaucoma is painful. 36. Line A-B (SR-SO): assesses CN III all but the superior oblique and the lateral rectus. Line C-D (IO-IR): assesses CN IV the superior oblique. Line E-F (LR-MR): assesses CN VI the lateral rectus.
file:///D|/USMLE%20Vids/Workbooks%202012/Chapter%2013%20=%20Neurology%202012.txt[8/26/2013 2:13:23 AM]

37. 38. The reason her left eye can adduct during convergence but not during right lateral gaze is because the oculomotor nerve itself works perfectly, but the connection between the abducens nuclei and the oculomotor nuclei is impeded. The message to look right does not reach the left medial rectus, causing the right eye to beat leftward because of the dysconjugate image. (If you are still confused, now would be an excellent time to review INO.) PATHOLOGY 39. A-2, B-3, C-4, D-1. 40. A. Generalized tonic-clonic (grand mal) seizures. Treat with phenytoin, carbamazepine, or valproic acid. B. Absence seizures. Treat with ethosuximide. C. Febrile seizures. Do not use anti-epileptics. Give acetaminophen for the fever. If it appears to be a simple febrile seizure (1 seizure/illness; no previous febrile seizures), recurrence of seizure is not likely. D. Simple partial seizures with secondary generalization. Virtually any anti-epileptic drug can be used; the most common are phenytoin, carbamazepine, levetiracetam, and valproic acid. 41. A. Tuberous sclerosis. B. Sturge-Weber syndrome. C. Cavernous hemangiomas can occur in isolation, but are associated with von Hippel-Lindau disease. D. Neurofibromatosis type 1. 42. A. Glioblastoma multiforme. B. Meningioma. C. Medulloblastoma. D. Craniopharyngioma. 43. Glioblastoma multiforme; meningioma; metastasis. PHARMACOLOGY 44. A-1, B-2, C-7, D-5, E-6, F-8, G-3, H-4. 45. Fast; slow. 46. Malignant hyperthermia and neuroleptic malignant syndrome. 47. A. Inhibits acetylcholine, which preferentially excites the inhibitory pathway over the excitatory pathway. The net effect of acetylcholine is inhibition of the thalamus. B. Dopamine receptor agonist. C. Inhibits peripheral breakdown of L-dopa, which crosses the blood-brain barrier and is converted to dopamine in the brain. The net effect is an increase of dopamine in the brain milieu. D. Inhibits MAO-B enzyme, resulting in less breakdown of dopamine in the CNS.

file:///D|/USMLE%20Vids/Workbooks%202012/Chapter%2013%20=%20Neurology%202012.txt[8/26/2013 2:13:23 AM]

Psychiatry

How to Use the Workbook with the Videos

Video Part Questions Psychiatry Psychology 12 Psychiatry Pathology part 1 38 Psychiatry Pathology part 2 8 Psychiatry Pathology part 3 812 Psychiatry Pathology part 4 1315, 27 Psychiatry Pharmacology part 1 1624, 27 Psychiatry Pharmacology part 2 22, 2427

file:///D|/USMLE%20Vids/Workbooks%202012/Chapter%2014%20-%20Psychiatry%202012.txt[8/26/2013 2:13:24 AM]

Questions

PSYCHOLOGY 1. Match the term with its definition. (p. 482) _____ A. Application of aversive stimulus extinguishes 1. Classical conditioning unwanted behavior 2. Countertransference _____ B. Discontinuation of reinforcement eliminates 3. Negative reinforcement behavior 4. Extinction _____ C. Patient projects feelings about someone 5. Positive reinforcement onto physician 6. Punishment _____ D. Physician projects feelings about someone 7. Transference onto patient _____ E. Removal of aversive stimulus produces desired action _____ F. Reward produces desired action _____ G. Response is elicited by a learned stimulus presented in conjunction with an unconditioned stimulus 2. In the chart below, identify the defense mechanism and label it as mature or immature. Behavior Defense Mechanism Mature or Immature A football player jokes about playing the defending state champions the following week (pp. 482-483)

A survivor of an earthquake describes the event and her towns destruction with no emotional response

A man wants to cheat on his wife, but instead buys flowers for her

A man throws a temper tantrum because he does not get a promotion at work

A child throws a temper tantrum when he cannot have candy

A heroin addict decides to volunteer at an anti.drug-abuse program at a local school

file:///D|/USMLE%20Vids/Workbooks%202012/Chapter%2014%20-%20Psychiatry%202012.txt[8/26/2013 2:13:24 AM]

A man who is angry at his wife thinks she is angry at him

A man who is angry at his wife becomes angry at his son

PATHOLOGY 3. What are the signs and symptoms of physical child abuse? (p. 484) ________________________ ______________________________________________________________________________ 4. What are the signs and symptoms of child neglect? (p. 484) ______________________________ ______________________________________________________________________________ 5. In the chart below, compare and contrast Aspergers syndrome vs. autism. (p. 485) Characteristic Aspergers Syndrome Autism Difficulty socializing

Gender predilection

Intelligence

Language impairment

Repetitive behaviors

6. What are the most common causes of a loss of orientation? (p. 485) _______________________ ______________________________________________________________________________ 7. In the chart below, compare and contrast delirium vs. dementia. (p. 486) Characteristic Delirium Dementia Caused by anticholinergic drugs

file:///D|/USMLE%20Vids/Workbooks%202012/Chapter%2014%20-%20Psychiatry%202012.txt[8/26/2013 2:13:24 AM]

Changing level of consciousness

EEG

Memory loss

Onset

Relationship to age

Reversible

8. Fill in the chart comparing the following disorders. (pp. 486-490) Disorder Criteria No. of Criteria Needed to Diagnose Criteria Must be Present How Long? Major depressive episode

Manic episode

Panic disorder

Schizophrenia

file:///D|/USMLE%20Vids/Workbooks%202012/Chapter%2014%20-%20Psychiatry%202012.txt[8/26/2013 2:13:24 AM]

9. In one word, patients with cluster A personality disorder can be described as _______________. Those with cluster B personality disorder can be described as ______________. Those with cluster C personality disorder can be described as _______________. (pp. 492-493) 10. Identify the type of personality disorder. (Numbers may be used more than once.) (pp. 492-493) _____ A. Antisocial personality disorder 1. Cluster A _____ B. Avoidant personality disorder 2. Cluster B _____ C. Borderline personality disorder 3. Cluster C _____ D. Dependent personality disorder _____ E. Histrionic personality disorder _____ F. Narcissistic personality disorder _____ G. Obsessive-compulsive disorder _____ H. Paranoid personality disorder _____ I. Schizoid personality disorder _____ J. Schizotypal personality disorder 11. _______________ is a common defense mechanism used by patients who have borderline personality disorder. They also frequently show signs of ________ on their hands and arms. (p. 492) 12. ______________ is a common defense mechanism used by patients who have paranoid personality disorder. (p. 492) 13. __________fractures are often seen in anorexic patients. (p. 493) 14. What is Russells sign? (p. 493) _____________________________________________________ 15. Cocaine intoxication is characterized by _______________ (constricted/dilated) pupils, while opioid intoxication is characterized by _______________ (constricted/dilated) pupils. (p. 495) PHARMACOLOGY 16. List four treatment options for bipolar disorder. (p. 497) __________________________________ ______________________________________________________________________________ 17. List three treatment options for panic disorder. (p. 497) _________________________________ ______________________________________________________________________________ 18. List two treatment options for obsessive-compulsive personality disorder. (p. 497) _____________ ______________________________________________________________________________ 19. A 24-year-old man began treatment for schizophrenia 1 month ago. Today he presents with feelings of restlessness. What type of medication is he most likely using? (p. 498) _____________ ______________________________________________________________________________ 20. What are the four extrapyramidal symptoms of typical antipsychotics, and at what timepoints do they normally occur? Which extrapyramidal symptom is not reversible? (p. 498) _____________ ______________________________________________________________________________ 21. Which adverse effects of both olanzapine and clozapine do not occur with other atypical antipsychotics? Which other adverse effect of clozapine does not occur with other atypical antipsychotics? (p. 498) ___________________________________________________________ 22. A patient presents to the emergency department with muscle rigidity and fever. He has an extensive psychiatric history, but his specific diagnosis and prescription history are not immediately available. Which symptoms are important in determining whether the patient is suffering from serotonin syndrome or neuroleptic malignant syndrome? (pp. 498 and 500) __________________ ______________________________________________________________________________ 23. What are the classic adverse effects of lithium? (p. 499) __________________________________ ______________________________________________________________________________ 24. List three treatment options for generalized anxiety disorder. Which would you give to a truck driver and why? (pp. 499-500) ___________________________________________________________ _______________________________________________________________________________ 25. Which drugs block reuptake of both serotonin and norepinephrine? Which agents block the reuptake of serotonin exclusively? Which agents block reuptake of norepinephrine exclusively? (pp. 500-501) ___________________________________________________________________
file:///D|/USMLE%20Vids/Workbooks%202012/Chapter%2014%20-%20Psychiatry%202012.txt[8/26/2013 2:13:24 AM]

______________________________________________________________________________ 26. Why is nortriptyline a better choice than amitriptyline for an elderly patient with depression? _______________________________________________________________________ 27. Match the medication with its most common usage. (pp. 495-501) _____ A. Bupropion 1. ADHD _____ B. Buspirone 2. Alcoholism _____ C. Clozapine 3. Bedwetting _____ D. Cyproheptadine 4. Bulimia _____ E. Dantrolene 5. Depression with insomnia _____ F. Disulfiram 6. Diabetic peripheral neuropathy _____ G. Duloxetine 7. Generalized anxiety disorder _____ H. Haloperidol 8. Heroin detoxification _____ I. Imipramine 9. Hypochondriasis _____ J. Lithium 10. Insomnia _____ K. Methadone 11. Mood stabilizer in bipolar disorder _____ L. Methylphenidate 12. Neuroleptic malignant syndrome _____ M. Mirtazapine 13. Schizophrenia (both positive and negative symptoms) _____ N. Phenelzine 14. Schizophrenia (primarily positive symptoms) _____ O. Sertraline 15. Serotonin syndrome _____ P. Thiamine 16. Smoking cessation _____ Q. Trazodone 17. Wernicke-Korsakoff syndrome Answers

(p. 500)

PSYCHOLOGY 1. A-6, B-4, C-7, D-2, E-3, F-5, G-1. 2. Behavior Defense Mechanism Mature or Immature A football player jokes about playing the defending state champions the following week Humor Mature A survivor of an earthquake describes the event and her towns destruction with no emotional response Isolation of affect Immature A man wants to cheat on his wife, but instead buys flowers for her Sublimation Mature A man throws a temper tantrum because he does not get a promotion at work Regression Immature A child throws a temper tantrum when he cannot have candy Acting out Immature A heroin addict decides to volunteer at an anti.drug-abuse program at a local school
file:///D|/USMLE%20Vids/Workbooks%202012/Chapter%2014%20-%20Psychiatry%202012.txt[8/26/2013 2:13:24 AM]

Altruism Mature A man who is angry at his wife thinks she is angry at him Projection Immature A man who is angry at his wife becomes angry at his son Displacement Immature PATHOLOGY 3. Healed fractures on x-ray, cigarette burns, multiple bruises, subdural hematomas, and retinal hemorrhage or detachment. 4. Poor hygiene, malnutrition, withdrawal, impaired social/emotional development, and failure to thrive. 5. Characteristic Aspergers Syndrome Autism Difficulty socializing Yes, milder Yes, severe Gender predilection Boys None Intelligence Normal Decreased Language impairment No Yes Repetitive behaviors Yes Yes

6. Alcohol, drugs, fluid/electrolyte imbalance, head trauma, hypoglycemia, and nutritional deficiency. 7. Characteristic Delirium Dementia Caused by anticholinergic drugs Yes No Changing level of consciousness
file:///D|/USMLE%20Vids/Workbooks%202012/Chapter%2014%20-%20Psychiatry%202012.txt[8/26/2013 2:13:24 AM]

Yes No EEG Abnormal Normal Memory loss Possible Yes Onset Sudden Gradual Relationship to age None More common in older adults Reversible Yes in most cases No

8. Disorder Criteria No. of Criteria Needed to Diagnose Criteria Must be Present How Long? Major depressive episode Sleep disturbance, loss of Interest, Guilt, Energy loss, Concentration loss, Appetite change, Psychomotor changes, Suicidal ideation, depressed mood (remember: SIG E CAPS) 5 out of 9 At least 2 weeks Manic episode Distractibility, Irresponsibility, Grandiosity, Flight of ideas, increase in goal-directed Activity, decreased need for Sleep, Talkativeness (remember: DIG FAST) 3 out of 7 At least 1 week Panic disorder Palpitations, Paresthesias, Abdominal distress, Nausea, Intense fear of dying, lIghtheadedness, Chest pain, Chills, Choking, disConnectedness, Sweating, Shaking, Shortness of breath (remember: PANICS) 4 out of 13 None Schizophrenia Delusions, hallucinations (esp. auditory), disorganized speech, disorganized behavior, any type of negative symptom (e.g., flat affect, social withdrawal)
file:///D|/USMLE%20Vids/Workbooks%202012/Chapter%2014%20-%20Psychiatry%202012.txt[8/26/2013 2:13:24 AM]

2 out of 5 More than 6 months

9. Weird; wild; worried. 10. A-2, B-3, C-2, D-3, E-2, F-2, G-3, H-1, I-1, J-1. 11. Splitting; self-mutilation. 12. Projection. 13. Metatarsal 14. Dorsal hand calluses due to inducing vomiting; a sign of bulimia nervosa. 15. Dilated; constricted. PHARMACOLOGY 16. Lithium, carbamazepine, valproate, and atypical antipsychotics. 17. SSRIs, TCAs, and benzodiazepines. 18. SSRIs and clomipramine. 19. He is experiencing an extrapyramidal symptom of typical antipsychotics, namely akathisia. 20. Acute dystonia (4 hours), akinesia (4 days), akathisia (4 wk), tardive dyskinesia (4 mo). Tardive dyskinesia is generally not reversible. 21. Weight gain can occur with both olanzapine and clozapine. Clozapine also can cause agranulocytosis. 22. With neuroleptic malignant syndrome, one would expect to see autonomic instability and myoglobinuria. Common symptoms of serotonin syndrome are flushing, diarrhea, and hypotension. 23. Lithium adverse effects include Movement (tremor), Nephrogenic diabetes insipidus, hypotension, and problems in Pregnancy (e.g., Ebstein anomaly). (Remember LMNOP.) 24. Benzodiazepines, buspirone, and SSRIs. Buspirone, because its non-sedating. 25. TCAs and SNRIs block reuptake of norepinephrine and serotonin. Trazodone blocks reuptake of serotonin exclusively. Maprotilene blocks reuptake of norepinephrine exclusively. 26. Nortriptyline has fewer anticholinergic adverse effects than third-generation TCAs such as amitriptyline. 27. A-16, B-7, C-13, D-15, E-12, F-2, G-6, H-14, I-3, J-11, K-8, L-1, M-5, N-9, O-4, P-17, Q-10.

file:///D|/USMLE%20Vids/Workbooks%202012/Chapter%2014%20-%20Psychiatry%202012.txt[8/26/2013 2:13:24 AM]

Renal

How to Use the Workbook with the Videos

Video Part Questions Renal Anatomy 12 Renal Physiology part 1 319 Renal Physiology part 2 2033 Renal Physiology part 3 3441 Renal Pathology part 1 4246 Renal Pathology part 2 4750 Renal Pathology part 3 5164 Renal Pharmacology 6571

file:///D|/USMLE%20Vids/Workbooks%202012/Chapter%2015%20-%20Renal.txt[8/26/2013 2:13:25 AM]

Questions

ANATOMY 1. Why is the left kidney harvested for transplantation rather than the right? (p. 504) ______________________________________________________________________________ 2. Ureters pass __________ (over/under) the uterine artery and the ductus deferens. (p. 505) PHYSIOLOGY 3. Extracellular fluid consists of __________ (high/low) sodium chloride and __________ (high/low) potassium, whereas intracellular fluid consists of __________ (high/low) sodium chloride and __________ (high/low) potassium. (p. 505) 4. What is the 60-40-20 rule of total body weight? (p. 505) ______________________________________________________________________________ ______________________________________________________________________________ 5. The fenestrated capillary endothelium of the glomerular filtration barrier is responsible for the filtration of plasma by which characteristic: size or charge? (p. 505) ______________________________________________________________________________ 6. The fused basement membrane of the glomerulus containing heparan sulfate is responsible for the filtration of plasma molecules by which characteristic, size or charge? (p. 505) ______________________________________________________________________________ 7. The epithelial layer of the glomerular filtration barrier is formed by which cells? (p. 505) ______________________________________________________________________________ 8. What is the formula for calculating the clearance of substance X, the volume of plasma from which the substance is cleared completely per unit of time? (p. 506) ______________________________________________________________________________ 9. If renal clearance is greater than the glomerular filtration rate (GFR) of substance X, then there is a net tubular __________ (reabsorption/secretion) of substance X. (p. 506) 10. Creatinine clearance slightly __________ (overestimates/underestimates) the GFR rate because creatinine is __________ (secreted/reabsorbed) by the renal tubules. (p. 506) 11. What is the formula for estimating renal blood flow if renal plasma flow is known? (p. 506) ______________________________________________________________________________ 12. What are the effects of prostaglandins on the glomerulus? (p. 506) ______________________________________________________________________________ 13. What are the effects of angiotensin II on the glomerulus? (p. 506) ______________________________________________________________________________ 14. Decreased plasma protein concentration causes __________ (decrease/increase/no change) in renal plasma flow and __________ (decrease/increase/no change) in GFR, which in turn results in __________ (decrease/increase/no change) in the filtration fraction. (p. 507) 15. Constriction of the afferent arteriole causes __________ (decrease/increase/no change) in renal plasma flow and __________ (decrease/increase/no change) in GFR, which in turn results in __________ (decrease/increase/no change) in the filtration fraction. (p. 507) 16. What is the formula for excretion rate? (p. 507) ______________________________________________________________________________ 17. In the nephron, glucose at normal plasma concentrations is reabsorbed in which structure? And by which transporter? (p. 507)_________________________________________________________
file:///D|/USMLE%20Vids/Workbooks%202012/Chapter%2015%20-%20Renal.txt[8/26/2013 2:13:25 AM]

18. At what plasma glucose concentration is the transport mechanism of the proximal tubule completely saturated, leading to glucose spilling into the urine? (p. 507) ______________________________________________________________________________ 19. How and where are amino acids reabsorbed in the kidney? (p. 507) ________________________ ______________________________________________________________________________ 20. What ion is secreted into the lumen of the early proximal convoluted tubule and acts to reabsorb bicarbonate? (p. 508) _____________________________________________________________ 21. Which two ions are actively reabsorbed in the thick ascending loop of Henle? (p. 508) ______________________________________________________________________________ 22. Which three ions are indirectly reabsorbed in the thick ascending loop of Henle? (p. 508) ______________________________________________________________________________ 23. Which hormone controls the reabsorption of calcium in the early distal convoluted tubule? (p. 508) ______________________________________________________________________________ 24. On which segment of the nephron does the hormone aldosterone act? (p. 508) ______________________________________________________________________________ 25. ADHs effect at V2 receptors results in what action? (p. 508) ______________________________________________________________________________ 26. The ratio of solute concentration in the tubular fluid versus plasma (TF/P) can indicate the level of secretion or reabsorption of that solute along the proximal renal tubule. If the TF/P ratio of that solute is less than that of inulin, there is net ________________ (reabsorption/secretion) along the proximal tubule. (p. 509) 27. Along the length of the proximal tubule, does the relative concentration of chloride increase, decrease, or stay the same? (p. 509) _________________________________________________ 28. Which five actions of angiotensin II serve to increase intravascular volume and blood pressure? (p. 510) ______________________________________________________________________________ _______________________________________________________________________________ 29. What is the site of action of angiotensin-converting enzyme? (p. 510) _______________________ 30. When blood pressure falls, the kidneys release which proteolytic enzyme? (p. 510) ______________________________________________________________________________ 31. ADH primarily regulates __________ (osmolarity/blood volume), whereas aldosterone primarily regulates __________ (osmolarity/blood volume). However, in __________ (low/high) volume states, both ADH and aldosterone act to protect __________ (osmolarity/blood volume). (p. 510) 32. What are the effects of aldosterone secretion? (p. 510) __________________________________ ______________________________________________________________________________ ______________________________________________________________________________ 33. Atrial natriuretic peptide __________ (decreases/increases) renin secretion and __________ (decreases/increases) the GFR. (p. 510) 34. Which cells produce renin in the kidney? (p. 511) _______________________________________ 35. Which hormone is released by the endothelial cells of renal peritubular capillaries in response to hypoxia? (p. 511) ________________________________________________________________ 36. Which enzyme from the kidney is activated by PTH, and what is the function of that enzyme? (p. 511) __________________________________________________________________________ 37. In the chart below, checkmark the effect that each condition has on the potassium shift. (p. 512) Effect Shifts K+ Into Cell . Hypokalemia Shifts K+ Out of Cell . Hyperkalemia Acidosis

Alkalosis
file:///D|/USMLE%20Vids/Workbooks%202012/Chapter%2015%20-%20Renal.txt[8/26/2013 2:13:25 AM]

-Adrenergic agonists

-Adrenergic antagonists

Cell lysis

Digitalis

Hyperosmolarity

Hypo-osmolarity

Insulin

Insulin deficiency

38. By which mechanism does digitalis cause hyperkalemia? (p. 512) __________________________ 39. What is the primary electrolyte disturbance in metabolic acidosis? (p. 513) ___________________ 40. What is the compensatory respiratory response to metabolic acidosis, and does PCO2 increase or decrease? (p. 513) _______________________________________________________________ ______________________________________________________________________ 41. What are the ten causes of increased anion gap metabolic acidosis? (p. 514) ______________________________________________________________________________ ______________________________________________________________________________ PATHOLOGY 42. What four clinical findings are associated with nephritic syndrome? (p. 515) ________________________________________________________________________________________________ _________________________________________________________ 43. Match the nephritic syndrome with its characteristic finding on light microscopy. (p. 516) _____ A. Acute poststreptococcal glomerulonephritis 1. Crescent-shaped scars _____ B. Alports syndrome 2. Immune complexes in mesangium _____ C. Diffuse proliferative glomerulonephritis 3. Lumps and bumps
file:///D|/USMLE%20Vids/Workbooks%202012/Chapter%2015%20-%20Renal.txt[8/26/2013 2:13:25 AM]

_____ D. IgA glomerulopathy 4. Split basement membrane _____ E. Rapidly progressive glomerulonephritis 5. Wire looping of capillaries 44. A 10-year-old boy presents with swollen ankles and periorbital edema, which were beginning to resolve without intervention. Electron microscopy of a kidney biopsy specimen shows subepithelial immune complex humps. Which form of nephritic syndrome does he most likely have? (p. 516) ______________________________________________________________________________ 45. Wegener's granulomatosis is __________ (c-ANCA/p-ANCA) positive, whereas microscopic polyarteritis is __________ (c-ANCA/p-ANCA) positive. (p. 516) 46. For which systemic disease is diffuse proliferative glomerulonephritis the most common cause of death? (p. 516) __________________________________________________________________ 47. What four clinical findings are associated with nephrotic syndrome? (p. 517) __________________ ______________________________________________________________________________ 48. Match the nephrotic syndrome with its characteristic findings. (pp. 517-518) _____ A. Amyloidosis 1. Associated with chronic disease _____ B. Diabetic glomerulonephropathy 2. Foot process effacement on EM _____ C. Focal segmental glomerulosclerosis 3. Hyalinosis on LM _____ D. Membranous glomerulonephritis 4. Kimmelstiel-Wilson lesion on LM _____ E. Membranoproliferative glomerulonephritis 5. Spike-&-dome appearance on EM _____ F. Minimal change disease 6. Tram-track appearance on EM 49. What is the most common glomerular disease seen in patients with HIV? (p. 517) ______________________________________________________________________________ 50. In diabetic glomerulonephropathy, what causes mesangial expansion? (p. 518) ______________________________________________________________________________ 51. Kidney stones are most commonly composed of what element? (p. 519) ______________________________________________________________________________ 52. Both antifreeze and vitamin C abuse can result in the formation of which type of crystals? (p. 519) ______________________________________________________________________________ 53. An 80-year-old man with leukemia presents with hematuria and right-sided flank pain. Which type of kidney stone is he most likely to have? And how would this stone appear on x-ray? (p. 519) ______________________________________________________________________________ 54. Name the four components of the WAGR complex. (p. 520) _________________ ______________________________________________________________________________ 55. Match the renal pathology with its characteristic findings. (pp. 520-523) _____ A. Acute pyelonephritis 1. Associated with aniline dye exposure _____ B. Acute tubular necrosis 2. Associated with diabetes _____ C. Bladder cancer 3. Associated with obstetric catastrophe _____ D. Chronic pyelonephritis 4. Associated with vHL syndrome _____ E. Diffuse cortical necrosis 5. Muddy brown casts in urine _____ F. Drug-induced interstitial nephritis 6. Nephroblastoma _____ G. Renal cell carcinoma 7. Painless hematuria _____ H. Renal papillary necrosis 8. Pyuria and azotemia _____ I. Transitional cell carcinoma 9. Thyroidization of kidney _____ J. Wilms tumor 10. WBC casts in urine 56. What are three causes of acute tubular necrosis? (p. 522) ________________________________ ______________________________________________________________________________ 57. Which three general types of renal dysfunction can lead to acute renal failure? (p. 522) ______________________________________________________________________________ 58. True or False: Unilateral postrenal outflow obstruction can lead to acute renal failure. (p. 522)___________________________________________________________________________ 59. A patient's urine osmolarity is <350 mOsm/L, urine sodium level is >40 mEq/L, fractional excretion of sodium is >4%, and BUN/creatinine ratio is >15:1. Is the etiology of the acute renal failure most likely to be prerenal, renal, or postrenal? (p. 522) _______________________________________
file:///D|/USMLE%20Vids/Workbooks%202012/Chapter%2015%20-%20Renal.txt[8/26/2013 2:13:25 AM]

60. A patients urine osmolarity is >500 mOsm/L, urine sodium level is <10 mEq/L, fractional excretion of sodium is <1%, and BUN/creatinine ratio is >20:1. Is the etiology of the acute renal failure most likely to be prerenal, renal, or postrenal? (p. 522) _______________________________________ 61. What are the eight consequences of renal failure? (p. 523) _______________________________ ________________________________________________________________________________________________ ____________________________________________________________ 62. Which type of mutation causes autosomal-dominant polycystic kidney disease (formerly adult polycystic kidney disease)? (p. 523) _________________________________________________ 63. What are the two major causes of death associated with autosomal-dominant polycystic kidney disease? (p. 523) ________________________________________________________________ 64. What are the complications of autosomal-recessive polycystic kidney disease in utero and after the neonatal period? (p. 523) __________________________________________________________ PHARMACOLOGY 65. What is the mechanism of action of acetazolamide? (p. 525) ______________________________ 66. What is the mechanism of action of furosemide? (p. 525) _________________________________ 67. Which loop diuretic is used for diuresis of patients who are allergic to sulfa drugs? (p. 525) ______________________________________________________________________________ 68. What are the effects of hydrochlorothiazide toxicity? (p. 526) ______________________________ 69. What is the mechanism of action of spironolactone? (p. 526) ______________________________ ______________________________________________________________________________ 70. What is the mechanism by which ACE inhibitors can cause angioedema? (p. 527) ______________________________________________________________________________ 71. What are three clinical uses of ACE inhibitors? (p. 527) __________________________________ ______________________________________________________________________________ Answers ANATOMY 1. Because the left kidney has a longer renal vein. 2. Under. (Remember: "water [ureters] under the bridge [artery and ductus deferens]") PHYSIOLOGY 3. High; low; low; high. (Remember: HIKIN': HIgh K Intracellular.) 4. 60% of total body weight is made up of total body water, 40% is made up of intracellular fluid, and 20% is made up of extracellular fluid. 5. Size. 6. Net charge. 7. Podocyte foot processes. 8. Renal clearance of X = the urine concentration of X times the urine flow rate, divided by the plasma concentration of X (Cx = Ux V/Px). 9. Secretion. 10. Overestimates; secreted. (The plasma concentration of creatinine is slightly lower than it would be from filtration alone.) 11. Renal blood flow = renal plasma flow divided by (1 - the hematocrit), or RBF = RPF/(1 - Hct). In a normal individual, renal blood flow will be approximately double the renal plasma flow. 12. Prostaglandins cause dilation of the afferent arteriole and an increase in the GFR. 13. Angiotensin II causes constriction of the efferent arteriole and an increase in the GFR. 14. No change; increase; increase. 15. Decrease; decrease; no change. 16. Excretion rate = VUx; where V is the urine flow rate and Ux is the urine concentration of X. 17. Glucose is reabsorbed in the proximal tubule by sodium/glucose co-transport. 18. 350 mg/dL.
file:///D|/USMLE%20Vids/Workbooks%202012/Chapter%2015%20-%20Renal.txt[8/26/2013 2:13:25 AM]

19. Amino acids are filtered and then reabsorbed from the urine at the level of the proximal tubule by three distinct sodium-dependent transporters, each with competitive inhibition. 20. Hydrogen ions. 21. Sodium, potassium, and chloride. 22. Magnesium and calcium. 23. PTH. 24. Collecting tubule. 25. Insertion of aquaporin water channels on the luminal side of the collecting tubules, resulting in increased water reabsorption. 26. Reabsorption. 27. Increase. (Chloride is reabsorbed distally in the nephron.) 28. Vasoconstriction; stimulation of sodium resorption in the proximal tubule; release of aldosterone from the adrenal cortex; release of ADH from the posterior pituitary; and simulation of thirst via the hypothalamus. 29. Lungs. 30. Renin. 31. Osmolarity; blood volume; low; blood volume. 32. Aldosterone secretion from the adrenal cortex increases sodium channel and sodium/potassium pump insertion in principal cells and enhances potassium and hydrogen excretion by upregulating potassium channels in the principal cells and hydrogen ion channels in the intercalated cells. These actions create a favorable gradient for sodium and water reabsorption. 33. Decreases; increases. 34. Juxtaglomerular cells. 35. Erythropoietin. 36. 1a-Hydroxylase, which converts 25-OH vitamin D to 1,25(OH)2 vitamin D.

37. Effect Shifts K+ Into Cell . Hypokalemia Shifts K+ Out of Cell . Hyperkalemia Acidosis v Alkalosis v -Adrenergic agonists v -Adrenergic antagonists v Cell lysis

file:///D|/USMLE%20Vids/Workbooks%202012/Chapter%2015%20-%20Renal.txt[8/26/2013 2:13:25 AM]

v Digitalis v Hyperosmolarity v Hypo-osmolarity v Insulin v Insulin deficiency v

38. Digitalis blocks the Na+/K+/APTase pump from pumping K+ into the cell in exchange for Na+, thus leaving the K+ outside the cell. 39. Decreased serum bicarbonate. 40. Hyperventilation, which causes PCO2 to decrease. 41. Methanol (formic acid), Uremia, Diabetic ketoacidosis, Paraldehyde or Phenformin, Iron tables or Isoniazid, Lactic acidosis, Ethylene glycol (oxalic acid), and Salicylates. (Remember: MUDPILES.) PATHOLOGY 42. Azotemia, oliguria, hypertension, and proteinuria <3.5 g/day. 43. A-3, B-4, C-5, D-2, E-1. 44. Acute poststreptococcal glomerulonephritis. 45. c-ANCA; p-ANCA. 46. Systemic lupus erythematosus. 47. Massive proteinuria (>3.5 g/day), edema, fatty casts, and hyperlipidemia. 48. A-1, B-4, C-3, D-5, E-6, F-2. 49. Focal segmental glomerular sclerosis. 50. Nonenzymatic glycosylation of the efferent arterioles, leading to an increased GFR and thus mesangial expansion. 51. Calcium in the form of calcium oxalate, calcium phosphate, or both. 52. Oxalate crystals. 53. Because of the patients leukemia (a disease with high cell turnover), he is at risk for developing uric acid stones due to hyperuricemia, which are radiolucent and do not appear on x-ray studies. 54. WAGR complex = Wilms' tumor, Aniridia, Genitourinary malformation, and mental-motor Retardation. 55. A-10, B-5, C-7, D-9, E-3, F-8, G-4, H-2, I-1, J-6. 56. Renal ischemia, crush injury, and toxins. 57. Prerenal (e.g., hypotension and reduced renal blood flow), intrinsic renal (e.g., tubular necrosis), and postrenal (outflow obstruction). 58. False; bilateral (not unilateral) postrenal outflow obstruction leads to acute renal failure. 59. Postrenal. 60. Prerenal. 61. Na+/H2O retention, hyperkalemia, metabolic acidosis, uremia, anemia, renal osteodystrophy, dyslipidemia, and retarded grown and development in children.
file:///D|/USMLE%20Vids/Workbooks%202012/Chapter%2015%20-%20Renal.txt[8/26/2013 2:13:25 AM]

62. An autosomal dominant mutation in the APKD1 or APKD2 gene. 63. Death usually results from complications of chronic kidney disease or hypertension (due to increased renin production). 64. Renal failure in utero, from autosomal recessive polycystic kidney disease, can lead to Potters syndrome (see page 132). After the neonatal period, potential complications include hypertension, portal hypertension, and progressive renal insufficiency. PHARMACOLOGY 65. Acetazolamide acts as a carbonic anhydrase inhibitor, causing self-limited sodium bicarbonate diuresis and a reduction in total-body bicarbonate stores. 66. Furosemide inhibits the Na+/K+/Cl- cotransport system in the thick ascending limb of the loop of Henle, thereby abolishing the hypertonicity of the medulla and preventing the concentration of urine. 67. Ethacrynic acid. 68. Glucose (hyperGlycemia), lipids (hyperLipidemia), uric acid (hyperUricemia), and calcium (hyperCalcemia). (Remember: HyperGLUC.) 69. Spironolactone competitively antagonizes the aldosterone receptor in the cortical collecting tubule. 70. ACE inhibitors prevent the inactivation of bradykinin, a potent vasodilator. Increased bradykinin levels can lead to angioedema in susceptible individuals. 71. To treat hypertension, to treat congestive heart failure, and to slow the progression of diabetic renal disease.

file:///D|/USMLE%20Vids/Workbooks%202012/Chapter%2015%20-%20Renal.txt[8/26/2013 2:13:25 AM]

Reproductive

How to Use the Workbook with the Videos

Video Part Questions Reproductive Anatomy 16 Reproductive Physiology part 1 715 Reproductive Physiology part 2 1619 Reproductive Pathology part 1 2022 Reproductive Pathology part 2 2326, 29 Reproductive Pathology part 3 27, 29 Reproductive Pathology part 4 2829 Reproductive Pathology part 5 30 Reproductive Pathology part 6 3133 Reproductive Pharmacology 3441

file:///D|/USMLE%20Vids/Workbooks%202012/Chapter%2016%20-%20Reproductive%202012.txt[8/26/2013 2:13:26 AM]

Questions ANATOMY 1. Describe the venous drainage flow from the left ovary/testicle. Describe the venous drainage flow from the right ovary/testicle. To which lymph nodes do these structures drain? (p. 530) ______________________________________________________________________________ ______________________________________________________________________________ 2. On which side are varicoceles more common? (p. 530) __________________________________ 3. Match the female reproductive system ligament to the structures it connects. (p. 530) _____ A. Connects cervix to side wall of pelvis 1. Broad ligament _____ B. Connects ovaries to lateral pelvic wall 2. Cardinal ligament _____ C. Connects ovaries to lateral uterus 3. Ligament of the ovary _____ D. Connects uterine fundus to labia majora 4. Round ligament _____ E. Connects uterus, fallopian tubes, and 5. Suspensory ligament of the ovary ovaries to side wall of pelvis 4. What is the pathway of sperm during ejaculation? (p. 531) ________________________________ ______________________________________________________________________________ 5. Why arent gametes attacked by a mans immune system? (p. 532) _________________________ ______________________________________________________________________________ 6. Which cells in the male reproductive tract secrete inhibin? Which secrete testosterone? (p. 532) ______________________________________________________________________________ PHYSIOLOGY 7. Which stages of spermatogenesis occur immediately outside and inside the blood-testis barrier? (p.533)_________________________________________________________________________ 8. What are the three major forms of androgens? How do they compare in potency? (p. 534) _______ ______________________________________________________________________________ 9. List the five functions of testosterone. (p. 534) __________________________________________ ______________________________________________________________________________ 10. What are the three major forms of estrogen? How do they compare in potency? (p. 535) ________ ______________________________________________________________________________ 11. What are the three major sources of estrogens? (p. 535) _________________________________ ______________________________________________________________________________ 12. Which estrogen is used as an indicator of fetal well-being? (p. 535) _________________________ 13. List the four functions of estrogen. (p. 535) ____________________________________________ ______________________________________________________________________________ ______________________________________________________________________________ 14. Identify which hormone levels are shown on the image below. (p. 536) 15. Define the following terms. (p. 536) A. Menometrorrhagia __________________________________________________________ B. Metrorrhagia _______________________________________________________________ C. Oligomenorrhea _____________________________________________________________ D. Polymenorrhea _____________________________________________________________ E. Menorrhagia _______________________________________________________________ 16. In a nonpregnant woman, where is hCG synthesized? Where is hCG synthesized after pregnancy? After pregnancy, when is hCG first detectable in the blood? In the urine? (pp. 537-538) ______________________________________________________________________________ 17. In what pathologic states can hCG levels be elevated? (p. 538) ____________________________ ______________________________________________________________________________
file:///D|/USMLE%20Vids/Workbooks%202012/Chapter%2016%20-%20Reproductive%202012.txt[8/26/2013 2:13:26 AM]

18. What hormonal changes occur during menopause? (p. 538) ______________________________ ______________________________________________________________________________ 19. What are the sequelae of menopause? (p. 538) ________________________________________ PATHOLOGY 20. Klinefelters syndrome is associated with which genotype? What are the clinical findings? (p. 539) ______________________________________________________________________________ 21. Turners syndrome is associated with which genotype? What are the clinical findings? (p. 539) ___ ______________________________________________________________________________ 22. In the chart below, indicate whether the lab findings are elevated, decreased, or normal. (p. 539) Diagnosis LH Testosterone Defective androgen receptor

Hypogonadotropic hypogonadism

Primary hypogonadism

Testosterone-secreting tumor or exogenous steroids

23. What are the risk factors for preeclampsia/eclampsia? (p. 541) ____________________________ ______________________________________________________________________________ 24. What is HELLP syndrome? (p. 541) __________________________________________________ 25. Define the following terms and list the risk factors. (pp. 541-542) A. Abruptio placentae __________________________________________________________ __________________________________________________________________________ B. Placenta accreta ____________________________________________________________ __________________________________________________________________________ C. Placenta previa _____________________________________________________________ __________________________________________________________________________ 26. What are the most common risk factors for ectopic pregnancy? (p. 542) _____________________ ______________________________________________________________________________ 27. Rank the incidence of gynecologic tumors in the United States from most common to least common: Cervical, endometrial, and ovarian. Then rank the prognosis of these tumors from worst to best. (p. 544) _________________________________________________________________ 28. What are the hormonal changes in Polycystic ovarian syndrome? (p. 545) ___________________ _____________________________________________________________________________ 29. Match these gynecologic conditions with their associated diagnostic findings. (pp. 542-547) _____ A. Cervical carcinoma in situ 1. <0.5 L of amniotic fluid _____ B. Choriocarcinoma 2. >1.5-2 L of amniotic fluid _____ C. Dysgerminoma 3. Call-Exner bodies
file:///D|/USMLE%20Vids/Workbooks%202012/Chapter%2016%20-%20Reproductive%202012.txt[8/26/2013 2:13:26 AM]

_____ D. Endometriosis 4. Chocolate cysts _____ E. Granulosa cell tumor 5. Dermoid cyst _____ F. Invasive cervical carcinoma 6. HPV types 16 and 18 _____ G. Krukenberg tumor 7. Increased AFP level _____ H. Oligohydramnios 8. Increased CA-125 level _____ I. Ovarian cancer 9. Increased hCG level _____ J. Polyhydramnios 10. Increased hCG and LDH levels _____ K. Teratoma 11. Koilocytes _____ L. Yolk sac tumor 12. Signet cells 30. Match these breast tumors with their associated diagnostic findings. (pp. 548-549) _____ A. Ductal carcinoma in situ 1. Bilateral _____ B. Fibroadenoma 2. Ductal hyperplasia _____ C. Inflammatory carcinoma 3. Eczematous patches on nipple _____ D. Intraductal papilloma 4. Hard mass with sharp margins _____ E. Invasive ductal carcinoma 5. Increased tenderness with menstruation _____ F. Invasive lobular carcinoma 6. Peau dorange _____ G. Pagets disease of breast 7. Serous nipple discharge; benign 31. Why is benign prostatic hyperplasia more likely to cause urinary retention than prostatic adenocarcinoma? (p. 551) _________________________________________________________ ______________________________________________________________________________ 32. How are most prostate cancers diagnosed? (p. 551) _____________________________________ 33. Match these testicular conditions with their associated diagnostic findings. (p. 552) _____ A. Choriocarcinoma 1. Androblastoma _____ B. Embryonal carcinoma 2. Associated with lack of circumcision _____ C. Hydrocele 3. Dilated epididymal duct _____ D. Leydig cell tumor 4. Dilated vein in pampiniform plexus _____ E. Peyronies disease 5. Fibrous tissue formation bends penis _____ F. Seminoma 6. Increased AFP and hCG levels _____ G. Sertoli cell tumor 7. Increased fluid around testicle _____ H. Spermatocele 8. Increased hCG level _____ I. Squamous cell carcinoma 9. Most common testicular tumor _____ J. Testicular lymphoma 10. Most common testicular tumor in older men _____ K. Varicocele 11. Reinke crystals _____ L. Yolk sac tumor 12. Schiller-Duval bodies PHARMACOLOGY 34. Continuous leuprolide has __________ (agonist/antagonist) properties, while pulsatile leuprolide has __________ (agonist/antagonist) properties. (p. 554) 35. What drug is commonly used to treat BPH? (p. 554) _____________________________________ 36. When would estrogen be prescribed to a man? (p. 554) __________________________________ 37. How does clomiphene stimulate ovulation? (p. 555) _____________________________________ ______________________________________________________________________________ 38. What is the main clinical use of tamoxifen? (p. 555) _____________________________________ 39. How do oral contraceptive pills prevent pregnancy? (p. 555) ______________________________ ______________________________________________________________________________ 40. In which patients are oral contraceptive pills contraindicated? (p. 555) _______________________ ______________________________________________________________________________ 41. What toxicities are associated with sildenafil and vardenafil? Which class of heart medications should these never be combined with? (p. 556) ________________________________________ ______________________________________________________________________________ Answers

file:///D|/USMLE%20Vids/Workbooks%202012/Chapter%2016%20-%20Reproductive%202012.txt[8/26/2013 2:13:26 AM]

ANATOMY 1. Left ovary/testicle . left gonadal vein . left renal vein . inferior vena cava. Right ovary/testicle . right gonadal vein . inferior vena cava. Para-aortic lymph nodes. 2. Left side. 3. A-2, B-5, C-3, D-4, E-1. 4. Remember SEVEN-UP: Seminiferous tubules . Epididymus . Vas deferens . Ejaculatory ducts . (Nothing) . Urethra . Penis. 5. Adjacent Sertoli cells form tight junctions that serve as a blood-testis barrier. 6. Sertoli cells. Leydig cells. PHYSIOLOGY 7. Spermatogonium and 1 spermatocyte 8. Dihydrotestosterone is more potent than testosterone, which is more potent than androstenedione. 9. Differentiation of the internal genitalia (except prostate), growth spurts, deeping of the voice, closing of the epipyseal plate, and libido. 10. Estradiol is more potent than estrone, which is more potent than estriol. 11. Ovary (estradiol), placenta (estriol), and blood (aromatization). 12. Estriol is used as a marker of fetal well-being. 13. Development of genitalia and breast/female fat distribution, growth of follicle/endometrial proliferation, upregulation of estrogen/luteinizing hormone/progesterone receptors and feedback inhibition of FSH/LH, and increased transport of proteins.

14. 15. A. Menometrorrhagia: heavy and irregular menstruation. B. Metrorrhagia: frequent/irregular menstruation. C. Oligomenorrhea: cycle of >35 days. D. Polymenorrhea: cycle of <21 days. E. Menorrhagia: heavy menstruation. 16. Ovaries; in syncytiotrophoblasts of the placenta; 1 week (blood); 2 weeks (urine). 17. hCG levels can be elevated in hydatidiform moles, choriocarcinoma, and gestational trophoblastic tumors. 18. Decreased estrogen and increased FSH, LH, and GnRH levels. 19. Hirsutism, Hot flashes, Atrophy of Vagina, Osteoporosis, and Coronary artery disease. (Remember: HHAVOC.) PATHOLOGY 20. XXY; testicular atrophy, eunuchoid body shape, tall stature, long extremities, and gynecomastia. 21. XO; short stature, ovarian dysgenesis, webbing of neck, preductal coarctation of aorta, and primary amenorrhea.

22. Diagnosis LH
file:///D|/USMLE%20Vids/Workbooks%202012/Chapter%2016%20-%20Reproductive%202012.txt[8/26/2013 2:13:26 AM]

Testosterone Defective androgen receptor . . Hypogonadotropic hypogonadism . . Primary hypogonadism . . Testosterone-secreting tumor or exogenous steroids . .

23. Preexisting hypertension, diabetes, chronic renal disease, and autoimmune disorders. 24. Hemolysis, Elevated LFTs, Low Platelets. 25. A. Abruptio placenta: premature detachment of placenta from site of implantation. Risk factors include smoking, hypertension, and cocaine use. C. Placenta accreta: defective decidual layer allows placenta to attach to the myometrium, which prevents the placenta from separating after birth. Risk factors include prior C-section, inflammation, and placenta previa. D. Placenta previa: placenta attaches to lower uterine segment. Risk factors include multiparity and prior C-section. 26. History of infertility, salpingitis (pelvic inflammatory disease), ruptured appendix, and prior tubal surgery. 27. . LH, . FSH, . testosterone 28. For incidence: endometrial > ovarian > cervical. For prognosis: ovarian > cervical > endometrial. 29. A-6, B-9, C-10, D-4, E-3, F-11, G-12, H-1, I-8, J-2, K-5, L-7. 30. A-2, B-5, C-6, D-7, E-4, F-1, G-3. 31. In BPH the periurethral lobes enlarge to compress the urethra. Prostatic adenocarcinoma occurs most commonly in the posterior lobe of the prostate, and the tumor would need to grow quite large before it impinged upon the urethra enough to cause urinary retention. 32. By digital rectal examination and prostate biopsy. 33. A-8, B-6, C-7, D-11, E-5, F-9, G-1, H-3, I-2, J-10, K-4, L-12. PHARMACOLOGY 34. Antagonist; agonist. 35. Finasteride. 36. If he has androgen-dependent prostate cancer. 37. By preventing normal feedback inhibition and increasing LH and FSH release from the pituitary. 38. To treat estrogen receptorpositive breast cancer. 39. OCPs prevent the estrogen surge, which in turn prevents the LH surge, and thus ovulation. 40. Smokers >35 years old, patients with a history of thromboembolism and stroke, and those with a history of estrogen-dependent tumor. 41. Headache, flushing, dyspepsia, impaired blue-green color vision, and severe hypotension. They should never be taken with nitrates, as this combination can exacerbate hypotension even further.

file:///D|/USMLE%20Vids/Workbooks%202012/Chapter%2016%20-%20Reproductive%202012.txt[8/26/2013 2:13:26 AM]

Respiratory

How to Use the Workbook with the Videos

Video Part Questions Respiratory Anatomy 112 Respiratory Physiology part 1 1315 Respiratory Physiology part 2 1618 Respiratory Physiology part 3 1924 Respiratory Physiology part 4 2532 Respiratory Pathology part 1 3336 Respiratory Pathology part 2 3742 Respiratory Pathology part 3 4346 Respiratory Pathology part 4 4748 Respiratory Pathology part 5 4950 Respiratory Pharmacology 5156

file:///D|/USMLE%20Vids/Workbooks%202012/Chapter%2017%20-%20Respiratory%202012.txt[8/26/2013 2:13:27 AM]

Questions

ANATOMY 1. Which six structures make up the conducting zone of the respiratory tree? (p. 558) _____________ ______________________________________________________________________________ 2. What are the four functions of the conducting zone of the respiratory tree? (p. 558) _____________ ______________________________________________________________________________ 3. Which anatomic areas are encompassed by the respiratory zone, and what is their major function? (p. 558) ________________________________________________________________________ 4. Match the functions and characteristics with the cell type that best describes them. (Numbers may be used more than once.) (p. 559) _____ A. Clear debris from alveoli 1. Clara cells _____ B. Ciliated 2. Macrophages _____ C. Comprise 3% of pneumocytes 3. Pseudostratified columnar cells _____ D. Cuboidal 4. Type I pneumocytes _____ E. Degrade toxins 5. Type II pneumocytes _____ F. Make up the majority of pneumocytes _____ G. Nonciliated _____ H. Precursors to type I pneumocytes _____ I. Produce surfactant _____ J. Squamous cells 5. What is the function of surfactant? (p. 559) ____________________________________________ 6. An O2 molecule is in the alveolus. In order, note the cells that the O2 molecule will cross to be able to bind to heme in RBCs. (p. 559) ___________________________________________________ 7. A CO2 molecule resides in an RBC within the capillary lumen. In order, note the cells that the CO2 molecule will cross before it is exhaled. (p. 559) ________________________________________ 8. If you aspirate a peanut while standing upright, where will it most likely go? (p. 560) ____________ ______________________________________________________________________________ 9. If you aspirate a peanut while while lying supine, where will it most likely go? (p. 560) ___________ ______________________________________________________________________________ 10. Match the structure and the thoracic vertebral level where it crosses the diaphragm. (Numbers may be used more than once.) (p. 560) _____ A. Aorta 1. T8 _____ B. Azygous vein 2. T10 _____ C. Esophagus 3. T12 _____ D. Inferior vena cava _____ E. Thoracic duct _____ F. Vagal trunk
file:///D|/USMLE%20Vids/Workbooks%202012/Chapter%2017%20-%20Respiratory%202012.txt[8/26/2013 2:13:27 AM]

11. During exercise, which three muscle groups are used for inspiration? (p. 560) _________________ ______________________________________________________________________________ 12. Name the four abdominal wall muscle groups and the chest wall muscles used for expiration during exercise. (p. 560) ________________________________________________________________ PHYSIOLOGY 13. Match the term with its description. (p. 561) _____ A. Additional air that can be inspired after a normal breath 1. ERV _____ B. Air remaining in lung after maximal expiration 2. FRC _____ C. Air that can still be exhaled after normal expiration 3. IC _____ D. Air that moves into lung with each quiet inspiration 4. IRV _____ E. IRV + TV 5. RV _____ F. IRV + TV + ERV + RV 6. TLC _____ G. RV + ERV 7. TV _____ H. TV + IRV + ERV 8. VC 14. On the image below, fill in the rectangles to describe the lung volume measurement. (p. 561) 15. In which three conditions is lung compliance decreased? (p. 562) __________________________ ______________________________________________________________________________ 16. What are the two forms of hemoglobin? Which has a greater affinity for O2? (p. 562) ___________ ______________________________________________________________________________ 17. When the oxygen-hemoglobin dissociation curve shifts to the right, the affinity of hemoglobin for O2 __________ (decreases/increases). When the oxygen-hemoglobin dissociation curve shifts to the left, the affinity of hemoglobin for O2 __________ (decreases/increases). (p. 563) 18. In the chart below, checkmark whether the effect shifts the oxygen-hemoglobin dissociation curve to the left or to the right. (p. 563) Effect Shift to the Left Shift to the Right Decreased 2,3-DPG

Decreased pH

Decreased temperature

Fetal hemoglobin

High altitude

Increased metabolic needs


file:///D|/USMLE%20Vids/Workbooks%202012/Chapter%2017%20-%20Respiratory%202012.txt[8/26/2013 2:13:27 AM]

Increased pH

Increased temperature

19. Name seven causes of secondary pulmonary hypertension. (p. 564) ________________________ _______________________________________________________________________________ 20. How is the A-a gradient calculated? (p. 565) ___________________________________________ 21. Which three pathologic processes can lead to an increased A-a gradient? (p. 565) ____________ ______________________________________________________________________________ 22. Which two processes lead to hypoxemia with a normal A-a gradient? (p. 565) ________________ ______________________________________________________________________________ 23. Which three processes can lead to hypoxemia with an increased A-a gradient? (p. 565) ________ ______________________________________________________________________________ 24. Name five processes that can lead to hypoxia (i.e., decreased O2 delivery to tissue). (p. 565) _____ ______________________________________________________________________________ 25. With respect to the lung apex (zone 1), arrange the following in order of increasing pressure: artery, vein, alveolus. (p. 566) ____________________________________________________________ 26. With respect to zone 2 of the lung, arrange the following in order of increasing pressure: artery, vein, alveolus. (p. 566) ____________________________________________________________ 27. With respect to the lung base (zone 3), arrange the following in order of increasing pressure: artery, vein, alveolus. (p. 566) ____________________________________________________________ 28. In which forms is CO2 transported from the tissues to the lungs? (p. 566) ____________________ 29. What enzyme catalyzes the conversion of CO2 and water into carbonic acid? (p. 566)___________ ______________________________________________________________________________ 30. What is the name for the effect in which oxygenation of hemoglobin within the lungs promotes the dissociation of CO2 from hemoglobin? (p. 566) _________________________________________ 31. In peripheral tissues, the right shift of the oxygen-hemoglobin dissociation curve that results from decreased pH causes an unloading of O2. What is the name for this effect? (p. 566) ____________ 32. For each item in the chart below, indicate whether altitude or exercise would induce the response. (p. 567) Effect Response to Altitude Response to Exercise Decreased pH

Increased 2,3-DPG

Increased CO2 production

file:///D|/USMLE%20Vids/Workbooks%202012/Chapter%2017%20-%20Respiratory%202012.txt[8/26/2013 2:13:27 AM]

Increased erythropoietin

Increased O2 consumption

Increased mitochondria

Increased pulmonary blood flow

Increased renal excretion of bicarbonate

Increased ventilation

More uniform V/Q ratio from apex to base

Right ventricular hypertrophy

PATHOLOGY 33. Name the six most common causes of emboli to the lungs. (p. 567) ________________________ ______________________________________________________________________________ 34. Which three factors that promote blood coagulation are collectively known as Virchow's triad? (p. 568) ________________________________________________________________________ 35. For each patient, indicate the most likely type of pulmonary embolus. (p. 568) A. A 30-year-old postpartum woman _______________________________________________ B. An 18-year-old man who sustained a motor vehicle collision __________________________ C. A 35-year-old professional wreck diver ___________________________________________ D. An 83-year-old woman with a hip fracture after falling _______________________________ 36. Match the characteristic finding with the obstructive lung disease with which it is associated. (Numbers may be used more than once.) (p. 568) _____ A. Associated with Kartageners syndrome 1. Asthma _____ B. Chronic productive cough 2. Bronchiectasis _____ C. Curschmanns spirals 3. Chronic bronchitis _____ D. Increased lung compliance 4. Emphysema _____ E. Hypertrophy of mucus-secreting glands _____ F. Increased susceptibility to aspergillosis _____ G. Mucus plugs _____ H. Permanently dilated airways _____ I. Reid index >50%
file:///D|/USMLE%20Vids/Workbooks%202012/Chapter%2017%20-%20Respiratory%202012.txt[8/26/2013 2:13:27 AM]

_____ J. Results from hyper-responsiveness of bronchi _____ K. Spontaneous pneumothorax _____ L. Wheezing and crackles on auscultation 37. For patients with restrictive lung disease, what is the typical range of FEV1/FVC? (p. 569) _______ ______________________________________________________________________________ 38. Which of the pneumoconioses is associated with an increased incidence of bronchogenic carcinoma? (p. 569) ______________________________________________________________ 39. What are the risk factors for neonatal respiratory distress syndrome? (p. 569) _________________ ______________________________________________________________________________ 40. What are the risk factors for acute respiratory distress syndrome? (p. 570) ___________________ ______________________________________________________________________________ 41. What is central sleep apnea? (p. 570) ________________________________________________ 42. What is obstructive sleep apnea? (p. 570) _____________________________________________ 43. Match the physical examination finding with its associated pathology. (p. 571) _____ A. Dullness to percussion with . fremitus 1. Bronchial obstruction _____ B. Dullness to percussion with . fremitus 2. Lobar pneumonia _____ C. Tracheal deviation away from affected side 3. Pleural effusion _____ D. Decreased resonance with tracheal deviation 4. Tension pneumothorax toward affected side 44. What are the four most common sites of lung cancer metastases? (p. 571) __________________ ______________________________________________________________________________ 45. Match the lung cancer with its characteristic. (Numbers may be used more than once.) (pp. 571-572) _____ A. Arises in sites of prior injury 1. Adenocarcinoma _____ B. Associated with asbestos exposure 2. Carcinoid tumor _____ C. Associated with smoking 3. Large cell carcinoma _____ D. Forms keratin pearls 4. Mesothelioma _____ E. Inoperable 5. Small cell carcinoma _____ F. May be associated with right heart failure 6. Squamous cell carcinoma _____ G. May lead to Lambert-Eaton syndrome _____ H. May produce ACTH or ADH _____ I. Most common lung cancer _____ J. Most common lung cancer among nonsmokers _____ K. Parathyroid-like activity _____ L. Pleomorphic giant cells with cytoplasmic leukocyte fragments in cytoplasm _____ M. Precursors are neuroendocrine cells _____ N. Psammoma bodies _____ O. Secretes serotonin 46. What are the three findings of Horners syndrome? (p. 572) _______________________________ ________________________________________________________________________________________________ ____________________________________________________________ 47. Match the organism with the most likely pneumonia type. (There may be more than one correct pneumonia type, and numbers may be used more than once.) (p. 573) _____ A. Adenoviruses 1. Bronchopneumonia _____ B. Chlamydia 2. Lobar pneumonia _____ C. H. flu 3. Interstitial pneumonia _____ D. Klebsiella _____ E. Legionella _____ F. Mycoplasma _____ G. RSV _____ H. S. aureus _____ I. S. pneumoniae
file:///D|/USMLE%20Vids/Workbooks%202012/Chapter%2017%20-%20Respiratory%202012.txt[8/26/2013 2:13:27 AM]

_____ J. S. pyogenes 48. What organisms are most likely found in a lung abscess? (p. 573) __________________________ ______________________________________________________________________________ 49. Name three causes of transudative pleural effusions? (p. 574) _____________________________ ______________________________________________________________________________ 50. Name three causes of exudative pleural effusions? (p. 574) _______________________________ ______________________________________________________________________________ PHARMACOLOGY 51. With respect to toxicity, what is the main difference between first- and second-generation H1-blockers? (p. 575) ________________________________________________________________ 52. What is the mechanism of action of albuterol? (p. 576) ___________________________________ 53. Ipratropium is a member of which class of drugs? (p. 576) ________________________________

54. In the image below, fill in the rectangles to identify the treatments for asthma. (p. 576) 55. What is the mechanism of action of bosentan? (p. 577) __________________________________ 56. What is the mechanism of action of dextromethorphan? (p. 577) ___________________________ Answers ANATOMY 1. Nose, pharynx, trachea, bronchi, bronchioles, and terminal bronchioles. 2. The conducting zone brings air in and out; it also warms, humidifies, and filters the air. 3. The respiratory bronchioles, the alveolar ducts, and alveoli, all of which function in gas exchange. 4. A-2, B-3, C-5, D-5, E-1, F-4, G-1, H-5, I-5, J-4. 5. Decreases alveolar surface tension. 6. Type I pneumocytes followed by endothelial cells. 7. The majority of CO2 will exit RBCs as bicarbonate (in exchange for chloride), will cross endothelial cells, then type I pneumocytes, before entering the alveolar space. 8. Lower portion of right inferior lobe. 9. Superior portion of right inferior lobe. 10. A-3, B-3, C-2, D-1, E-3, F-2. 11. Scalene muscles, external intercostals, and sternomastoids. 12. Rectus abdominus, internal obliques, external obliques, transversus abdominus; the internal intercostals. PHYSIOLOGY 13. A-4, B-5, C-1, D-7, E-3, F-6, G-2, H-8. 14. 15. In pulmonary fibrosis, in pulmonary edema, and if surfactant production is insufficient. 16. The relaxed (R) form has a higher affinity for O2 than the taut (T) form.
file:///D|/USMLE%20Vids/Workbooks%202012/Chapter%2017%20-%20Respiratory%202012.txt[8/26/2013 2:13:27 AM]

17. Decreases; increases. 18. Effect Shift to the Left Shift to the Right Decreased 2,3-DPG v Decreased pH v Decreased temperature v Fetal hemoglobin v High altitude v Increased metabolic needs v Increased pH v Increased temperature v 19. COPD, mitral stenosis, recurrent thromboemboli, autoimmune disease, left-to-right shunt, sleep apnea, or living at high altitude. 20. A-a gradient = PAO2 PaO2; normal is 10-15 mm Hg. 21. Shunting, ventilation/perfusion mismatch, and fibrosis of the lungs. 22. High altitude and hypoventilation. 23. Ventilation/perfusion mismatch, diffusion limitation, and right-to-left shunt. 24. Hypoxemia, anemia, cyanide poisoning, carbon monoxide poisoning, and decreased cardiac output. 25. Vein < artery < alveolus. 26. Vein < alveolus < artery. 27. Alveolus < vein < artery. 28. As bicarbonate, bound to hemoglobin as carbaminohemoglobin, and dissolved. 29. Carbonic anhydrase. 30. The Haldane effect.
file:///D|/USMLE%20Vids/Workbooks%202012/Chapter%2017%20-%20Respiratory%202012.txt[8/26/2013 2:13:27 AM]

31. The Bohr effect.

32. Effect Response to Altitude Response to Exercise Decreased pH v Increased 2,3-DPG v Increased CO2 production v Increased erythropoietin v Increased O2 consumption v Increased mitochondria v Increased pulmonary blood flow v Increased renal excretion of bicarbonate v Increased ventilation v v More uniform V/Q ratio from apex to base v

file:///D|/USMLE%20Vids/Workbooks%202012/Chapter%2017%20-%20Respiratory%202012.txt[8/26/2013 2:13:27 AM]

Right ventricular hypertrophy v

PATHOLOGY 33. Fat, air, thrombus, bacteria, amniotic fluid, and tumor. 34. Stasis, hypercoagulability, and endothelial damage. 35. A = amniotic fluid; B = fat; C = air; D = thrombus. 36. A-2, B-3, C-1, D-4, E-3, F-2, G-1, H-2, I-3, J-1, K-4, L-3. 37. >80%. 38. Asbestosis. 39. Maternal diabetes, cesarean delivery, and premature birth. 40. Trauma, sepsis, shock, gastric aspiration, uremia, acute pancreatitis, and amniotic fluid embolism. 41. A form of sleep apnea that results from lack of respiratory effort. 42. A form of sleep apnea in which there is a drive to breathe but respiration is prevented because of mechanical airway obstruction (usually from obesity). 43. A-3, B-2, C-4, D-1. 44. Adrenals, brain, bone, and liver. 45. A-1, B-4, C-6, D-6, E-5, F-2, G-5, H-5, I-5, J-1, K-6, L-3, M-5, N-4, O-2. 46. Ptosis, miosis, anydrosis 47. A-3, B-3, C-1, D-1 or 2, E-3, F-3, G-3, H-1, I-2, J-1. 48. S. aureus, Bacteroides, Fusobacterium, and Peptostreptoccocus. 49. CHF, nephrotic syndrome and hepatic cirrhosis. 50. Malignancy, pneumonia, collagen vascular disease, and trauma. PHARMACOLOGY 51. Second-generation H1-blockers are far less sedating because their CNS penetration is much lower than that of first-generation agents. 52. Albuterol relaxes bronchial smooth muscle through its agonism of 2-adrenergic receptors. 53. Muscarinic antagonists. 54. (Modified, with permission, from Katzung BG, Trevor AJ. Pharmacology: Examination & Board Review, 5th ed. Stamford, CT: Appleton & Lange, 1998: 159, 161.) 55. It is a competitive antagonist of endothelin-1 at the endothelin (ET-A) and endothelin-B (ET-B) receptors. 56. It antagonizes NMDA glutamate receptor and acts as a non-competitive channel blocker.

file:///D|/USMLE%20Vids/Workbooks%202012/Chapter%2017%20-%20Respiratory%202012.txt[8/26/2013 2:13:27 AM]

Anda mungkin juga menyukai